Sunteți pe pagina 1din 130

MEDICINA *5 Bilanţul biologic la pacientul cu sincopă

- propuneri intrebari - urmăreşte depistarea următoarelor


modificări, cu excepţia
Tema nr. 1 A. hipercalcemie
Starea de rău, pierderea cunoştinţei, criza comiţială B. supradozaj digitalic
la adult
C. diskaliemie
Bibliografie asociata temei:
D. necroza miocardică
Book Des ECN, editia in limba romana, redactor
Laurent Karila, 2011 E. creşterea D-Dimerilor
pag. 27-29 (pag. 28)

*1 În care din următoarele situaţii de pierdere a 6 Lipotimiile şi sincopele pot avea următoarele
stării de cunoştinţă nu este necesară cauze cardiace
spitalizarea A. disautonomia neurovegetativă
A. scurtă pierdere a cunoştinţei asociată unor B. fibrilaţia ventriculară
anomalii neurologice
C. stenoza aortică strânsă
B. sincopa vasovagală tipică fără anomalie ECG
D. embolia pulmonară masivă
C. scurtă pierdere a cunoştinţei de cauză aritmică
suspectată E. disfuncţia sinusală
D. sincopa vasovagală repetitivă (pag. 27)
E. sincopa de etiologie necunoscută 7 Cauze vasculare de lipotimie sau sincopă
(pag. 28) sunt
A. hipersensibilitatea sinocarotidiană
*2 Elementul definitoriu al pierderii de
cunoştinţă din sincopă este B. hipertensiunea arterială pulmonară severă
A. prezenţa prodromului C. disautonomia neurovegetativă
B. revenirea lent progresivă D. furtul de arteră subclavie
C. debutul brusc E. embolia pulmonară masivă
D. caracterul complet (pag. 27)
E. pierderea de urină 8 Sincopa vasovagală tipică
(pag. 27) A. nu necesită spitalizare dacă nu sunt anomalii
ECG
*3 Cauze cardiace de lipotimie şi sincopă sunt
următoarele, cu excepţia B. nu necesită nicio explorare
A. stenoza aortică strânsă C. necesită evaluare prin ecografie cardiacă
B. cardiomiopatia hipertrofică obstructivă D. este de cauză vasculară
C. mixom atrial stâng E. necesită înregistrare Holter de 24 ore
D. tromboza de proteză valvulară mecanică (pag. 27-29)

E. hipotensiunea arterială 9 Lipotimiile şi sincopele pot fi de cauză


(pag. 27) A. cardiacă
*4 Cauze vasculare de lipotimie şi sincopă sunt B. metabolică
următoarele, cu excepţia C. neurologică
A. furtul de arteră subclavie D. reflexă
B. hipersensibilitatea sinocarotidiană E. vasculară
C. hipertensiunea arterială pulmonară severă (pag. 27)
D. disautonomia neurovegetativă
E. hipotensiunea arterială
(pag. 27)

Pag. 1 din 130


10 Explorările de a doua intenţie utile la *4 Stadiul II al Clasificării Killip se
pacienţii cu sincopă au următoarele caracterizează prin
caracteristici
A. Infarct miocardic complicat cu tulburări de ritm
A. înregistrarea Holter de 24 ore are valoare ventricular
predictivă negativă mare B. Infarct miocardic complicat cu raluri crepitante
B. ecografia cardiacă depistează cardiopatia care nu depăşesc jumătate din ariile pulmonare
ischemică, dilatativă sau hipertrofică C. Infarct miocardic complicat cu edem pulmonar
C. coronarografia poate fi indicată la anumiţi pacienţi acut
D. testul mesei înclinate permite diagnosticarea D. Infarct miocardic cu şoc cardiogen
sincopelor vasovagale E. Infarct miocardic cu hipotensiune şi fără raluri de
E. electrocardiograma poate identifica mecanismul stază pulmonară
sincopei (pag. 71)
(pag. 29)
*5 In evoluţia unui infarct miocardic acut
Tema nr. 2 apariţia unui suflu holosistolic în spiţe de
Boala cardiacă ischemică roată este sugestivă pentru

Bibliografie asociata temei: A. Insuficienţă mitrală ischemică


Book Des ECN, editia in limba romana, redactor B. Infarct de ventricul drept asociat unui infarct
Laurent Karila, 2011 inferior
pag. 59-74, 127-137 * fara denumiri comerciale pag. C. Ruptura peretelui liber ventricular
66, 70, 71, 73, 74, 127, 129, 130, 132, 133, 134,
D. Ruptura septului interventricular
135, 136
E. Embolie pulmonară cu regurgitare tricuspidiană
*1 Etiologia anginei pectorale stabile are în 95% severă
din cazuri drept cauză ateroscleroza (pag. 71)
coronariană şi în 5% din cazuri următoarele,
exceptând una 6 Diagnosticul de anevrism ventricular stâng
la un pacient cu infarct miocardic acut se
A. coronarită
bazează pe următoarele
B. anemie
A. Persistenţa supradenivelării segmentului ST
C. pericardită peste 3 săptămîni
D. tahicardie B. Fenomene de insuficienţă ventriculară stângă
E. colaps C. Apariţia Sindromului Dressler
(pag. 64-65) D. Tulburări de ritm ventricular recurente
*2 Supradenivelarea persistentă de segment ST E. Suflu sistolic de insuficienţă mitrală ischemică
prezentă în derivaţiile V1,V2,V3,DII,DIII,aVF, nou apărut
corespunde unui infarct miocardic acut (pag. 72)
A. anteroseptal 7 Complicaţii mecanice precoce ale unui
B. inferior şi anterior infarct miocardic acut de ventricul stâng sunt
C. inferior şi anteroseptal A. Ruptura peretelui liber ventricular
D. septal profund B. Sindromul Dressler
E. inferior şi posterior C. Insuficienţa mitrală ischemică
(pag. 69) D. Tromboembolismul pulmonar
*3 La 2 ore de la debutul durerii, pentru E. Blocurile atrio-ventriculare
diagnosticul pozitiv al unui sindrom (pag. 71)
coronarian acut, dintre markerii de necroză
miocardică se poate utiliza
A. Troponina I
B. Troponina T
C. Mioglobina
D. CK-MB
E. ASAT
(pag. 69)

Pag. 2 din 130


8 Indicaţia de cardiostimulare temporară la un Tema nr. 3
pacient cu infarct miocardic anterior se Hipertensiunea arterială esenţială
impune în prezenţa
Bibliografie asociata temei:
A. BAV de gradul 2 cu sediul infrahisian
Book Des ECN, editia in limba romana, redactor
B. BAV de gradul 3 cu sediul infrahisian Laurent Karila, 2011
C. Bloc complet de ramură stângă cu ritm de pag. 52-58 * fara prevalente in Franta pag. 52, fara
fibrilaţie atrială denumiri comerciale pag. 55, 57
D. Alternanţă de bloc de ramură stângă cu bloc de
*1 Hipertensiunea arteriala de gradul II se
ramură dreaptă
defineste prin urmatoarele valori tensionale
E. Asocierea bloc de ramură dreaptă cu hebloc
anterior stâng A. TA sistolica 140-159 mm Hg
(pag. 71) B. TA diastolica 90-99 mm Hg
C. TA sistolica 160-179 mm Hg
9 Sindromul coronarian acut fără
supradenivelare de segment ST se D. TA diastolica > 110 mm Hg
caracterizează prin următoarele afirmaţii E. TA sistolica > 180 mm Hg
A. Apare în special la pacienţii tineri fără circulaţie (pag. 52)
colaterală eficientă
*2 HTA este un factor de risc cardiovascular
B. Apare deseori la pacienţi cu leziuni trivasculare independent deoarece determina
severe şi complexe
A. risc de AVC crescut de 4 ori
C. Evoluează frecvent spre insuficienţă cardiacă
ischemică B. mortalitate cardiovasculara globala crescuta de 2
ori
D. Asociază frecvent complicaţii mecanice gen
ruptura de perete liber ventricular C. risc de aparitie a insuficiente cardiace crescut de
7 ori
E. Apare consecutiv rupturii plăcii de aterom pe
fondul unei circulaţii colaterale deficitare D. risc de arteriopatie a membrelor inferioare si a
(pag. 72) aortei crescut de 3 ori
E. risc de aparitie a unei coronaropatii crescut de 2
* 10 Aspectul electrocardiografic sugestiv pentru ori
sindromului Prinzmetal este sugerat de
(pag. 52)
următoarele
A. Supradenivelare de segment ST difuză tranzitorie *3 Sunt considerati ca factori de risc
cardiovascular
B. Supradenivelare de segment ST concavă în sus
A. Antecedentele familiale de accident
C. Supradenivelare de segment ST gigantă
cardiovascular precoce
convexă în sus, unde T gigante
B. Tabagismul oprit de peste 3 ani
D. Supradenivelare de segment ST corectată de
nitroglicerină C. Varsta < 50 ani la barbat si < 60 ani la femeie
E. Subdenivelare de segment ST focalizată la un D. Consumul redus de alcool
teritoriu vascular E. HDL-colesterol > 0,40 g/l (1 mmol/l), indiferent de
(pag. 60) sex
(pag. 53)

*4 Unul din semnele paraclinice de afectare a


organelor tinta (de rasunet ale HTA) este
A. Grosimea intima-medie < 0,9 mm
B. Hipertrofia ventriculara stanga
C. Velocitatea undei de puls carotido-femurale
(PWV) < 12 m/s
D. Albuminuria < 30 mg/zi
E. Clearance creatinina > 60 ml/min
(pag. 53)

Pag. 3 din 130


*5 Unul din semnele clinice de rasunet ale HTA 10 Medicamentele antihipertensive
este recomandate post infarct miocardic sunt
A. Sindromul metabolic A. Diuretice tiazidice
B. Boala hepatica B. Betablocante
C. Boala pulmonara C. Antialdosteronice
D. Boala cardiaca D. IEC sau ARA II daca exista intoleranta la IEC
E. Insuficienta venoasa cronica E. Anticalcice
(pag. 53) (pag. 55)

6 Examinarile sistematice efectuate pentru a Tema nr. 4


depista o etiologie curabila (HTA secundara), Insuficienţa cardiacă
a detecta alti factori de risc cardiovascular si
a depista afectarea organelor tinta sunt Bibliografie asociata temei:
Book Des ECN, editia in limba romana, redactor
A. Ecocardiografia transtoracica
Laurent Karila, 2011
B. Acidul uric seric pag. 84-92, 138-141, 1270-1272, 1346 * fara
C. Glicemia plasmatica denumiri comerciale (pag. 89, 90, 92, 138, 140, 141)
D. Microalbuminuria 1 Edemele de retenţie hidrosodată pot fi
E. Kaliemia fara garou cauzate de
(pag. 54) A. Insuficienţa cardiacă
7 Tratamentul nemedicamentos al HTA se B. Tromboflebită
refera la C. Insuficienţa hepatocelulară
A. Activitate fizica regulata D. Sindromul nefrotic
B. Reducerea consumului de lipide saturate E. Erizipel
C. Depistarea si tratamentul celorlalti factori de risc (pag. 1346)
D. Favorizarea consumului de alcool 2 Etiologia insuficienţei cardiace diastolice
E. Limitarea aportului de sare (5-6 g/zi) cuprinde
(pag. 54) A. Pericardita constrictivă
8 Tratamentul medicamentos al HTA B. Cardiomiopatia hipertrofică obstructivă
A. Debuteaza cu monoterapie sau biterapie cu doze C. Stenoza mitrală
mici pentru fiecare medicament D. Cardiopatia ischemică
B. Favorizeaza medicamentele administrate intr-o E. Stenoza aortică
singura priza, cu eficienta timp de 24 ore
(pag. 91)
C. Clasele terapeutice de utilizat in prima intentie
sunt aliskirenul, alfablocantele si 3 La un pacient cu insuficienţă cardiacă
antihipertensivele centrale antivitaminele K sunt indicate în următoarele
D. Cel mai important principiu este scaderea in mod cazuri
eficient a TA, oricare ar fi medicamentul utilizat A. dacă este în clasa NYHA IV
E. Medicamentele de utilizat in a doua intentie sunt: B. în caz de fibrilaţie atrială permanentă
diureticele, betablocantele, inhibitorii enzimei de
conversie, inhibitorii calcici si antagonistii C. dacă are proteză valvulară mecanică
receptorilor angiotensinei II D. în caz de tromb intracavitar
(pag. 54) E. dacă insuficienţa cardiacă este refractară la
medicaţia maximală
9 Medicamentele antihipertensive
recomandate in contextul insuficientei (pag. 90)
cardiace sunt
A. Anticalcice
B. IEC sau ARA II
C. Betablocante
D. Diuretice tiazidice
E. Antialdosteronice
(pag. 55)

Pag. 4 din 130


4 Indicatori de prognostic negativ în *9 Următoarea afirmaţie despre tratamentul cu
insuficienţa cardiacă sunt betablocante în insuficienţa cardiacă este
falsă
A. Hiponatremia
B. Sexul masculin A. reduce mortalitatea cardiovasculară

C. Complex QRS larg B. tratamentul se introduce progresiv, la distanţă de


o decompensare
D. Anemia
C. se poate folosi oricare dintre betablocante, cu
E. Obezitatea condiţia să fie selectiv
(pag. 89) D. reduce numărul de spitalizări
*5 Examenul cheie pentru diagnosticul E. reduce riscul de moarte subită
insuficienţei cardiace diastolice este (pag. 89-90)
A. Determinarea BNP (peptidul natriuretic tip B) sau 10 Contraindicaţii ale diureticelor
NT-proBNP (capătul amino-terminal al BNP)
antialdosteronice sunt
B. Radiografia toracică
A. Miastenia
C. Ecografia cardiacă transtoracică
B. Hiponatremia < 125 mmol/l
D. Anamneza şi examenul clinic
C. Insuficienţa hepatică severă
E. Electrocardiograma
D. Hiperkaliemia
(pag. 91)
E. Disfuncţia sistolică ventriculară stângă severă
*6 Următoarea afirmaţie despre tratamentul cu (pag. 140)
digitalice în insuficienţa cardiacă este greşită
A. reduce frecvenţa spitalizărilor Tema nr. 5
Tulburări de ritm şi de conducere
B. reduce mortalitatea
Bibliografie asociata temei:
C. digitalicele sunt utile pentru controlul frecvenţei
ventriculare la pacienţii cu fibrilaţie atrială Book Des ECN, editia in limba romana, redactor
Laurent Karila, 2011
D. sunt utile la pacienţii cu insuficienţă cardiacă
pag. 30-43
refractară în ciuda unui tratament maximal
E. în general sunt utilizate la pacienţii cu clasă III-IV *1 Care dintre următoarele afirmaţii despre
NYHA flutterul atrial este neadevărată
(pag. 90) A. Tahicardie regulată
*7 Medicaţia uzuală a insuficienţei cardiace B. Macroreintrare la nivelul atriului drept
clasă I-II NYHA nu include C. Activitatea atrială are o frecvenţă de peste 350
A. Betablocante bătăi pe minut
B. Inhibitori ai enzimei de conversie a angiotensinei D. Transmiterea la ventriculi poate fi de 2:1, 3:1, 4:1
C. Blocante ale canalelor de calciu E. Undele de activitate atrială se identifică cel mai
bine în DII,DIII,aVF
D. Antialdosteronice
(pag. 30)
E. Blocante ale receptorilor angiotensinei
(pag. 90) *2 Principala cauză etiologică a fibrilaţiei atriale
este reprezentată de
*8 Regimul igieno-dietetic în insuficienţa
A. valvulopatiile mitrale
cardiacă include următoarele, cu excepţia
B. valvulopatiile aortice
A. corecţia factorilor de risc cardiovascular
C. hipertiroidismul
B. regim sărac în sare (< 4 g/zi) în forma puţin
evoluată D. hipertensiunea arterială
C. restricţie hidrică (< 500 - 750 ml/zi) la pacienţii cu E. cardiopatia ischemică
edeme (pag. 38)
D. activitate fizică moderată, regulată
E. vaccinare antigripală, antipneumococică
(pag. 89)

Pag. 5 din 130


*3 Complicaţiile posibile ale fibrilaţiei atriale 7 Tahicardiile ventriculare sunt manifestări
permanente cuprind toate aceste complicaţii clinice frecvente în următoarele boli
cu excepţia
A. Sindromul Brugada
A. Complicaţiile tromboembolice B. Sindromul de QT lung
B. Sîngerările consecutive tratamentului C. Displazia de ventricul drept
anticoagulant
D. Sindromul Bouveret
C. Cardiomiopatia tahiaritmică
E. Sindromul de preexcitaţie tip WPW
D. Anevrismul ventricular
(pag. 30-31)
E. Insuficienţa cardiacă
(pag. 39) 8 Care dintre afirmaţiile următoare despre
tahicardia joncţională prin reintrare sunt
*4 Blocul atrioventricular de gradul I nu este adevărate
caracterizat de una din următoarele afirmaţii
A. circuitul de reintrare se află la nivelul nodului
A. Prezintă întârzierea conducerii la nivel nodal, atrioventricular
hisian sau infrahisian B. fascicolul His este obligatoriu implicat în
B. Ocazional o undă p este blocată apoi ciclul se formarea căii descendente
reia C. calea ascendentă este o cale accesorie
C. Alungirea intervalului PR peste 210 msecunde atrioventriculară
D. Este foarte rar simptomatic în absenţa altor D. poate fi susţinută sau nesusţinută
complicaţii E. fenomenul de încălzire este deseori prezent
E. Poate traduce o incetinire nodală benignă dar şi (pag. 31)
un bloc distal care poate evolua spre BAV
complet 9 Sindromul Brugada se defineşte prin
(pag. 41) următoarele

*5 Care dintre afirmaţiile de mai jos despre A. cordul este de regulă indemn
hemiblocul posteroinferior stâng este corectă B. aspectul ECG este de bloc de ramură dreaptă
A. Durata complexului QRS depăşeşte 120 C. se manifestă de regulă cu fibrilaţie şi flutter atrial
msecunde D. este consecinţa unei mutaţii la nivelul canalului
B. Axul complexului QRS este la peste – 45 grade de sodiu
C. Aspectul complexului QRS are morfologie de tip E. segmentul ST este supradenivelat în derivaţiile
S3, Q1 V1,V2,V3
D. Nu poate genera bloc complet atrioventricular (pag. 32)
E. Etiologia cea mai frecventă este cea ischemică 10 Displazia aritmogenă de ventricul drept se
(pag. 43) caracterizează prin

6 Care dintre următoarele afirmaţii despre A. tulburări de ritm ventriculare maligne


extrasistolele ventriculare maligne sunt B. bloc complet de ramură dreaptă pe ECG
adevărate C. extrasistole ventriculare cu aspect de bloc de
A. Se pot manifesta cu lipotimie, sincopă sau ramură dreaptă
moarte cardiacă subită D. undă epsilon prezentă
B. Se agravează la efort E. ventriculul drept este akinetic cu aspect
C. Funcţia ventriculului stîng poate fi normală trabeculat la examenul ecocardiografic,
scintigrafic sau RMN
D. Electrocardiograma poate fi normală în afara
episodului aritmic (pag. 32)
E. Potenţialele tardive sunt de regulă prezente
(pag. 34)

Pag. 6 din 130


Tema nr. 6 *5 Etiologia distrofica a insuficientei mitrale
Valvulopatii mitrale, aortice cronice prezinta una din urmatoarele
caracteristici
Bibliografie asociata temei:
A. Valve si cordaje ingrosate, calcificate, retractate,
Book Des ECN, editia in limba romana, redactor
comisuri fuzionate
Laurent Karila, 2011
pag. 93-96, 108-119, 124-126, 1123-1124 * fara B. Degenerescenta mixoida a valvelor, care se
denumiri comerciale pag. 124, 125, 126 ingroasa, devin redundante
C. Dilatarea inelului mitral
*1 Insuficienta aortica cronica de etiologie
D. Insuficienta este frecvent asociata unei stenoze
reumatismala prezinta una din urmatoarele
caracteristici E. Este asociata unui mixom al atriului stang
A. afectarea distrofica a valvei aortice si/sau a (pag. 112)
inelului aortic si/sau a aortei ascendente
6 Semnele fizice care pot fi prezente in
B. este etiologia cea mai frecventa in tarile insuficienta mitrala sunt
industrializate
A. La palpare soc “in dom”, descris de Bard,
C. valvele sunt ingrosate, apoi calcificate, deplasat in jos si la stanga
comisurile fuzioneaza, se retracta, iar mobilitatea
lor este diminuata B. Suflu holosistolic, maximal in focarul mitral, in jet
de vapori, de intensitate uniforma,,care iradiaza
D. se asociaza cu bicuspidia aortica in axila
E. apare frecvent in sindromul Marfan C. Semne de insuficienta cardiaca dreapta
(pag. 108) D. Z3 supraadaugat
*2 Insuficienta aortica acuta poate avea una din E. Suflul diastolic Flint in focarul mitral
urmatoarele etiologii (pag. 113)
A. bicuspidia aortica
7 Ecografia Doppler cardiaca transtoracica
B. traumatismul toracic efectuata la pacientii cu insuficienta mitrala
C. sindromul Laubry si Pezzi poate evidentia
D. lupusul eritematos diseminat A. Dilatarea aortei ascendente
E. spondilartrita anchilozanta B. Dilatarea atriului stang
(pag. 108) C. Dilatarea ventriculului stang
D. Directia jetului regurgitant in venele pulmonare
*3 Unul din semnele stetacustice care apare in
insuficienta aortica este E. Dimensiunea cavitatilor drepte
A. Click mezosistolic urmat de un suflu telesistolic (pag. 114)
B. Z2 diminuat sau abolit 8 Complicatiile insuficientei mitrale sunt
C. Suflu holosistolic, maximal in focarul mitral, reprezentate de
piolant, care iradiaza in axila A. Endocardita bacteriana
D. Suflu mezosistolic ejectional, aspru, maximal in B. Insuficienta cardiaca
focarul aortic, iradiat la nivelul vaselor gatului
C. Tulburarile de conducere de grad inalt
E. Suflu protodiastolic fin, dulce, cu caracter
aspirativ, cu maxim in focarul aortic D. Fibrilatia atriala si accidentele embolice
(pag. 109) E. Tulburarile de ritm ventriculare si moartea subita
(pag. 114)
*4 Semnul Durozier prezent in insuficienta
aortica inseamna
A. Dublu suflu intermitent crural
B. Marirea presiunii arteriale diferentiale
C. Balansarea capului la fiecare bataie cardiaca
D. Hippus pupilar
E. Puls capilar
(pag. 109)

Pag. 7 din 130


9 Stenoza aortica degenerativa are *3 Tratamentul chirurgical al endocarditei
urmatoarele trasaturi infecţioase nu este indicat în următoarele
situaţii:
A. Este cauza cea mai frecventa la subiectii de
varsta medie (50-60 ani) A. infecţie întinsă sau necontrolată
B. Exista calcificari al valvelor si ale inelului aortic B. pentru prevenţia riscului embolic,
care se pot intinde pe sept
C. în situaţiile cu risc hemodinamic,
C. Comisurile sunt fuzionate
D. în şoc, EPA, insuficienţă cardiacă,
D. Este in scadere ca frecventa in ultimii douazeci
E. dacă sub un tratament antibiotic corect condus,
de ani
dimensiunile vegetaţiilor nu se reduc şi
E. Valvele sunt ingrosate, rigide, cu o mobilitate hemoculturile rămân pozitive după 3 zile de
limitata antibioterapie.
(pag. 116) (pag. 102)
10 Semnele clinice prezente in stenoza aortica *4 Următoarea afirmaţie legată de
sunt antibioprofilaxia endocarditei infecţioase nu
A. Angorul este corectă:

B. Dispneea A. toţi pacienţii valvulari trebuie să primească un


carnet de profilaxie a endocarditei pe care
C. Sincopa trebuie sa-l prezinte medicului sau
D. Hemoptiziile stomatologului la fiecare consultaţie,
E. Palpitatiile B. prevenţia începe prin măsuri stricte de igenă,
(pag. 117) C. antibioprofilaxia este cea mai importantă
pentru prevenţia endocarditei infecţioase,
Tema nr. 7 D. în prezent ultimele recomandări inernaţionale
Endocardita infecţioasă menţin antibioprofilaxia sistematice indiferent de
Bibliografie asociata temei: grupa de risc din care face parte pacientul,
Book Des ECN, editia in limba romana, redactor E. antibioprofilaxia trebuie să fie de acum înainte
Laurent Karila, 2011 rezervată doar pacienţilor din grupa A supuşi
pag. 97-103 * fara denumiri comerciale pag. 101, procedurilor celor mai riscante.
102 (pag. 103)

*1 Care din următoarele complicaţii cardiace *5 Care din următoarele afirmaţii legate de
nu sunt caracteristice endocarditei endocardita infecţioasă este corectă:
infecţioase: A. nu există examen specific pentru
A. insuficientă cardiacă, diagnosticarea endocarditei infecţioase, pentru
acest motiv au fost stabilite criterii diagnostice
B. infarct miocardic prin embolie coronară, precise (criterii Duke).
C. abces septal, B. mortalitatea spitalicească în endocardita
D. distrucţie valvulară, infecţioasă este de 50%,
E. hipertensiune pulmonară. C. criteriile Duke constau din 3 criterii majore şi 5
criterii minore,
(pag. 100)
D. endocardita infecţioasă nu poate surveni pe un
*2 Tratamentul endocarditei infecţioase pe cord fără o afectare prexistentă,
proteză, cu hemoculturi negative se iniţiază E. toate cardiopatiile prezintă acelaşi risc de
cu: endocardită.
A. asociere de vancomicina cu rifampicină şi (pag. 99)
gentamicină,
6 Din grupa A, a cardiopatiilor cu risc crescut
B. amoxicilină 200 mg/kg/zi sau aminozide:
gentamicină 3 mg/kg/zi, de a dezvolta endocardită infecţioasă fac
parte:
C. penicilină G sau amoxicilină 200 mh/kg/zi sau
aminozide gentamicină 3 mg/kg/zi, A. proteze valvulare,
D. doxiciclina 100 mg de 2 ori / zi în asociere cu B. cardiopatii congenitale cianogene neoperate,
ofloxacină 400 mg/zi C. cardiomiopatie obstructivă,
E. vancomicina 30 mg/kg/zi în asociere cu D. bicuspidie aortică,
rimfampicină.
E. PVM cu IM.
(pag. 102)
(pag. 97)

Pag. 8 din 130


7 Semnele periferice de endocardită Tema nr. 8
infecţioasă sunt: Pericardita acută
A. splenomegalia, Bibliografie asociata temei:
B. purpura peteşială, cutaneomucoasă, descrisă Book Des ECN, editia in limba romana, redactor
clasic la nivelul feţei, Laurent Karila, 2011
C. noduli Roth localizaţi la nivel abdominal, pag. 104-107
D. plăci eritematoase palmoplantare Janeway,
*1 În pericadita acută electrocardiograma:
E. fals panariţiu Osler.
A. este normală în majoritatea cazurilor,
(pag. 98)
B. evoluează în funcţie de patru stadii,
8 În endocardita infecţioasă se recomandă: C. prezintă caracteristic supradenivelarea
A. recoltarea de hemoculturi înaintea oricărei segmentului ST cu semn în oglindă
antibioterapii, D. prezintă alternanţă electrică frecventă a
B. recoltarea bilanţului inflamator: complexului QRS ce nu se corelează cu
hemoleucogramă, VSH, CRP, electroforeza mărimea revărsatului pleural,
proteinelor plasmatice, E. supradenivelarea segmentului ST este concavă
C. proteinurie pe 24 de ore, în jos ( criterii discriminante faţă de
supradenivelarea segmentului ST de cauză
D. recoltarea obligatorie de markeri imunologici ischemică).
indiferent de formele de endocardită acute sau
subacute, (pag. 104)
E. dacă nici un germen nu este izolat se fac *2 În pericadita acută electrocardiograma:
recoltări pe medii speciale (Bartonella, Coxiella ),
serologii Chlamidiae şi germeni atipici. A. este normală în majoritatea cazurilor,
(pag. 98) B. evoluează în funcţie de patru stadii,
C. prezintă caracteristic supradenivelarea
9 Care din următoarele afrmaţii nu este segmentului ST cu semn în oglindă,
corectă:
D. prezintă alternanţă electrică frecventă a
A. endocardită posibilă: 1 criteriu major plus unul complexului QRS ce nu se corelează cu
minor sau 3 criterii minore, mărimea revărsatului pleural,
B. endocardită sigură: 5 criterii minore, E. supradenivelarea segmentului ST este concavă
C. endocardită sigură: 1 criteriu major plus 2 în jos ( criterii discriminante faţă de
criterii minore, supradenivelarea segmentului ST de cauză
ischemică).
D. endocardită sigură: 2 criterii majore,
(pag. 104)
E. diagnosticul de endocardită sigură nu se poate
pune doar pe examenul anatomopatologic, sau *3 Care din următoarele afirmaţii sunt false:
pe cultură de valve pozitivă.
A. pericardita acută reprezintă o inflamaţie a
(pag. 99) pericardului complicată sau nu, cu un revărsat
pericardic,
10 Criteriile Duke minore sunt:
B. un revărsat pericardic este întotdeauna datorat
A. febră peste 38 de grade, unei pericardite,
B. fenomene vasculare: embolie, anevrism C. tamponada este o urgenţă terapeutică şi
micotic, purpură Janeway, etc., diagnostică,
C. fenomene imunologice: noduli Osler, pete D. un revărsat pericardic abundent cu apariţie
Roth, factor reumatoid, etc., rapidă poate conduce la o tamponadă,
D. hemoculturi pozitive pentru endocardită E. pericardita uscată reprezintă inflamarea
infeţioasă, pericardului neînsoţită de revărsat pericardic.
E. predispoziţie: valvulopatie sau altă condiţie (pag. 104)
cardiacă favorizantă sau toxicomanie iv.
(pag. 99)

Pag. 9 din 130


*4 Frecătura pericardică se caracterizează 9 Drenajul chirurgical este indicat în
prin: pericardita acută în următoarele situaţii:
A. se aude mai bine în decubit, A. tamponadă cardiacă,
B. zgomot profund, strict în diastolă, B. revărsat voluminos, necompresiv persistent
C. la fel ca şi frecătura pleurală nu persistă în sub tratament,
apnee, C. revărsat voluminos, necompresiv recidivant,
D. este constantă în timp, D. trebuie efectuat obligatoriu în orice formă de
E. zgomot superficial sistolodiastolic. pericardită,

(pag. 104) E. revărsat compresiv.


(pag. 106)
*5 Care din următoarele elemente nu fac parte
din tratamentul simptomatic al pericarditei 10 Următoarele afirmaţii legate de pericardita
acute: acută nu sunt adevărate:
A. repaus, concediu medical, A. etiologia virală este cea mai fecventă,
B. tratamentul antiinflamator nesteroidian, B. pericadita de etiologie neoplazică are o evoluţie
rapidă, zgomotoasă,fiind adeseori diagnosticată
C. continuarea obligatorie a tratamentului
în fază precoce,
anticoagulant,
C. reumatismul articular acut este o cauză
D. aspirină în doze descrescătoare timp de trei
frecventă de pericardită acută,
săptămâni,
D. pericadita acută de etiologie TBC este rară şi
E. nu este nevoie de spitalizare în formele
complică o tuberculoză pulmonară,
benigne.
(pag. 106) E. pericardita acută idiopatică corespunde cel mai
adesea unei cauze virale nediagnosticate.
*6 Diagnosticul diferenţial al tamponadei (pag. 105, 106)
cardiace se face cu:
11 Semnele clinice de tamponadă cardiacă
A. embolia pulmonară masivă, sunt:
B. infarctul de ventricul stăng,
A. A.şocul cardiogen
C. pleurezia,
B. B.semne de edem pulmonar,
D. stenoza aortică severă,
C. C.puls paradoxal,
E. endocardita infecţioasă.
D. D.semne drepte majore,
(pag. 107)
E. E.creşterea presiunii arteriale în inspir cu peste
7 În pericardita acută durerea are 10 mmHg.
următoarele caracteristici: (pag. 107)
A. prezintă strict localizare retrosternală,
B. acentuată de efort şi sensibilă la trinitrină,
C. are caracter de arsură, constricţie,
D. este intensificată de respiraţia profundă şi în
timpul efortului de tuse,
E. se calmează în decubit.
(pag. 104)

8 În primul stadiu de evoluţie a pericarditei


acute întâlnim următoarele modificări pe
electrocardiogramă:
A. supradenivelarea segmentului ST, fără imagine
în oglindă,
B. unde T negative,
C. segment ST supradenivelat, difuz, cu
concavitatea în sus,
D. subdenivelarea difuză a segmentului ST,
E. segment ST de aspect normal şi unde T
aplatizate.
(pag. 104)

Pag. 10 din 130


Tema nr. 9 4 Printre cauzele de ischemie acută a
Patologia aortei, arterelor periferice şi patologia membrelor sunt:
venoasă a membrelor inferioare: arteriopatia A. embolia cu sursă din cavitățile cardiace stângi
obliterană a aortei şi a membrelor inferioare,
B. embolia provenită dintr-un anevrism arterial sau
ischemia acută, insuficienţa venoasă cronică, varice
leziune ateromatoasă ulcerată situată în amonte
Bibliografie asociata temei:
C. tromboză in situ pe o leziune atrosclerotică
Book Des ECN, editia in limba romana, redactor preexistentă
Laurent Karila, 2011
D. trombocitopenia după tratament cu heparină
pag. 75-80, 81-83, 120-123 * fara denumiri
comerciale pag. 80 E. hipertensiunea arterială
(pag. 81)
1 Ischemia permanentă cronică a membrelor
inferioare denumită și ischemia critică: 5 In ischemia acută a membrelor:
A. se definește prin durere de decubit sau tulburări A. leziunile devin ireversibile după 12 ore
trofice ale piciorului B. membrul afectat este palid, rece, cu puls abolit la
B. se însoțește de un index al presiunii sistolice unul sau mai multe niveluri
gleznă/braț de 0,9-1,3 C. apare alungirea timpului de recolorare cutanată
C. asociază o presiune sistolică la haluce sub 30 la membrul afectat
mmHg D. aparița hipoesteziei și scăderea forței musculare
nu sunt semne de gravitate
asociază o presiune sistolică la haluce sub 30
mmHg E. abolirea pulsului popliteu indică obstrucția
arterei femurale superficiale sau a arterei poplitee
D. asociază o presiune sistolică la gleznă între 50-
70 mmHg (pag. 81,82)
E. asociază o presiune sistolică la haluce peste 30 *6 Tratamentul de urgență în ischemia acută a
mmHg membrelor inferioare:
(pag. 78) A. este numai medicamentos (antialgice, heparină,
2 Sindromul Leriche se caracterizează prin: vasodilatatore)
B. embolectomia cu sonda Fogarty se face numai
A. durere lombară, hematurie, insuficiență renală
pe leziuni ateromatose ce realizează stenoze
rapid progresivă
lungi, distale
B. durere fesieră
C. revascularizarea prin by pass este tratamentul
C. claudicație a celor două membre inferioare ischemiei acute pe artere patologice
D. impotență D. tromboliza in situ și tromboaspirația nu sunt
E. apare prin afectarea aterosclerotică aortoiliacă indicate

(pag. 77) E. aponevrotomia de descărcare se face de la


început
3 Următoarele afirmații referitoare la (pag. 82,83)
tratamentul în arteriopatia cronică obliterantă
a membrelor inferiore sunt adevărate: *7 Arteriopatia obliterantă a membrelor
inferioare:
A. controlul factorilor de risc cardiovascular este
obligatoriu în toate stadiile bolii A. la o valoare mai mică de 10 mmHg a presiunii
transcutanate de oxigen prognosticul de
B. tratamentul cu prostaglandine nu este indicat în
viabilitate tisulară este încă bun
nici un stadiu al bolii
B. în stadiul III Leriche durerea este în decubit, se
C. angioplastia și endarteriectomia sunt utile în
calmează în poziție declivă și se însoțește de
stadiile 3,4 pentru stenoze strânse, proximale,
tulburări trofice distale
scurte, puțin calcificate
C. valoarea indexul gleznă- braț între 0,9 – 1,3
D. pontajul (by-passul) nu este indicat în stenoze
indică o formă compensată de arteriopatie
complexe, întinse cu dilatare anevrismală
D. o presiune transcutanată între 10-30 mmHg
E. simpatectomia se folosește la pacienții cu dureri
indică o bună compensare metabolică a
de decubit
arteriopatiei
(pag. 79, 80)
E. este cauzată de ateroscleroză la pacientul
vârstnic cu factori de risc cardiovascular
(pag. 77,78)

Pag. 11 din 130


*8 Pacientul cu ischemie acută a membrelor Tema nr. 10
prin tromboză pe un vas cu leziune Dispneea acută şi cronică, BPOC
ateromatoasă:
Bibliografie asociata temei:
A. nu are factori de risc cardiovascular prezenți
Book Des ECN, editia in limba romana, redactor
B. debutul simptomelor este subacut cu ischemie Laurent Karila, 2011
mai puțin severă și modificarea pulsului și la alte pag. 142-144, 161-168 * fara denumiri comerciale
nivele pag. 166
C. apariția hipoesteziei și a deficitului motor nu este
un indicator de gravitate *1 Următoarele cauze pot determina dispneea
laringiană:
D. are o cardiopatie emboligenă
E. arteriografia evidențiază oprire netă cu aspect de A. Edemul pulmonar acut
cupolă a substanței de contrast B. Edemul Quincke
(pag. 82) C. Pleurezia masivă
*9 Insuficiența venoasă cronică: D. Pneumotoraxul spontan
A. este întotdeauna secundară trombozei venoase E. Pneumopatia acută infecțioasă
profunde (pag. 143)
B. varicele pot apare prin compresie pelvină
*2 Următoarele cauze nu determină apariția
tumorală sau prin agenezia venelor profunde
dispneei cronice:
C. incontinența valvulară poate fi apreciată clinic
prin proba Trendelenburg (transmiterea vibrației A. Insuficiența cardiacă stângă
prin percuție de- a lungul venei) B. Pericardita cronică constrictivă
D. fenomenele hemodinamice nu se însoțesc de C. Epiglotita infecțioasă
activare leucocitară cu inflamație locală
D. Anemia cronică
E. ulcerul maleolar indică insuficiență venoasă
E. Bolile neuromusculare
minoră
(pag. 143)
(pag. 121)
*3 Mai multe afecţiuni pot determina dispnee
* 10 Tratamentul în insuficiența venoasă cronică:
acută fără zgomote anormale, cu excepția:
A. venotonicele tratează jena funcțională și previn
A. Embolia pulmonară;
complicațiile cutanate
B. strippingul venei safene interne cu ligatura B. Edemul glotei;
crosei, a venelor perforante, scleroza venelor C. Anemia acută;
mici previn complicațiile cutanate D. Acidoza metabolică;
C. drenejul postural și contența elastică sunt
E. Tamponada cardiacă.
singurele eficiente
(pag. 143)
D. terapia chirurgicală prin stripping împiedică
recidiva *4 Emfizemul panlobular se caracterizează prin
E. nu se indică contenția elastică cu ciorap sau A. VEMS/CV scăzut;
benzi elastice
B. CPT scăzută;
(pag. 123)
C. PaO2 normală;
D. PaCO2 normală;
E. Complianță crescută.
(pag. 162)

*5 Care dintre următoarele explorări nu fac


parte din diagnosticul BPOC:
A. Radiografia toracică;
B. Probe funcționale respiratorii;
C. Electrocardiogramă;
D. Gazometria;
E. Ecografia Doppler vascular.
(pag. 162)

Pag. 12 din 130


6 Care dintre următoarele explorări trebuie Tema nr. 11
realizate de primă intenție în caz de dispnee: Tusea şi hemoptizia la adult
A. Radiografia toracică față; Bibliografie asociata temei:
B. Radiografia toracică profil; Book Des ECN, editia in limba romana, redactor
C. Gazometria arterială; Laurent Karila, 2011
pag. 145-147, 181-182 * fara denumiri comerciale
D. EKG;
pag. 182
E. Fibroscopia bronșică.
(pag. 142) *1 Principalele cauze ale tusei acute, cu o
excepție, sunt
7 Precizaţi care sunt semnele de gravitate într-
A. Infecțiile căilor aeriene superioare;
o dispnee acută:
B. Embolia pulmonară;
A. Respirația abdominală;
C. Astmul bronșic;
B. Semne neuropsihice;
D. Emfizemul pulmonar;
C. Bradicardia;
E. Edemul pulmonar.
D. Hipertensiunea arterială;
(pag. 145)
E. Bradipnee.
(pag. 142) *2 Precizaţi care dintre următoarele explorări nu
se efectuează de primă intenție într-o
8 Precizaţi care sunt explorările ce se hemoptizie:
efectuează pentru orientarea diagnosticului
A. Hemograma completă;
în cazul unei dispnei acute:
B. Examenul de spută;
A. PFR;
C. Radiografia toracică;
B. Ecografia cardiacă;
D. Fibroscopia bronșică;
C. Ecografie abdominală;
E. CT toracică.
D. CT torace;
(pag. 181)
E. Cateterism cardiac drept.
(pag. 143) *3 Care dintre următoarele afirmații nu sunt
adevărate în cazul unei hemoptizii:
9 Emfizemul centrolobular se caracterizează
A. Orice hemoptizie necesită supraveghere
prin:
spitalicească;
A. VEMS/CV scăzut; B. Reprezintă eliminarea de sânge din căile aeriene
B. CPT subnormală; subglotice;
C. PaO2 scăzută; C. Hemoptizia masivă nu este precedată de
hemoptizii santinele;
D. PaCO2 scăzută;
D. Apare în timpul unui efort de tuse;
E. DLCO crescută.
E. Sângele eliminat este roşu şi aerat.
(pag. 162)
(pag. 181)
10 BPOC foarte severă se caracterizează prin
*4 La un fost pacient tuberculos următoarele
A. VEMS/CV sub 0,70; cauze pot determina apariția unei hemoptizii,
B. VEMS sub 30% din cel teoretic; cu excepția:
C. VEMS sub 50% din valoarea prezisă în absența A. Recidiva BK;
insuficienței respiratorii;
B. Dilatarea postcicatriceală a bronhiilor;
D. 30%≤VEMS›50% decât cel teoretic;
C. Aspiraţia de corpi străini;
E. VEMS sub 50% din valorile prezise cu PaO2 sub
D. Cancerul bronhopulmonar pe cicatrice;
60 mmHg.
(pag. 163) E. Bronholitiaza.
(pag. 182)

Pag. 13 din 130


*5 În hemoptiziile de mare abundență nu se 10 Hemoragia alveolară se întâlnește în
efectuează: următoarele afecțiuni:
A. Administrarea de perfuzii macromoleculare; A. Poliangeita microscopică;
B. Administrarea de terlipresină; B. Sindromul Goodpasture;
C. Embolizare; C. LES;
D. Oxigenoterapie nazală în flux puternic; D. Tuberculoza pulmonară;
E. Administrarea de hipotensoare. E. Anevrism arterio-venos al bolii Rendu-Osler.
(pag. 182) (pag. 181)

6 Care dintre următoarele afirmații definesc o Tema nr. 12


hemoptizie masivă: Alergiile respiratorii la adult (rinita, astmul bronsic)
A. Deces în câteva minute; Bibliografie asociata temei:
B. Intubație în extremă urgență; Book Des ECN, editia in limba romana, redactor
C. >300-500 ml/24h; Laurent Karila, 2011
pag. 148-150, 151-156 * fara paragraful referitor la
D. Sputa sanghinolentă cu 100 ml/24h;
prevalenta astmului bronsic in Franta pag. 151 si
E. > 200ml odată. fara denumiri comerciale pag. 153, 154, 155, 156
(pag. 181)
*1 Care dintre următoarele afirmații privind
7 Tusea cronică productivă cu radiografie dozarea Ig E serice specifice nu este
toracică normală apare în următoarele adevărată:
afecţiuni:
A. Reprezintă explorarea de primă intenție în
A. Mucoviscidoză; testarea unei alergii;
B. Astmul hipersecretant; B. Este limitată la 5 pneumoalergeni;
C. Pneumocistoza; C. Este utilă când testele cutanate nu se pot
efectua;
D. Pneumopatia infiltrantă difuză;
D. Este inutilă dacă testele cutanate sunt negative;
E. Tuberculoza bronșică.
E. Este inutilă dacă examenul clinic este puțin
(pag. 145)
evocator.
8 Tusea cronică cu radiografie toracică (pag. 149)
anormală nu apare în următoarele
circumstanţe: *2 Testele de provocare specifice cuprind
următoarele aspecte:
A. Fibroza pulmonară;
A. Declanșarea unei reacții alergice la nivelul
B. Mucoviscidoză; mucoasei nazale, bronșice sau conjunctivale la
C. Tratament cu inhibitori ai enzimei de conversie; alergenul suspectat;
D. Dilatare bronșică difuză; B. Se realizează în timpul terapiei antialergice;
E. Reflux gastroesofagian. C. Sunt utile în caz de suspiciune de alergii
(pag. 145) profesionale;
D. Se efectuează în timpul episoadelor infecțioase;
9 Printre complicațiile tusei cronice se
regăsesc: E. Nu necesită supraveghere medicală ulterioară.
(pag. 149)
A. Tulburările de ritm;
B. Cefaleea; *3 Rinita persistentă se caracterizează prin:
C. Fracturile costale; A. simptome < 4 zile/săpt.;
D. Tulburările de tranzit intestinal; B. simptome > 4săpt.;
E. Incontinența urinară. C. simptome > 4zile/săpt.;
(pag. 146) D. simptome < 4 săpt.;
E. simptome > 4 zile/săpt sau < 4 săpt.
(pag. 148)

Pag. 14 din 130


*4 Obținerea unui control optim al pacientului 9 Astmul instabil se caracterizează prin:
astmatic nu presupune:
A. Scăderea frecvenței crizelor diurne;
A. Simptome diurne < 4 zile/săpt.; B. Agravare nocturnă;
B. Beta 2 mimetice cu acțiune rapidă > 4 doze/săpt.; C. Sensibilitate mai mică a crizelor la
C. VEMS sau PEF > 85% din cea mai bună valoare bronhodilatatoarele obișnuite;
personală; D. Scăderea PEF;
D. Simptome nocturne < 1 noapte/săpt.; E. Creșterea consumului de beta 2 agoniști.
E. Variație nictemerală a PEF < 15%. (pag. 153)
(pag. 155)
10 Care dintre următoarele criterii reprezintă
*5 Spitalizarea unei crize astmatice este semne de gravitate ale unui astmă:
indicată atunci când:
A. PEF < 30% decât cel teoretic;
A. PEF este < 50% din valoarea optimă la 2-3 ore B. FR < 30/min.;
după tratamentul inițial;
C. Cianoza;
B. PEF este < 40% din valoarea optimă la 2-3 ore
după tratamentul inițial; D. Silențium respirator;
C. PEF cuprins între 40-60% din valoarea optimă la E. Hipocapnia.
2-3 ore după tratamentul inițial; (pag. 152)
D. PEF cuprins între 30-50% din valoarea optimă la
2-3 ore după tratamentul inițial; Tema nr. 13
E. PEF este > 50% din valoarea optimă la 2-3 ore Tuberculoza
după tratamentul inițial. Bibliografie asociata temei:
(pag. 154) Book Des ECN, editia in limba romana, redactor
Laurent Karila, 2011
6 Astmul persistent moderat se caracterizează pag. 157-160 * fara denumiri comerciale pag. 157,
prin: 158, 159 fara randul 2 legat de legislatia Franceza
A. Simptome cotidiene; pag. 160, fara ultimul chenar cu semnale si
notificare pag. 160
B. VEMS < 40% din valorile prezise și cu o
variabilitate > 30%;
*1 Prezența bacililor acido-alcoolo-rezistenți la
C. Alterarea somnului; examenul direct se evidențiază prin:
D. Utilizarea zilnică de beta 2 agoniști inhalanți cu A. Colorația Mallory;
durată scurtă;
B. Colorația hematoxilină-eozină;
E. Simptome de astm nocturn < 1/săpt.
C. Colorația Ziehl-Nielsen;
(pag. 155)
D. Colorația Verhoeff;
7 Diagnosticul astmului presupune: E. Colorația nitrat de argint.
A. Episoade recidivante de wheezing, dispnee, tuse; (pag. 157)
B. VEMS/CV diminuat;
*2 La pacientul aflat sub tratament
C. VEMS mai mare cu cel mult 200 ml față de antituberculos, bilanțul hepatic
VEMS inițial după administrarea (monitorizarea transaminazelor) se face:
bronhodilatatorului;
A. La o săptămână după inițierea tratamentului;
D. Absența unor alergeni sau factori iritanți;
B. La 15 zile după inițierea tratamentului și apoi o
E. Absența istoricului de atopie personală/familială. dată pe lună până la sfârșitul tratamentului;
(pag. 151) C. Trimestrial;
8 Tratamentul astmului acut grav cuprinde D. O dată la 6 luni;
următoarele măsuri: E. Nu este necesar un bilanț hepatic.
A. Oxigen nazal 3-4 l/min.; (pag. 160)
B. Corticoizi sistemici;
C. Nebulizare cu doze crescute de beta 2 agoniști;
D. Digitalizare;
E. Flebotomie.
(pag. 153)

Pag. 15 din 130


*3 Confirmarea prezenței bacilului Koch se 8 În privința tratamentului antituberculos al
realizează prin efectuarea de culturi pe unei femei însărcinate, următoarele afirmații
mediul: sunt adevărate:
A. Geloză simplă; A. Pirazinamida este contraindicată;
B. Chapman; B. Se instituie triterapie IHN + Rifampicină +
C. Sabouraud; Pirazinamidă timp de 9 luni;
C. Etambutolul este contraindicat;
D. Löwenstein-Jensen;
D. Se instituie triterapie IHN + Rifampicină +
E. Leifson.
Etambutol 3 luni, urmată de 6 luni cu IHN +
(pag. 157) Rifampicină, fără Etambutol;
*4 Înaintea instituirii tratamentului E. Durata totală a tratamentului este de 9 luni.
antituberculos cu Etambutol se efectuează: (pag. 159)
A. Examen oftalmologic cu vizualizarea culorilor; 9 Urmărirea pacientului aflat sub tratament
B. Examen de urină; antituberculos are ca obiective:
C. Audiogramă; A. Vindecarea pacientului;
D. Identificarea unei eventuale sarcini; B. Evitarea răspândirii bolii de către un pacient
E. Examen neurologic. tratat neadecvat;
(pag. 159) C. Dezvoltarea rezistenței la medicamentele
antituberculoase;
*5 Tratamentul antituberculos se va administra: D. Supravegherea pacientului până la finalul bolii;
A. În trei prize, în timpul meselor; E. Documentarea sfârșitului tratamentului.
B. Dimineața, à jeun, la mare distanță de mese; (pag. 159)
C. Dimineața și seara, înaintea meselor;
10 Tuberculoza miliară prezintă următoarele
D. Numai seara, înainte de culcare; caracteristici:
E. La prânz, imediat după masă. A. Alterarea rapidă a stării generale;
(pag. 159) B. Dispnee posibilă pentru forma evoluată;
6 În tratamentul antituberculos administrat C. Sindrom interstițial micronodular difuz și intens la
adultului se regăsesc următoarele preparate: radiografia toracică;
A. Etambutolul; D. BK adesea prezent la examenul direct
microscopic;
B. Pirazinamida;
E. Histologic: granulom epitelioid și giganto-celular
C. Acidul clavulanic; cu necroză cazeoasă.
D. Metronidazolul; (pag. 157)
E. Rifampicina.
Tema nr. 14
(pag. 159)
Infecţiile bronhopulmonare la adult
7 Tuberculoza pulmonară comună prezintă Bibliografie asociata temei:
următoarele caracteristici:
Book Des ECN, editia in limba romana, redactor
A. Debut brusc, cu febră înaltă, dispnee, tahicardie, Laurent Karila, 2011
dureri toracice; pag. 169-175, 189-192
B. Subfebrilități, transpirații nocturne, tuse
persistentă; *1 Durata tratamentului antibiotic într-o
pneumonie necomplicată este:
C. Episoade de wheezing, opresiune toracică și
tuse; A. 3 zile;
D. Infiltrate, noduli, caverne la radiografia toracică; B. 3-5 zile;
E. Opacitate parenchimatoasă unică, sistematizată, C. 7-14 zile;
cu bronhogramă aerică la radiografia toracică.
D. 21 de zile;
(pag. 157)
E. 90 de zile.
(pag. 171)

Pag. 16 din 130


*2 Proba funcțională respiratorie (PFR) într-o 7 Printre criteriile biologice și radiologice de
pneumopatie interstițială difuză arată: spitalizare a unui pacient cu pneumopatie se
numără:
A. Creșterea capacității pulmonare totale (CPT);
B. Creșterea raportului DLCO/VA; A. Leucopenia (< 4000 GB/ml);

C. Scăderea raportului Tiffeneau (VEMS/CV); B. Anemia (hemoglobina < 9 g/dl);

D. Disfuncție ventilatorie restrictivă; C. Insuficiența renală (uree > 7 mmol/l sau 0,5 g/l,
creatinina > 12 mg/l);
E. Disfuncție ventilatorie obstructivă.
D. PaO2 > 90 mmHg sau PaCO2 < 45 mmHg;
(pag. 190)
E. Cavitate pe radiografia toracică.
*3 Într-o pneumopatie alveolară tratată la (pag. 170)
domiciliu, antibioticul de primă intenție este:
8 Legioneloza pulmonară prezintă următorul
A. Amoxicilina;
tablou tipic:
B. Ciprofloxacina;
A. Debut lent, insidios, cu subfebră;
C. Rifampicina;
B. Tuse cu expectorație mucopurulentă;
D. Ceftriaxona;
C. Apar semne neurologice (confuzie, agitație,
E. Levofloxacina. cefalee);
(pag. 171) D. Oligurie;
*4 Într-o pneumopatie francă lobară acută cu E. Radiografie: opacități interstițiale neconfluente,
pneumococ se constată că: unilaterale.
A. Debutul este insidios; (pag. 172)

B. Există subfebră; 9 Tabloul tipic într-o pneumopatie cu


C. Apare hipersonoritate la percuție; Mycoplasma pneumoniae cuprinde:

D. La auscultație: raluri crepitante sau suflu tubar; A. Debut brusc, cu febră ridicată, cu frisoane
repetate;
E. Radiologic: opacități alveolare vagi, confluente,
nesistematizate, uneori bilaterale. B. Tuse seacă, tenace;
(pag. 172) C. Anemie hemolitică cu aglutinine la rece (test
Coombs direct pozitiv);
*5 Într-o fibroză pulmonară primitivă: D. Radiografie: opacități interstițiale bilaterale;
A. Debutul este brusc, cu tuse și expectorație E. Tratament: cefalosporine de a treia generație, 5-
mucopurulentă; 7 zile.
B. Apar frecvent semne extrarespiratorii; (pag. 172)
C. Vârsta medie de apariție este de 20 de ani;
10 Histiocitoza langerhansiană (histiocitoza X)
D. Evoluția este favorabilă, spre vindecare; prezintă următoarele caracteristici:
E. Lavajul bronhoalveolar (LBA): alveolită cu PNN A. Afectează subiecții între 20-40 de ani care
frecvent cu eozinofile. consumă cantități importante de tutun;
(pag. 191) B. Prezintă tuse, dispnee de efort, febră, pierdere în
greutate;
6 Spitalizarea unui pacient cu pneumopatie
este recomandată în prezența unor semne de C. Aspect CT toracic: mici cavități chistice care
gravitate imediată: confluează și realizează un aspect de „fagure de
miere“;
A. Confuzie;
D. Biopsiile transbronșice sunt eficiente în
B. Frecvența cardiacă ≥ 125/min.; confirmarea diagnosticului;
C. Roșeață la nivelul pomeților; E. Tratament: oprirea fumatului, eventual
D. Temperatura de 37-38ºC; corticoterapie orală.
E. Presiune arterială < 90/60 mmHg. (pag. 192)
(pag. 170)

Pag. 17 din 130


Tema nr. 15 *5 Aspectul radiologic al unui pneumotorace
Afectiuni ale pleurei (pneumotoraxul, revarsatul poate releva:
pleural) A. Opacitate omogenă între peretele toracic și
Bibliografie asociata temei: parenchimul pulmonar;
Book Des ECN, editia in limba romana, redactor B. Opacități interstițiale neconfluente reticulo-
Laurent Karila, 2011 nodulare sau miliare;
pag. 183-185, 186-188 C. Hiperclaritate omogenă și avasculară între
peretele toracic și parenchimul pulmonar;
*1 Transsudatul pleural apare în una din
D. Excavație cu nivel hidroaeric unic sau multiplu;
următoarele condiții:
E. Opacitate alveolară sistematizată.
A. Mezoteliom malign;
(pag. 183)
B. Sindrom nefrotic;
C. Poliartrită reumatoidă; 6 Semnele radiologice de gravitate într-un
pneumotorace sunt:
D. Febră mediteraneană;
A. Mediastin deplasat pe partea contralaterală;
E. Pancreatită.
B. Cupolă diafragmatică convexă;
(pag. 187)
C. Bridă pleurală;
*2 Examenul clinic al unui sindrom de revărsat
D. Nivel hidroaeric;
pleural relevă:
E. Pneumotorace bilateral.
A. Accentuarea murmurului vezicular;
(pag. 184)
B. Accentuarea transmiterii vibrațiilor vocale;
C. Suflu tubar; 7 Un lichid pleural este un exsudat în
următoarele condiții:
D. Bradipnee, bradicardie, ca semne de intoleranță;
A. Raport proteine în lichid/proteine în sânge > 0,5;
E. Matitate declivă.
B. Rport LDH în lichid/LDH în sânge > 0,6;
(pag. 186)
C. Raport proteine în lichid/proteine în sânge < 0,5 ;
*3 Între semnele de gravitate dintr-un
D. Raport LDH în lichid/LDH în sânge < 0,6;
pneumotorace nu intră:
E. LDH în lichid > două treimi din limita superioară
A. Dispnee majoră; normală a LDH în sânge.
B. Cianoză; (pag. 187)
C. Bradipnee;
8 Condițiile în care apare pneumotoracele
D. Puls paradoxal; spontan primitiv sunt:
E. Desaturare. A. adulți tineri, longilini;
(pag. 184) B. consecutiv unei fracturi costale;
*4 Una dintre afirmațiile următoare, referitoare C. fumatul este un factor favorizant;
la examenul clinic al unui pneumotorace, nu D. ruptura unei cavități aerice aflate în contact cu
este adevărată: pleura apicală (bula sau blebsuri);
A. Timpanism la percuție; E. după un infarct pulmonar.
B. Accentuarea transmiterii vibrațiilor vocale; (pag. 183)
C. Diminuarea murmurului vezicular la auscultație;
9 Aspectele corecte ale radiografiei toracice
D. Dacă apare matitate bazală: suspiciune de într-un revărsat pleural sunt:
hemopneumotorace;
A. Hiperclaritate omogenă, densă;
E. Uneori, examenul clinic poate fi normal.
B. Opacitate omogenă, densă, cu convexitatea în
(pag. 183) sus;
C. Opacitate omogenă, densă, cu concavitatea în
sus;
D. Deplasarea mediastinului de partea opusă
opacității;
E. Retracția mediastinului de partea opacității.
(pag. 187)

Pag. 18 din 130


10 Condițiile patologice în care apare un *4 Care din următoarele afirmații referitoare la
transsudat pleural sunt: cancerul bronhopulmonar fără celule mici
este adevărată:
A. Insuficiența cardiacă;
B. Ciroza; A. Este mai frecvent întâlnit la femei;

C. Dializa peritoneală; B. Apare mai frecvent la nefumători;

D. Pancreatita; C. Este mai frecvent întâlnit la bărbați;

E. Cancer bronhopulmonar. D. Expunerea la factori profesionali nu este


implicată în etiologie;
(pag. 187)
E. Alcoolismul favorizează apariția acestuia.
Tema nr. 16 (pag. 198)
Tumori ale plămânului primitive şi secundare
*5 Una dintre următoarele afirmații referitoare la
Bibliografie asociata temei: tratamentul cancerului bronhopulmonar fără
Book Des ECN, editia in limba romana, redactor celule mici este falsă:
Laurent Karila, 2011
A. În stadiul IA tratamentul este exclusiv chirurgical;
pag. 197-202
B. În stadiul I sau II inoperabil tratamentul include și
1 În cancerul broho-pulmonar fără celule mici, radioterapia;
bilanţul iniţial include: C. În stadiul IV tratamentul include chimioterapie
sau terapie țintită;
A. Fibroscopia bronșică;
D. În stadiul II se poate efectua tratament
B. Probele funcționale respiratorii;
chirurgical, radioterapie sau chimioterapie;
C. Markerii tumorali;
E. În stadiul IIIB tratamentul include doar
D. Biopsia medulară; chimioterapia.
E. CT toracic. (pag. 199)
(pag. 198) 6 Cancerele bronhopulmonare cu celule mici
2 Care dintre următoarele investigații aparțin au următoarele caracteristici:
evaluării inițiale al cancerului broho- A. Reprezintă 80% din tumorile primitive ale
pulmonar cu celule mici: plămânului;
A. Fibroscopia bronșică; B. Asociază frecvent prezența metastazelor;
B. Probe funcționale respiratorii; C. Rar, asociază prezența metastazelor;
C. Markeri tumorali; D. Baza tratamentului este chimioterapia;
D. Biopsie medulară; E. Tratamentul de bază este cel chirurgical.
E. Scintigrafie osoasă. (pag. 199)
(pag. 198) *7 Care din următoarele afirmații referitoare la
3 Cancerul brohopulmonar fără celule mici cancerul bronhopulmonar cu celule mici este
regrupează următoarele tipuri histologice: falsă:

A. Epidermoid; A. Are o prezentare centrală;

B. Cu celule mici; B. Are o frecvență crescută a semnelor generale;

C. Cu celule mari; C. Are o evoluție lentă, secundară unui timp de


dedublare de peste o lună;
D. Adenocarcinoamele;
D. Prezintă totdeauna afectare mediastinală;
E. Slab diferențiat.
E. Prezintă o frecvență crescută a sindroamelor
(pag. 198) paraneoplazice tip Schwartz-Barter.
(pag. 199)

Pag. 19 din 130


*8 Tumorile pulmonare secundare se prezintă Tema nr. 17
sub mai multe aspecte, cu excepţia: Insuficienţa respiratorie cronică
A. Nodulilor pulmonari; Bibliografie asociata temei:
B. Limfangitei carcinomatoase; Book Des ECN, editia in limba romana, redactor
C. Adenopatiilor cervicale; Laurent Karila, 2011
pag. 203-205 * fara denumiri comerciale pag. 203
D. Metastazelor endobronșice;
E. Adenopatiilor mediastinale. 1 Următoarele afirmaţii privind insuficienţa
(pag. 200) respiratorie sunt adevărate:
A. Insuficiența respiratorie cronică reprezintă
9 Miliara metastatică din cadrul tumorilor
imposibilitatea aparatului circulator de a menține
pulmonare secundare este mai frecvent
gazometria;
întâlnită în caz de:
B. Insuficiența respiratorie obstructivă se
A. Melanom malign; caracterizează prin diminuarea capacității
B. Cancer renal; pulmonare totale;
C. Cancer hepatic; C. Insuficiența respiratorie obstructivă se
caracterizează prin afectarea schimbului
D. Cancer testicular; respirator;
E. Cancer ovarian. D. Insuficiența respiratorie restrictivă se
(pag. 200) caracterizează prin diminuarea capacității
pulmonare totale;
* 10 Care afirmație referitoare la tumorile
pulmonare secundare este falsă: E. Insuficiența respiratorie restrictivă se
caracterizează prin afectarea pompei ventilatorii.
A. Plămânul este al doilea sit ca frecvență pentru (pag. 203)
metastazare;
B. Difuzarea metastatică se poate face pe cale *2 Insuficiența respiratorie restrictivă de cauză
hematogenă; mecanică apare în următoarele situații
patologice, cu excepția:
C. Difuzarea metastatică se poate face pe cale
limfatică de la ganglionii abdominali; A. Cifoscolioză;
D. Difuzarea metastatică se poate face pe cale B. Spondilartrită anchilozantă;
limfatică de la ganglionii mediastinali;
C. Malnutriție;
E. Difuzarea metastatică se poate face pe cale
D. Pleurezie;
limfatică de la pleura viscerală invadată.
E. Toracoplastie.
(pag. 200)
(pag. 203)

3 Insuficiența respiratorie restrictivă datorată


unei afectări neurologice sau musculare
apare în următoarele circumstanţe
patologice:
A. Poliomielită anterioară acută;
B. Poliradiculonevrită;
C. Miastenie;
D. Zona Zoster;
E. Dermatopolimiozită.
(pag. 203)

Pag. 20 din 130


*4 Insuficiența respiratorie restrictivă apare în *9 Singura cauză de insuficiența respiratorie
următoarele afecțiuni caracterizate prin cronică restrictivă al cărui tratament se
afectare pulmonară parenchimatoasă, cu bazează în principal pe oxigenoterapie pe
excepția: termen lung este:
A. Fibroză pulmonară idiopatică; A. Astmul;
B. Sarcoidoză; B. BPOC;
C. Astm; C. Fibrozele interstițiale;
D. Silicoză; D. Sindromul Guillan-Barre;
E. Pneumopatii postradice. E. Miastenia gravis.
(pag. 203) (pag. 204)

5 Diagnosticul de tulburare ventilatorie 10 Mecanismele de acțiune a asistenței


restrictivă are la bază următoarele elemente: ventilatorii mecanice includ:
A. Diminuarea CPT; A. Creșterea ventilației alveolare;
B. Scăderea raportului VEMS/CV; B. Punerea în repaus a mușchilor respiratori;
C. Diminuarea complianței pulmonare; C. Creșterea complianței pulmonare;
D. Creșterea raportului DLCO/VA; D. Creșterea clearance-ului mucociliar;
E. Creșterea CPT. E. Diminuarea hiperinflaței pulmonare.
(pag. 203) (pag. 204)

*6 Ameliorarea oxigenării la pacienții cu Tema nr. 18


insuficiență respiratorie cronică include Detresa respiratorie acută la adult
următoarele mijloace terapeutice, cu
excepția: Bibliografie asociata temei:
Book Des ECN, editia in limba romana, redactor
A. Kinetoterapie respiratorie;
Laurent Karila, 2011
B. Oxigenoterapie de lungă durată; pag. 176-178 * fara denumiri comerciale pag. 178
C. Tratament diuretic;
1 Sindromul de detresă respiratorie acută
D. Ventilație asistată;
reprezintă:
E. Prevenirea episoadelor infecțioase.
A. Insuficiența respiratorie acută ce necesită
(pag. 204) ventilație asistată;
7 Tratamentul etiologic instituit pacienților cu B. Insuficiența respiratorie acută cu un raport
insuficiența respiratorie restrictivă include: PaO2/FiO2 sub 200;

A. Chirurgie corectoare a unei scolioze; C. Insuficiența respiratorie acută cu un raport


PaO2/FiO2 peste 200;
B. Decorticare pleurală;
D. Insuficiența respiratorie acută cu o presiune de
C. Scăderea ponderală în caz de obezitate; ocluzie pulmonară sub 18 mmHg;
D. Pneumectomie; E. Insuficiența respiratorie acută cu o presiune de
E. Stimularea diafragmei prin pacemaker ocluzie pulmonară peste 18 mmHg.
diafragmatic. (pag. 176)
(pag. 204)
2 Sindromul de detresă respiratorie acută are
*8 În caz de afectare a schimbului pulmonar la următoarele caracteristici:
pacienții cu insuficiența respiratorie A. Asociază un edem pulmonar lezional;
markerul biologic cel mai precoce este:
B. Asociază un edem pulmonar cardiogenic;
A. Hipercapnia arterială cronică;
C. Prezintă creșteri ale presiunii hidrostatice
B. Hipoxemia arterială cronică; microvasculare;
C. Hipokaliemia; D. Prezintă creșterea permeabilității alveolocapilare;
D. Hiponatremia; E. Are prognostic foarte sever.
E. Hipovolemia. (pag. 176)
(pag. 204)

Pag. 21 din 130


*3 Sindromul de detresă respiratorie acută *8 Sindromul de detresă respiratorie acută
apare secundar următoarelor cauze generale, apare în următoarele circumstanţe
cu exceptia: accidentale:
A. Infecții respiratorii; A. Expunere la fumuri toxice;
B. Traumatisme; B. Expunere la temperaturi scăzute;
C. Infarct miocardic acut; C. Expunere la temperaturi ridicate;
D. Agresiune pulmonară toxică; D. Expunere la pulberi minerale neiritante;
E. Infecţii extrarespiratorii. E. Expunere la gaze lipsite de toxicitate.
(pag. 177) (pag. 177)

*4 Sindromul de detresă respiratorie acută 9 Sindromul de detresă respiratorie acută


secundar agresiunii pulmonare toxice apare apare în următoarele forme de şoc:
în următoarele situații, cu excepția:
A. Șoc septic;
A. Stările cu șoc non-cardiogen; B. Șoc cardiogen;
B. Stările cu șoc cardiogenic; C. Șoc hemoragic;
C. Hemopatii; D. Șoc anafilactic;
D. Vasculite; E. Orice altă formă de şoc non-cardiogen.
E. Stările cu șoc hemoragic. (pag. 177)
(pag. 177)
* 10 Care dintre următoarele afirmații este falsă:
*5 Sindromul de detresă respiratorie acută
A. În SDRA perturbarea principală este hipoxemia;
poate apare în următoarele situații
patologice, cu excepția: B. În SDRA capacitatea reziduală funcțională este
diminuată;
A. Înec;
C. În SDRA volumul de închidere al alveolelor este
B. Sindrom Mendelson; crescut;
C. Septicemie; D. În SDRA nu există afectarea proprietăților
D. Politraumatism; elastice ale plămânului;
E. Pleurită. E. În SDRA este posibilă apariția hipertensiunii
pulmonare.
(pag. 177)
(pag. 178)
6 Sindromul de detresă respiratorie acută
poate apare în următoarele stări patologice: Tema nr. 19
A. Acidoză diabetică; Evaluarea gravităţii şi investigarea complicaţiilor
precoce la un pacient cu traumatism toracic
B. Pancreatită acută necrotică;
Bibliografie asociata temei:
C. Sindrom nefrotic;
Book Des ECN, editia in limba romana, redactor
D. Pneumopatie bacteriană; Laurent Karila, 2011
E. Gripă malignă. pag. 206-209
(pag. 177)
*1 Un pacient stabil din punct de vedere
7 Cauzele traumatice ale detresei respiratorii hemodinamic şi respirator, ce a suferit un
acute sunt: traumatism toracic, va putea beneficia
imediat de următoarele investigaţii, cu
A. Contuzia pulmonară; excepţia:
B. Traumatismul cranian; A. radiografie toracică
C. Fracturi ale oaselor mici; B. ecografie cardiacă transtoracică
D. Zdrobirea membrelor; C. tomografie toracica cu substanţă de contrast
E. Monotraumatismul. D. bronhoscopie
(pag. 177)
E. urografie
(pag. 206)

Pag. 22 din 130


*2 Care dintre afirmaţiile următoare este falsă: 6 Complicaţiile precoce după un traumatism
toracic sunt:
A. fracturile costale pot produce un pneumotorax
B. fractura coastelor inferioare la stanga pot ridica A. pneumotoracele şi hemotoracele
suspiciunea unei leziuni splenice B. pneumoperitoneul
C. pneumotoraxul apare doar în urma unui C. pneumomediastinul
traumatism toracic
D. retroperitoneul
D. hemotoraxul poate fi izolat sau asociat unui
E. contuzia pulmonară
pneumotorax
(pag. 207, 208)
E. pneumotoraxul sufocant necesită drenaj toracic
imediat 7 Contuzia miocardică produsă prin
(pag. 207) traumatism toracic se traduce prin:.

*3 Ruptura de istm aortic se produce prin A. emfizem subcutanat


următoarele mecanisme: B. tulburări de repolarizare
A. cădere de la înălţime în picioare C. durere in hipocondrul drept
B. decelerare brutală şi violentă D. creşterea troponinei
C. fractura primei coaste E. tulburări de ritm
D. plagă tăiată la nivelul hipogastrului (pag. 208)
E. traumatism sever pelvin 8 In cazul unui traumatism toracic, leziunile
(pag. 208) axului traheobronşic sunt:

*4 Afirmaţiile următoare sunt adevărate, cu A. ruptura bronşică


excepţia: B. ruptura membranei traheale
A. voletul costal corespunde unor fracturi costale C. ruptura de istm aortic
bifocale D. dezinserţia laringo-traheală
B. este frecvent asociat unui hemotorax prin E. contuzia miocardică
lezarea pediculului intercostals
(pag. 208)
C. pacientul cu volet costal va beneficia de corset
gipsat 9 Soferul unui vehicul implicat într-un accident
D. decelerarea brutală şi violentă produce ruptura rutier frontal, fără să fi avut centura fixată, se
de istm toracic loveşte cu toracele de volan; ce leziuni
cardiace pot să apară în acest caz?
E. pacientul care a suferit un traumatism toracic şi
care este stabil hemodinamic poate beneficia de A. contuzie miocardică
radiografie toracică B. hemopericard
(pag. 207)
C. pneumomediastin
*5 Care dintre afirmaţiile următoare este falsă: D. dezinserţia cuspidelor
A. ruptura unei bronhii poate să apara printr-un E. ruptura de pilier
traumatism toracic (pag. 208)
B. intr-un traumatism toracic detresa circulatorie
poate să se datoreze unei tamponade cardiace 10 Intr-un traumatism toracic, detresa
circulatorie poate fi datorată:
C. ruptura diafragmatică este mai frecventă pe
stănga A. unui şoc hemoragic
D. prezenţa unui nivel hidroaeric intratoracic B. unui şoc cardiogenic legat de o contuzie
precizează diagnosticul de ruptura diafragmatica miocardică sau o disecţie coronară
E. radiografia toracică este metoda de elecţie in C. unei tamponade (hemoragică sau gazoasă în
diagnosticul unei rupturi diafragmatice caz de pneumotorace compresiv)
(pag. 207, 208) D. unei contuzii pulmonare
E. unei luxaţii extrapericardice
(pag. 206)

Pag. 23 din 130


Tema nr. 20 *4 Mecanismul patologic in starea de soc
Stopul cardio-respiratoriu şi starea de şoc hipovolemic se poate rezuma in :

Bibliografie asociata temei: A. ↓ volumul sangvin circulant →↓ preincarcarea


cardiaca →↓ debitul cardiac→↓ aportul de O2 la
Book Des ECN, editia in limba romana, redactor celule
Laurent Karila, 2011
pag. 217-219, 220-223 B. ↓ volumul sangvin circulant →↓ preincarcarea
cardiaca →↑ debitul cardiac→↓ aportul de O2 la
*1 In cadrul manevrelor de baza in reanimarea celule
cardio – pulmonara, rata de compresiuni C. ↓ volumul circulant →↑ preincarcarea cardiaca
toracice / insuflari va fi de : →↓ debitul cardiac→↓ aport de O2 la celule
A. 30 compresiuni toracice la 2 insuflari D. ↑ volumul circulant →↓ prin preincarcarea
cardiaca →↓ debitul cardiac→↑ aport de O2 la
B. 20 compresiuni toracice la 3 insuflari celule
C. 20 insuflari la 3 compresiuni E. ↓ volumul circulant →↑ preincarcarea cardiaca
D. 30 insuflari la 2 compresiuni →↑ debitul cardiac→↓ aportul de O2 la celule
E. 3 compresiuni la 1 insuflare (pag. 220)
(pag. 217) *5 Mecanismul patologic in starea de soc
cardiogen se poate rezuma in :
*2 In cazul stopului cardiac, dupa administrarea
unui soc electric extern se recomanda: A. ↓ functia de pompare cardiaca →↓ debitul
cardiac si ↓ preincarcarea cardiaca →↓ aportul
A. reluarea manevrelor de reanimare cardio -
de O2 la celule
pulmonara de baza timp de 3 minute
B. ↓ functia de pompare cardiaca →↓ debitul
B. reluarea manevrelor de reanimare cardio -
cardiac si ↑preincarcarea cardiaca →↓ aportul de
pulmonara de baza ( masaj cardiac si ventilatie)
O2 la celule
timp de 2 min
C. ↓ volumul sangvin circulant →↓ preincarcarea
C. administrarea 1 mg Adrenalina iv
cardiaca →↓ functia de pompare cardiaca
D. administrarea de 300 mg Amiodarona iv
D. vasodilatatie masiva → scade volumul sangvin
E. verificarea imediat a disparitiei FV / TV pe traseul circulant →↓ functia de pompare cardiaca→↓
EKG aportul de O2 la celule
(pag. 218) E. vasodilatatie masiva → ↓ perfuzia organelor →↓
functia de pompare cardiaca→↑ debitul
*3 Definitia clinica a starii de soc este: cardiac→↓ aportul de O2 la celule
A. hTA cu TAS < 90 mmHg + semne de (pag. 220)
hipoperfuzie a organelor ;
6 In cadrul manevrelor de reanimare cardio –
B. hTA cu TAS < 100 mmHg + semne de
hipoperfuzie a organelor pulmonara sunt adevarate urmatoarele :

C. hTA cu TAS < 90 mmHg in absenta semnelor de A. masajul cardiac extern se face cu un ritm de 100
hipoperfuzie a organelor compresiuni / min
D. hTA cu TAS < 90 mmHg indiferent de prezenta B. realizarea ventilatiei e mai importanta decat
sau absenta semnelor de hipoperfuzie a realizarea masajului cardiac
organelor C. intubarea si ventilatia mecanica trebuie realizate
E. hTA cu TAS < 100 mmHg indiferent de prezenta cat mai repede posibil
sau absenta semnelor de hipoperfuzie a D. adrenalina se administreaza in doza de 10 mg iv
organelor
E. In stopul cardiac prin FV / TV fara puls , socurile
(pag. 220) externe bifazice sunt de 150-200 J
(pag. 217, 219)

7 Dintre medicamentele intalnite in reanimarea


cardio – pulmonara ( in stopul cardiac ) fac
parte :
A. Noradrenalina
B. Adrenalina
C. Amiodarona
D. Digogxin
E. Lidocaina
(pag. 218,219)
Pag. 24 din 130
8 In cadrul tratamentului socului anafilactic Tema nr. 21
sunt adevarate urmatoarele : Tromboza venoasă profundă şi embolia pulmonară
A. administrarea in bolus iv de Adrenalina 0,1 mg Bibliografie asociata temei:
pentru a restabili tensiunea arteriala
Book Des ECN, editia in limba romana, redactor
B. administrarea de antibiotice este esentiala Laurent Karila, 2011
C. In absenta caii intravenoase adrenalina poate fi pag. 210-216 * fara denumiri comerciale pag. 201,
administrata subcutanat 211, 214
D. administrarea de adrenalina se face in bolus iv
1 Următoarele semne clinice pot releva o
de 0,5 mg
tromboză venoasă profundă:
E. In absenta caii intravenoase , adrenalina poate fi
administrata intramuscular 0,5 mg. A. Febra ridicată (39 – 40 grade), în platou
(pag. 222) B. Tahicardia cu aspect progresiv
C. Durerea de-a lungul traiectului venos
9 In cadrul tratamentului starilor de soc , sunt
adevarate urmatoarele : D. Păstrarea capacităţii de balans a gambei
A. expansiunea volemica este indicata in toate E. Existenţa unui cordon venos dur.
starile de soc cu exceptia edemelor pulmonare (pag. 210)
evidente
*2 Manifestările clinice întâlnite contant în
B. Daca s-a concretizat o diminuare a contractilitatii
embolia pulmonară sunt, cu exceptia:
cardiace se indica un inotrop pozitiv
C. Noradrenalina( 10 mg / h iv cu seringa automata A. Polipneea
) pentru obtinerea unei TA medie ≥ 65 mmHg B. Hemoptizia
D. In caz de soc cardiogen cu IM acut , se evita C. Bradicardia
revascularizarea
D. Durerea toracică
E. Recurgerea rapida la intubare si ventilatie
E. Sincopa
mecanica in caz de detresa respiratorie
grava. (pag. 211)
(pag. 221,222) 3 În embolia pulmonară examenul radiologic
toracic poate releva:
10 In stopul cardiac , alcalinizarea prin
administrarea de bicarbonat este indicata in A. O atelectazie în bandă
caz de :
B. Existenţa unei pleurezii
A. Hipopotasemie confirmata sau suspectata C. Opacifierea unui câmp pulmonar
B. Hiperpotasemie confirmata sau suspectata D. Supraelevaţia domului pleural
C. Hipercalcemie E. O opacitate triunghiulară pe fond pleural.
D. Hipocalcemie suspectata (pag. 211)
E. Stop cardiac in cadrul unei intoxicatii cu
antidepresive triciclice. 4 Confirmarea diagnosticului, dar nu a
gravităţii emboliei pulmonare este asigurată
(pag. 218) de:
A. Dozajul D-dimerilor
B. Scintigrafia pulmonară de ventilaţie – perfuzie
C. Troponina I sau C cardiacă
D. Peptidul natriuretic de tip B (BNP)
E. Angio-CT spiralat al arterelor pulmonare.
(pag. 212)

5 În faza iniţială a emboliei pulmonare o


anticoagulare eficientă poate fi realizată cu:
A. Acid acetilsalicilic
B. Heparină nefracţionată
C. Heparină cu masă moleculară mică
D. Oxigenoterapie
E. Clopidogrel
(pag. 212)
Pag. 25 din 130
*6 După un prin accident tromboembolic Tema nr. 22
tratamentul anticoagulant durează toată Patologia vasculară cerebrală (accidente vasculare
viaţa, în caz de: cerebrale hemoragice şi ischemice) şi meningita
A. Recidiva accidentului tromboembolic (hemoragia)
B. Neoplazie evolutivă Bibliografie asociata temei:
C. Deficit de antitrombină Book Des ECN, editia in limba romana, redactor
Laurent Karila, 2011
D. În absenţa unui factor favorabil
pag. 250-259, 260-262 * fara denumiri comerciale
E. În prezenţa unui factor favorabil reversibil. medicamente
(pag. 213)
*1 Procentul de AVC ischemice din totalul de
7 În identificarea unei trombofilii se recomandă AVC este
următoarele investigaţii: A. 20%
A. Hemogramă completă
c. 50%
B. Dozarea antitrombinei
d. 80%
C. Determinarea indicelui de protrombină 20%
D. Dozarea factorului VIII B. 40%
E. Dozarea hemocisteinemiei. C. 50%
(pag. 213) D. 80%
*8 Diagnosticul diferenţial al emboliei E. 75%
pulmonare masive se face mai ales cu: (pag. 250)
A. Insuficienţa cardiacă stângă cronică *2 Embolia cerebrală cu punct de plecare o
B. Angina pectorală stabilă endocardită infecţioasă trebuie căutată în
caz de
C. Tamponada pericardică
D. Şocul toxico-septic controlat terapeutic A. AVC însoţit de febră, suflu de insuficienţă mitrală
sau aortică
E. Cordul pulmonar cronic compensat.
B. AVC cu cefalee şi cervicalgie
(pag. 213)
C. AVC şi fibrilaţie atrială
*9 Ecocardiografia transtoracică permite D. AVC şi cardiomiopatie dilatativă
evidenţierea următoarelor aspecte în embolia
pulmonară masivă: E. AVC şi diabet zaharat decompensate
(pag. 252)
A. Tablou cardiac stâng acut
B. Dilatare majoră a cavităţii drepte *3 În caul unui AIT deficitul neurologic
regresează în mai puţin de
C. Vizualizarea trombusului în cavităţile cardiace
drepte în toate cazurile A. 24 ore
D. Identificarea trombusului în arterele pulmonare B. 30 minute
în toate situaţiile
C. 48 ore
E. Hipertrofia marcată a septului interventricular.
D. 1 oră
(pag. 213)
E. 72 ore
* 10 Tromboliza intravenoasă este indicată în (pag. 253)
următoarele situaţii:
*4 Tratamentul neurochirurgical în infarctul
A. Emboliile pulmonare demonstrate la examenul cerebral se indică în
CT, fără hipotensiune arterială
A. tromboză venoasă cerebrală
B. Emboliile pulmonare însoţite de creşterea
lactatemiei arteriale B. infarctul transformat hemoragic
C. Emboliile pulmonare confirmate cu stare de şoc C. infarctul lacunar multiplu
D. Emboliile pulmonare indiferent de formă, dar D. infarct emisferic cu edem cerebral “malign”
neînsoţite de stare de şoc extensiv
E. Emboliile pulmonare diagnosticate clinic, dar E. infarct în teritoriul arterei cerebrale anterioare
neconfirmate prin angioscanner sau (pag. 254)
ecocardiografic.
(pag. 214)

Pag. 26 din 130


*5 În tratamentul antiplachetar al infarctului Tema nr. 23
cerebral, cel mai bun raport eficacitate-cost îl Tumorile intracraniene
are
Bibliografie asociata temei:
A. dipiridamolul
Book Des ECN, editia in limba romana, redactor
B. clopidogrelul Laurent Karila, 2011
C. combinaţia dipiridamol-aspirină pag. 274-276
D. combinaţia clopidogrel-aspirină
*1 Tumorile intracraniene intraparenchimatoase
E. aspirina la copil sunt situate mai ales
(pag. 255) A. în lobul frontal
6 În cazul suspiciunii de AVC se impune de B. în hipofiza
urgenţă (pentru diagnostic de certitudine) C. în fosa cerebrală posterioară
A. măsurarea tensiunii arteriale D. în lobul temporal
B. corectarea glicemiei E. în ventriculul lateral
C. repausul la orizontală în pat (pag. 274)
D. CT craniu
*2 Metastazele cerebrale hemoragice apar în
E. RMN craniu
A. cancerul mamar
(pag. 253)
B. cancerul de tiroidă
7 Infarctul sylvian superficial se manifestă prin: C. melanomul malign
A. deficit senzitivo-motor predominant brahio-facial D. cancerul de colon
B. sindrom frontal E. cancerul de prostată
C. afazie dacă este lezată emisfera cerebrală (pag. 274)
majoră
*3 Hidrocefalia acută apărută ca urmare a unei
D. agnozie vizuală
tumori cerebrale intraparenchimatoase se
E. hemianopsie laterală omonimă tratează prin
(pag. 251) A. echilibrarea aportului hidro-electrolitic
8 Sindromul Wallenberg este B. corticoterapie
A. sindrom altern C. derivaţie ventriculară externă
B. secundar infarctului protuberanţial bilateral D. antiepileptice
C. secundar ischemiei latero-bulbare E. antialgice
D. secundar infarctului în teritoriul vertebro-bazilar (pag. 274)
E. secundar ischemiei capsulei interne *4 Schwanomul acustic este o tumoră benignă
(pag. 251) derivată din
A. meninge
9 Sindromul pseudobulbar constă în
B. ependim
A. râs şi plâns spasmodic
C. celulele tecii Schwan
B. sindrom cerebelos bilateral
D. cortul cerebelului
C. tulburări de fonaţie şi deglutiţie
E. conductul auditiv intern
D. crize epileptice
(pag. 275)
E. demenţă
(pag. 252) *5 Riscul tratamentului chirurgical al
schwanomului acustic constă în
10 Bolile arterelor mici sunt
A. pareză de plex brachial
A. la originea infarctelor lacunare
B. crize epileptice
B. responsabile de 50% din infarctele cerebrale
C. paralizie facială periferică postoperatorie
C. produse prin lipohialinoza arterelor mici
D. sindrom de hipertensiune intracraniană
perforante
E. migrenă
D. generatoare de emboli cerebrale
(pag. 275)
E. legate de HTA sau diabet zaharat dezechilibrat
(pag. 252)
Pag. 27 din 130
6 Circumstanţele descoperirii tumorilor Tema nr. 24
intracraniene intraparenchimatoase sunt Febra acută la adult
A. epilepsie Bibliografie asociata temei:
B. cefalee Book Des ECN, editia in limba romana, redactor
C. sindrom de hipertensiune intracraniană Laurent Karila, 2011
pag. 309-311 * fara denumiri comerciale - ex. zovirax
D. deficit neurologic central instalat progresiv
E. pareză de sciatic popliteu extern *1 Termenul de febră acută recentă se referă la
(pag. 274) febra ce durează de:
A. peste 20 de zile
7 Sindromul de hipertensiune intracraniană are
în tabloul clinic următoarele semne şi B. 5-20 de zile
simptome C. mai puțin de 5 zile
A. cefalee predominant matinală D. mai mult de 30 de zile
B. sindrom parkinsonian E. mai puțin de 20 de zile
C. vărsături în jet (pag. 309)
D. edem papilar la examenul de fund de ochi
*2 Funcția de termoreglare este asigurată de:
E. vertij Meniere
A. hiotalamus
(pag. 274)
B. hipofiză
8 Glioamele se clasifică în C. epifiză
A. astrocitom D. suprarenale
B. meningiom E. corticosuprarenale
C. ependimom (pag. 309)
D. oligodendrogliom
*3 La toxicomanii pe cale intravenoasă,
E. glioblastom etiologia febrei acute poate fi:
(pag. 274) A. infecție urinară
9 Imagistica prin CT în cazul tumorilor B. hepatită alcoolică acută
cerebrale are următoarele roluri C. pancreatită acută
A. identifică tumora D. tuberculoză
B. identifică dimensiunea E. endocardită dreaptă
C. identifică tipul anatomo-patologic al tumorii (pag. 311)
D. identifică efectul de masă
*4 Pentru febra acută neînsoțită de semne de
E. identifică riscul de angajare toleranță dificilă și a semnelor orientative se
(pag. 274) recomandă:
A. antibioticoterapie
10 Aspectul tipic RMN pentru meningiom
constă în B. antiinflamatoare nesteroidiene
A. tumoră extranevraxială C. antiinflamatoare steroidieine
B. limite imprecise D. tratament simptomatic
C. baza de implantare meningiană extracentrală E. antivirale
D. captare intensă şi omogenă a substanţei de (pag. 311)
contrast
5 Dintre substanțele pirogene fac parte:
E. demielinizare extinsă în trunchiul cerebral
A. interleuchinele
(pag. 275)
B. lizozimul
C. factorul de necroză tumorală
D. interferonul
E. lactoferina
(pag. 309)

Pag. 28 din 130


6 Din categoria teren cu risc pentru febra Tema nr. 25
acută, fac parte : Meningitele infecţioase şi meningoencefalitele
A. nou-născuții adultului
B. gravidele Bibliografie asociata temei:
C. homosexualii Book Des ECN, editia in limba romana, redactor
Laurent Karila, 2011
D. subiecții cu proteze valvulare
pag. 289-293 * fara figuri si explicatiile acestora
E. subiecții recent operați
(pag. 309) *1 Care dintre următoarele condiții nu apare în
cursul evoluţiei unei meningite cu LCR
7 Dintre patologiile neinfecțioase, pot fi cauză purulent?
de febră acută:
A. şoc septic
A. bolile metabolice B. comă, epilepsie, hidrocefalie
B. intoxicația cu fier la menopauză C. sindrom de hipertensiune intracraniană
C. osteoporoza D. coagulare intravasculară diseminată (CIVD)
D. bolile inflamatorii sistemice E. dezvoltare de xantoame
E. tumorile hematologice (pag. 291)
(pag. 310)
*2 Care dintre următoarele afirmaţii referitoare
8 Care sunt urgențele infecțioase în fața unei la tratamentul etiologic al meningitelor este
febre acute: falsă?
A. meningoencefalita A. în meningita pneumococică se administrează
Cefotaxim ( ± Vancomicină) timp de 15 zile
B. meningita bacteriană
B. în meningita meningococică se administrează
C. faringoamigdalita virală
Cefotaxim timp de 7 zile
D. septicemia
C. în meningita cu Listeria monocytogenes se
E. endocardita infecțioasă administrează Amoxicilină + aminozide timp de
(pag. 311) 21 de zile
D. în meningita tuberculoasă se instituie
9 Etiologia febrei acute la pacientul alcoolic cvadriterapie antibacilară
poate fi:
E. în neuropaludism se administrează Aciclovir, 30
A. infecția cu HIV mg/kgc/zi, i.v, timp de 21 de zile
B. pneumopatia prin inhalare (pag. 289, 290, 292)
C. infecție urinară
*3 În cazul unui pacient diagnosticat cu
D. hepatită alcoolică acută meningită de etiologie probabil infecţioasă,
E. pancreatita acută care dintre situaţiile următoare nu
corespunde cu suspiciunea etiologică?
(pag. 311)
A. copii mici, nevaccinaţi, cu focare supurative în
* 10 Examene complementare mai specifice sfera ORL – se suspicionează implicarea
pentru o etiologie a febrei acute sunt: Haemophilus influenzae
A. transaminaze B. tineri ce prezintă un sindrom purpuric – se
suspicionează implicarea Neisseria meningitidis
B. testele serologice virale și bacteriene
C. pacient cu elemente de meningoencefalită acută,
C. hemoleucogramă
recent întors dintr-o călătorie în ţările scandinave
D. proteina C reactivă – se suspicionează neuropaludismul
E. VSH D. LCR clar, cu predominenţa limfocitelor şi
(pag. 311) hipoglicorahie- se suspicionează
neurotuberculoza
E. LCR clar, normoglicorahic, cu proteinorahie
<1g/l, pacient cu evoluţie spontană bună – se
suspicionează etiologia virală
(pag. 289-292)

Pag. 29 din 130


*4 Care dintre următoarele aspecte imagistice 8 Care dintre elementele următoare pot fi
(CT sau RMN) nu pledează pentru etiologia întâlnite în meningita pneumococică?
herpetică în cazul unui pacient internat cu
A. sindrom meningean cu debutul brutal şi tulburări
diagnosticul de meningoencefalită acută?
de conştienţă
A. leziuni intracraniene bilaterale B. suferinţă neuromeningee apărută la persoanele
B. leziuni intracraniene asimetrice vârstnice, alcoolici, pacienţi splenectomizaţi sau
cu traumatisme craniene în antecedente
C. leziuni intracraniene ce capteaza substanta de
contrast C. frotiu din LCR ce depistează coci Gram negativi
în diplo
D. anomalii radiologice temporale interne
D. frotiu din LCR ce depistează coci Gram pozitivi
E. anomalii radiologice periventriculare
E. LCR purulent
(pag. 291)
(pag. 289)
*5 Care dintre următoarele aspecte ale LCR nu
se corelează cu suspiciunea etiologică? 9 Care dintre următoarele meningite se declară
serviciului epidemiologic?
A. LCR purulent cu vizualizarea de coci Gram
negativi – probabil meningococ A. meningita meningococică
B. LCR purulent cu vizualizarea de coci Gram B. meningita pneumococică
pozitivi – probabil pneumococ C. meningita tuberculoasă
C. LCR purulent cu evidenţierea de bacili Gram D. meningita cu Listeria monocytogenes
pozitivi – posibil Listeria monocytogenes
E. meningita cu Haemophilus influenzae
D. LCR clar, cu predominenţă limfocitară, asociind
prezenţă de hematii, cu normoglicorahie şi fără (pag. 290)
evidenţierea unui germen pe frotiu la examenul
10 Care dintre elementele următoare pot sugera
direct – posibil malarie cerebrală
diagnosticul de meningită cu haemophilus
E. LCR clar, cu predominenţă limfocitară şi influenzae?
hipoglicorahie – probabil tuberculoză
A. suferinţă meningeană ce apare la copii
(pag. 289-291) nevaccinaţi, cu focare supurative în sfera ORL
6 În care dintre următoarele situaţii clinice B. frotiu din LCR ce evidenţiază bacili Gram negativi
trebuie întotdeauna suspectat diagnosticul C. LCR purulent
de meningită infecţioasă?
D. LCR clar
A. pacient ce prezintă sindrom meningean
E. LCR hemoragic
B. bolnav icteric
(pag. 290)
C. in contextul unui sindrom infecţios (febră)
D. pacient cu trismus, contracturi musculare şi Tema nr. 26
tulburări vegetative Gripa
E. pacient cu paralizii flasce, predominent la Bibliografie asociata temei:
rădăcina membrelor, asimetrice, cu evoluţie Book Des ECN, editia in limba romana, redactor
ascendentă Laurent Karila, 2011
(pag. 289) pag. 320-324 * fara epidemiologie (cap I. pct.3), fara
denumiri comerciale (ex. Tamiflu, Relenza)
7 Care dintre elementele următoare se
întâlnesc în meningita meningococică? *1 Care dintre următoarele măsuri nu contribuie
A. debut brutal al unui sindrom meningean la întreruperea transmiterii virusului gripal?
B. suferinţă prodromală în sfera ORL (rinofaringită A. izolarea pacienţilor şi a cazurilor suspecte
acută), febră, cefalee, vărsături B. spălarea sistematică a mâinilor cu soluţii
C. sindrom meningean şi decelarea unei purpure hidroalcoolice (pacienţi, vizitatori, personal
echimotice la nivel tegumentar sanitar)
D. sindrom meningean şi depistarea de coci Gram C. purtarea măştilor de protecţie de către pacienţi /
pozitivi, în diplo, lanceolaţi pe frotiul din LCR personalul de îngrijire
E. suferinţă neuromeningee diagnosticată mai D. purtarea ochelarilor de protecţie, a halatului şi a
frecvent la tineri si la cei cu deficit de mănuşilor de protecţie de către personalul de
complement seric îngrijire aflat în contact cu pacienţii
(pag. 290) E. permisiunea de vizitare a pacienţilor de către rude
(pag. 323)

Pag. 30 din 130


*2 Care dintre următoarele afirmaţii referitoare 6 Care dintre următoarele afirmaţii referitoare
la diagnosticul gripei este falsă? la virusurile gripale sunt adevărate?
A. contextul epidemic este important pentru A. virusurile gripale au un genom bazat pe ADN
stabilirea diagnosticului de gripă B. trei genuri sunt patogene pentru om: Myxovirus
B. perioada de incubaţie în gripă este lungă influenza A, B şi C
(aproximativ 10 zile) C. toate cele trei genuri sunt patogene strict pentru
C. simptomatologia prodromală (febră ridicată, om
frisoane, artromialgii, cefalee, stare generală de D. subtipurile virale sunt clasificate în funcţie de
rău) este puţin specifică tipurile de hemaglutinină şi de neuraminidază
D. tabloul radiologic nu relevă semne specifice E. recomandările genetice sunt responsabile de
E. suprainfecţiile bacteriene sunt responsabile de o epidemiile gripale anuale şi de pandemii
mare parte dintre decese (pag. 320)
(pag. 321-322)
7 Care dintre următoarele subtipuri ale
*3 În care dintre situații, diagnosticul de gripă virusului gripal au determinat pandemii în
este improbabil: secolul XX ?
A. bolnav cu manifestări de viroză respiratorie în A. H1N1
context epidemic gripal
B. H2N2
B. simptome respiratorii şi contact recent cu un
C. H3N2
pacient confirmat cu gripă
D. H5N1
C. debut brusc al unei suferinţe respiratorii de
aspect viral, după o incubaţie scurtă (1-3 zile) E. H1N1v
D. tablou clinic caracterizat de febră ridicată, (pag. 323-324)
frisoane, semne respiratorii, artromialgii, cefalee,
ameţeli 8 Care dintre următoarele afirmaţii referitoare
la fiziopatologia gripei sunt adevărate?
E. tablou hematologic periferic caracterizat prin
leucocitoză şi hipereozinofilie A. virusul gripal penetrează celular cu ajutorul
neuraminidazei
(pag. 321)
B. după penetrare virusul gripal se replică
*4 Care complicaţie nu apare în infecţia cu intracelular
virusul gripal?
C. eliberarea noilor virioni este facilitată de către
A. suprainfecţii bacteriene hemaglutinină
B. decompensare cardiacă la persoane D. la nivelul aparatului respirator virusul gripal
diagnosticate cu insuficienţă cardiacă gravă sau determină liză celulară şi reacţii inflamatorii
valvulopatii grave E. suprainfecţiile bacteriene apar obligatoriu în
C. decompensarea respiratorie la persoanele evoluţia gripei
cunoscute cu bronhopneumopatii cronice (pag. 320)
D. sindromul de detresă respiratori acută
9 În suspiciunea de gripă, care dintre
E. anizocorie următoarele explorări biologice pot fi
(pag. 321) utilizate pentru confirmare?

*5 Care dintre afirmaţiile următoare referitoare A. izolarea virusului gripal pe culturi celulare
la tratamentul gripei este falsă ? B. detectarea structurilor genomice prin RT-PCR
A. în prezent se utilizează următoarele antivirale cu C. detectarea antigenelor virale prin tehnica ELISA
acţiune specifică: oseltamivir, zanamivir, D. detectarea anticorpilor specifici prin
amantadină imunofluorescenţă indirectă (probă din prelevatul
B. antibioticele antibacteriene nu se folosesc decât nazal)
în cazurile de suprainfecţie dovedită E. detectarea anticorpilor specifici prin reacţia Paul-
C. este indicată hidratarea bolnavului pentru Bunnell-Hăngănuţiu (probă de sânge)
combaterea pierderilor secundare febrei ridicate (pag. 322)
D. se utilizează medicamente antalgice, respectiv
antipiretice
E. se indică repausul fizic
(pag. 322)

Pag. 31 din 130


10 Următoarele categorii de persoane au *4 Tabloul clinic al pneumocistozei pulmonare
indicaţie de vaccinare antigripală: este de tip:
A. sugarii peste vârsta de 3 luni A. pneumonie francă lobară
B. vârstnicii (peste 65 de ani) B. abces pulmonar
C. persoanele cu suferinţe bronhopulmonare C. pneumopatie interstițială febrilă, rezistentă la
cronice antibioticele clasice
D. gazdele speciale care prezintă deficite imunitare D. sindrom pertusiform cu expectorație purulentă
celulare (mai ales infecţia HIV) E. sindrom al disfuncţiei reactive de căi aeriane
E. persoanele cunoscute ca fiind alergice la (RADS)
ovalbumină (pag. 327)
(pag. 323)
*5 Cea mai frecventă localizare a infecției cu
Tema nr. 27 citomegalvirus la pacienții infectați cu HIV
Infecţia cu HIV este:
Bibliografie asociata temei: A. hepatică
Book Des ECN, editia in limba romana, redactor B. ganglionară
Laurent Karila, 2011 C. pulmonară
pag. 325-328 * fara denumiri comerciale ex.
Bactrium, fara date epidemiologice, fara pc. VI D. retiniană
Altele - care care cuprinde informatii despre E. cerebrală
legislatia Franceza (pag. 327)
*1 Diagnosticul infecției cu HIV se bazează pe: 6 Principalele enzime din structura HIV sunt:
A. tipare limfocitară CD4/CD8 A. transcriptaza inversă
B. test ELISA combinat de generația a patra B. proteaza
C. determinarea nivelului limfocitelor CD4 C. neuraminidaza
D. test genotipic de rezistență D. integraza
E. imunofenotipare E. hemaglutinina
(pag. 325) (pag. 325)
*2 Care este cea mai frecventă infecție 7 Care dintre enunțurile referitoare la
oportunistă întâlnită la pacienții infectați cu tratamentul antiretroviral sunt adevărate:
HIV:
A. se recomandă pacienților cu CD4>500 cel/mm³
A. criptococoza
B. cuprinde clasic asocierea de 3 antiretrovirale
B. toxoplasmoza cerebrală
C. este indicat pacienților simptomatici
C. pneumocistoza pulmonară
D. se recomandă pacienților cu CD4<200 cel/mm³
D. sarcomul Kaposi
E. scade morbiditatea și mortalitatea, refăcând
E. leucoencefalopatia multifocală progresivă parțial capacitatea imună
(pag. 327) (pag. 326)
*3 Profilaxia primară pentru pneumocistoză și 8 Care dintre enunțurile referitoare la
toxoplasmoză la pacienții infectați cu HIV se toxoplasmoza cerebrală sunt false:
face cu:
A. trebuie avută în vedere la un pacient infectat cu
A. cotrimoxazol HIV în prezența oricărui semn neurologic
B. fluconazol B. în general apare la pacienții cu CD>500 cel/mm³
C. pirimetamină C. tratamentul se face cu fluconazol
D. ganciclovir D. aspectul imagistic este de leziuni rotunjite, cu
E. foscarnet aspect în cocardă

(pag. 326) E. nu este necesară profilaxia secundară


(pag. 327)

Pag. 32 din 130


9 În legătură cu leucoencefalopatia multifocală *3 Care dintre urmatoarele afirmatii este falsa?
progresivă sunt adevărate afirmaiile:
A. IST sunt infectii asimptomatice
A. este determinată de Cryptococcus neoformans B. IST prezinta arsura, prurit, durere
B. se manifestă prin tulburări neurologice grave C. IST prezinta scurgere
C. este o suferință demielinizantă D. IST poate produce prostatite
D. se tratează cu voriconazol E. IST poate produce salpingite
E. este determinată de virusul JC (pag. -)
(pag. 327)
*4 Diagnosticul de IST nu se pune pe:
10 Profilaxia transmiterii materno-fetale a
A. TPHA, VDRL
infecției cu HIV cuprinde:
B. Serologie virusurilor hepatitice A, B, C
A. tratarea mamei pe timpul sarcinii
C. Serologie HIV 1 si 2
B. alimentația la sân a nou-născutului timp de 6
săptămâni D. Prelevat uretral sau vaginal
C. profilaxia cu cotrimoxazol E. Doar pe examenul clinic
D. tratament profilactic antiretroviral pentru nou- (pag. -)
născut timp de 6 săptămâni
*5 Managementul unei IST nu cuprinde
E. aducerea la zi a vaccinărilor
A. Profilaxia primara si secundara
(pag. 328)
B. Tratamentul curativ
Tema nr. 28 C. Vaccinarea anti-HPV indiferent de varsta
Boli cu transmitere sexuală D. Profilaxia recidivelor
Bibliografie asociata temei: E. Tratamentul factorilor de risc
Book Des ECN, editia in limba romana, redactor (pag. -)
Laurent Karila, 2011
pag. 329-333 * fara date epidemiologice specifice 6 Infectia cu gonococ are urmatoarele
Frantei, fara denumiri comerciale (ex. Extencilline) caracteristici:

*1 Care dintre urmatoarele afirmatii referitoare A. Are perioda de incubatie intre 14-21 zile
la infectiile cu transmitere sexuala (IST) nu B. Are perioada de incubatie intre 2-7 zile
este corecta? C. La barbat manifestarea clinica este de uretrita
A. In prezenta unei IST se vor cauta intotdeauna clasica
alte IST. D. Boala nu apare niciodata la nou-nascut
B. Se va insista asupra masurilor de profilaxie. E. La femei poate produce sterilitate tubara
C. Antecedentele de IST nu constituie un factor de (pag. -)
risc
7 Care dintre urmatoarele afirmatii privind
D. Se vor depista si trata partenerii
infectiile gonococice sunt adevarate?
E. IST cuprind si formele asimptomatice de boala
A. Diagnosticul este sugerat de examenul clinic
(pag. -)
B. Examenul direct este concludent atat la barbati
*2 Care dintre urmatoarele infectii nu este IST cat si la femei
A. Infectiile microbiene stafilococice si C. Examenul direct arata prezenta diplococilor
streptococice Gram negativi in boabe de cafea
B. Infectiile cu gonococ D. In formele neclasice se poate realiza o cultura pe
medii speciale
C. Infectiile cu chlamidia
E. Tratamentul se face cu ceftriaxona 250 mg im
D. Sifilisul
(pag. -)
E. Ectoparazitozele (scabia si pediculozele)
(pag. -)

Pag. 33 din 130


8 Infectia cu chlamidia trachomatis are *2 Cele mai frecvente infecţii nosocomiale sunt:
urmatoarele caracteristici:
A. pneumopatiile infecţioase
A. Produce uretrite si cervicite B. infecţiile urinare
B. Serotipurile L1, L2, L3 produc C. infecţiile digestive
limfogranulomatoza venerica
D. infecţiile leagate de catetere
C. Sindromul Fitz-Hugh-Curtis reprezinta o
colecistita acuta E. infecţiile de plagă operatorie
D. Sindromul Fiessinger-Leroy- Reiter este alcatuit (pag. 340)
doar din uretrita si poliartrita
*3 Infecţiile nosocomiale de plagă operatorie se
E. Tratamentul uretritei si cervicitei se face cu definesc etiologic prin:
azitromicina 1g per os
A. aspect clinic: prezența obligatorie a secrețiilor
(pag. -) purulente
9 Care dintre urmatoarele afirmatii ale infectiei B. serologic: teste bacteriologice pozitive
sifilitice sunt adevarate: C. microbiologic: cultură pozitivă dintr-o probă
A. Este produsa de o spirocheta recoltată de la nivelul plăgii operatorii
B. Are trei stadii: sifilis primar, secundar si tertiar D. semne clinice şi biologice apărute în primele 15
zile de la momentul intervenţiei, indiferent de
C. Neurosifilisul apare doar in stadiul tertiar tipul acesteia
D. Diagnosticul serologic cuprinde doua teste: unul E. semne clinice şi biologice apărute în decurs de
treponemic, altul netreponemic 30 de zile de la intervenţie, în cazul implantării de
E. VDRL este un test sensibil dar putin specific material străin
(pag. -) (pag. 340)

10 Tratamentul sifilisului se face cu: *4 Privitor la măsurile de izolare instituite la


identificarea unor infecţii nosocomiale sunt
A. De electie sunt ciclinele
adevărate următoarele, cu excepţia:
B. Penicilina G sau cicline in caz de alergie
A. este indicată purtarea măştii de către pacientul
C. In sifilisul precoce se administreaza 2,4 Mui cu tuberculoză
benzatin penicilina x3 injectii saptamanale
B. este indicată purtarea măştii de către personalul
D. In neurosifilis se administreaza penicilina G 18- de îngrijire a pacientului cu tuberculoză
24 Mui/zi, 14-21 zile
C. limitarea ieşirilor şi vizitelor pacientului cu
E. In sifilisul tardiv se poate utiliza Doxiciclina 200 tuberculoză până la momentul externării
mg/zi 18 zile
D. izolarea pacientului imunodeprimat pentru
(pag. -) evitarea contaminării cu microbi oportunişti

Tema nr. 29 E. limitarea transportului pacienţilor cu infecţie


nosocomiale cu bacterii multirezistente
Infecţiile nosocomiale
(pag. 341)
Bibliografie asociata temei:
Book Des ECN, editia in limba romana, redactor *5 Pneumopatiile infecţioase nosocomiale se
Laurent Karila, 2011 caracterizează prin următoarele, cu excepţia:
pag. 339-341 * fara date procentuale intrucat nu A. principalul factor de risc este infectarea
sunt valabile pentru Romania, fara epidemiologie endotraheală

*1 Referitor la infecţiile nosocomiale sunt B. semne clinice: febră, expectoraţii purulente


adevărate următoarele aspecte, cu excepţia: C. semne biologice: hemoculturi pozitive
A. sunt infecţii asociate îngrijirilor medicale D. semne biologice: leucocitoza
B. sunt infecţii asociate activităţilor de îngrijire E. semne radiologice: opacităţi pulmonare recent
curativă apărute
C. sunt infecţii asociate activităţilor de profilaxie (pag. 340)
D. sunt infecţii asociate activităţilor de diagnostic
E. sunt infecții asociate automedicației la domiciliu
(pag. 339)

Pag. 34 din 130


6 Semnele biologice pe care se bazează 10 Măsurile profilactice ale infecţiilor
diagnosticul infecţiilor pulmonare nosocomiale cuprind:
nosocomiale sunt:
A. spălarea mîinilor înaintea fiecărei îngrijiri
A. hemoculturi pozitive acordate
B. izolarea unei bacterii din lavajul bronhoalveolar B. spălarea mîinilor după fiecare îngrijire acordate
C. izolarea unei bacterii din expectoraţie C. purtarea mănuşilor sterile indiferent de tipul de
îngrijire acordată
D. leucocitoză cu neutrofilie
D. sterilizarea instrumentarului
E. leucopenie cu limfocitoză
(pag. 340) E. purtarea măştilor
(pag. 340-341)
7 Referitor la infecţiile nosocomiale de cateter
sunt adevărate următoarele: Tema nr. 30
A. sunt mai frecvent localizate la cateterele centrale Septicemiile
decât la cele periferice Bibliografie asociata temei:
B. sunt mai frecvent localizate la cateterele Book Des ECN, editia in limba romana, redactor
periferice decât la cele centrale Laurent Karila, 2011
C. sunt cele mai frecvente tipuri de infecţiile pag. 370-375 * fara date epidemiologice
nosocomiale
*1 Care dintre următoarele date definesc
D. diagnosticul pozitiv se stabileşte prin minimum
bacteriemia?
trei culturi pozitive de la nivelul cateterului
E. diagnosticul pozitiv se stabileşte print-o cultură A. prezența de bacterii în sânge obiectivată prin
pozitivă de la nivelul cateterului una sau mai multe hemoculturi
(pag. 340) B. răspunsul inflamator al organismului la o
agresiune, infecţioasă sau nu
8 Definirea infecţiilor urinare nosocomiale se C. o infecţie sistemică confirmată
bazează pe următoarele criterii:
D. o infecţie asociată cu hipotensiune
A. epidemiologic: infecţie aflată în perioada de
incubaţie la momentul internării într-o unitate E. o infecţie asociată cu disfuncţia unui organ
sanitară (pag. 370)
B. clinic: febră, polachiurie, imperiozitate micţională, *2 Care dintre următoarele date definesc
dureri pelvine Sindrom de Răspuns Inflamator Sistemic
C. biologic: prezenţa unei bacteriurii asimptomatice (SIRS)?
D. biologic: examen citobacteriologic al urinei A. prezenţa de bacterii în sânge obiectivată prin
pozitiv (cultură>100 CFU/ml+leucociturie una sau mai multe hemoculturi
>100/ml)
B. răspunsul inflamator al organismului la o
E. biologic: examen citobacteriologic al urinei pozitiv agresiune, infecţioasă sau nu
(cultură>1000 CFU/ml+leucociturie >10000/ml)
C. bacteriemie si noţiunea clinică a impactului
(pag. 340) acestei bacteriemii
9 Diagnosticul pneumopatiilor infecţioase D. o infecţie confirmată clinic şi/sau microbiologic
nosocomiale se bazează pe următoarele E. o infecţie asociată cu disfuncţia unui organ
semne:
(pag. 370)
A. clinice: tuse, dispnee, wheezing
B. clinice: febră, expectoraţii purulente
C. biologice: hemoculturi pozitive
D. radiologice: opacităţi pulmonare recent apărute
E. radiologice: infiltrat interstiţial recent apărut
(pag. 340)

Pag. 35 din 130


*3 Care dintre următoarele date definesc şocul 7 Care dintre următoarele date asociate unei
septic? infecţii cu criterii de SIRS definesc sepsisul
grav?
A. depsis şi hipotensiune arterială
B. depsis şi insuficienţă respiratorie (hipoxemie A. hipotensiune arterială: presiunea arterială sub 90
inexplicabilă) mmHg sau scădere cu 40 mmHg

C. depsis şi disfuncţie neurologică: confuzie, B. hipoperfuzie periferică cu cianoză, marmorare


agitaţie, dezorientare, torpoare C. insuficienţă respiratorie: hipoxemie inexplicabilă
D. depsis şi disfuncţie hematologică: tulburări de D. tulburări de coagulare
coagulare
E. hiperleucocitoză peste 12000/mm³, leucopenie
E. depsis grav cu hipotensiune arterială persistentă sub 4000/mm³ sau mai mult de 10% celule
în ciuda unei umpleri vasculare adaptate (cel imature
puţin 500 ml şi/sau necesitatea de a recurge la (pag. 370)
amine vasoactive)
(pag. 371) 8 În cazul unui pacient cu criterii de sepsis
care dintre următoarele date constituie
*4 Care dintre următoarele microorganisme semne de gravitate imediată?
sunt mai frecvent incriminate în etiologia
unui sepsis cu punct de plecare pulmonar, în A. hipotensiune şi semne de anomalie a perfuziei
tisulare
cazul unui pacient alcoolic?
B. febră peste 38ºC sau sub 36ºC
A. stafilococi, streptococi
C. semne cutanate: cianoză, marmorare, creşterea
B. enterobacterii
timpului de recolorare cutanată, scăderea locală
C. pneumococi, Klebsiella pneumoniae a temperaturii cutanate
D. pseudomonas sp, enterococi D. oligurie: diureză spontană sub 0,5 ml/kg/oră
E. bacili Gram negativi, stafilococi E. anomalii ale funcţiilor superioare
(pag. 373) (pag. 371)

*5 Ce tratament etiologic de primă intenţie aţi 9 Care dintre afirmaţiile următoare referitoare
aplica în cazul unui pacient diagnosticat cu la tratamenul sepsisului sunt adevărate?
sepsis, fără semne de gravitate, la care
A. este iniţiat de urgenţă după recoltarea probelor
presupuneţi implicarea unor enterobacterii
microbiologice
dobândite în mediu comunitar?
B. este iniţial probabilistic, ghidat de contextul clinic
A. monoterapie cu Cefotaxim sau Ceftriaxonă sau şi epidemiologic
fluorochinolonă
C. se face pe cale iv, cel mai adesea in biterapie,
B. oxacilină, cloxacilină pe o perioadă de 15 zile, sau mai mult
C. ticarcilină sau piperacilină asociată cu amikacină D. se face per os, în monoterapie, pe o perioadă de
sau isepamicină cel putin 3 săptămâni
D. amoxicilina E. va fi ulterior adaptat rezultatelor microbiologice
E. amoxicilină asociată cu gentamicină sau (pag. 374)
netilmicină
(pag. 374) 10 Persistenţa febrei la un pacient cu sepsis
impune:
6 Definiţia clinică a sindromului de răspuns A. repetarea hemoculturilor
inflamator sistemic cuprinde:
B. identificarea unor noi localizări secundare
A. febră peste 38ºC sau sub 36ºC
C. continuarea tratamentului iniţiat, febra nefiind un
B. frecvenţa cardiacă peste 90/min indicator fidel al eficienţei terapeutice
C. oligurie D. verificarea porţii de intrare
D. frecvenţa respiratorie peste 20/min E. identificarea unei complicaţii iatrogene
E. hiperleucocitoză peste 12000/mm³, leucopenie (pag. 375)
sub 4000/mm³ sau mai mult de 10% celule
imature
(pag. 370)

Pag. 36 din 130


Tema nr. 31 *5 Tratamentul în diareea acută implică:
Diareea acută şi deshidratarea la adult A. spitalizare obligatorie
Bibliografie asociata temei: B. rehidratare parenterală în caz de deshidratare
Book Des ECN, editia in limba romana, redactor severă
Laurent Karila, 2011 C. modulatoare de motilitate cu efect de încetinire a
pag. 403-407 * fara date epideologice, fara denumiri tranzitului intestinal în suspciunile de diaree
comerciale (ex. Tiorfan) invazivă
D. rehidratare orală în caz de deshidratare severă
*1 Diareea acută se defineşte ca:
E. antibioticoterapie empirică, indicată în toate
A. emisia de minimum 5 scaune moi sau lichide, cu cazurile de diaree acută, până la elucidarea
debut brusc şi durata de mai puţin de 14 zile cauzei
B. emisia unui scaun lichid sau moale şi abundent (pag. 406-407)
C. emisia de mai mult de 2 scaune moi sau lichide,
cu debut brusc şi durata de mai puţin de 14 zile 6 Sindromul holeriform se caracterizează prin:

D. emisia de mai mult de 2 scaune moi sau lichide, A. diaree "banală", puţin severă
cu debut brusc şi durata de minimum 14 zile B. diaree profuză şi apoasă
E. emisia de scaune numeroase, puţin abundente şi C. diaree cu scaune numeroase, reduse cantitativ şi
care conţin sânge şi puroi conţinut bogat de glere, sânge şi puroi
(pag. 403) D. risc de deshidratare severă, în absenţa terapiei
*2 Sindromul gastroenteric se caracterizează E. evoluţie rapid favorabilă în majoritatea cazurilor
prin: (pag. 404)
A. diaree profuză şi apoasă
7 Sindromul dizenteric se caracterizează prin:
B. evoluţie rapid favorabilă
A. afectare intestinală
C. evoluţie letală în absenţa tratamentului
B. afectarea invazivă a colonului
D. afectare colică invazivă
C. sindrom septic moderat sau chiar absent
E. risc de şoc septic
D. risc de şoc septic
(pag. 404)
E. diaree cu scaune numeroase, reduse cantitativ şi
*3 În etiologia sindromului dizenteric este conţinut bogat de glere, sânge şi puroi
implicat: (pag. 404)
A. clostridium perfrigens 8 Privitor la toxiinfecţiile alimentare
B. calicivirus stafilococice sunt adevărate următoarele:
C. rotavirusuri A. se definesc prin apariţia a minimum trei cazuri
D. shigella sp. grupate, cu simpromatologie similară, în general
digestivă, a căror cauză poate fi corelată cu
E. vibrio cholerae aceeaşi origine alimentară
(pag. 405-406) B. incubaţie lungă
*4 În diareea acută, coprocultura pe medii C. incubaţie scurtă
selective este indicată în caz de: D. vărsăturile sunt în prim plan, diareea în plan
A. sindrom dizenteric secund
B. sindrom gastroenteritic E. diareea este în prim plan, vărsăturile în plan
secund
C. diaree > 5 zile
(pag. 405)
D. diaree apărută după un tratament antibiotic
E. diaree apărută după chimioterapie
(pag. 404)

Pag. 37 din 130


9 Febra tifoidă se caracterizează prin *3 Sunteţi în faţa unui pacient vaccinat
următoarele aspecte, cu excepţia: antitetanic incomplet, care prezintă o plagă
cutanată cu risc tetanigen foarte crescut.
A. clinic: erupţie rozeoliformă, puls disociat, scaune
Care dintre măsurile următoare nu este
diareice
indicată pentru profilaxia tetanosului?
B. biologic: leucocitoză, citoliză hepatică
A. rapel antitetanic
C. biologic: leucopenie, citoliză hepatică
B. aducerea la zi a vaccinării conform schemelor
D. coprocitologic: hematii și eozinofile abundente recomandate
E. hemoculturile şi serologia Widal pot ajuta la C. administrarea de anticorpi umani specifici (IgG,
stabilirea diagnosticului 500 ul)
(pag. 406) D. injectarea de permanganat de potasiu (soluţie
1%) în jurul plăgii
10 Privitor la tratamentul cu antibiotice
administrat în diareea acută, sunt adevărate E. antibioterapie empirică
următoarele: (pag. 369)
A. trebuie iniţiat după recoltarea probelor biologice
*4 Sunteţi în faţa unui pacient, victimă a unui
B. macrolidele sunt indicate în salmoneloze accident rutier, ce prezintă multiple plăgi
C. chinolonele sunt indicate în salmoneloze murdare; pacientul a fost adus tardiv la
medic, la locul accidentului efectuându-se
D. doxiciclina este indicată în shigeloze doar o debridare incompletă. Ţinând cont de
E. metronidazolul este indicat în diareea cu statusul vaccinal, care dintre următoarele
Clostridium difficile afirmaţii este falsă?
(pag. 407) A. rapel antitetanic în urmă cu 3 ani - instituiţi
antibioterapie, seroterapie şi vaccinare completă
Tema nr. 32
B. rapel antitetanic în urmă cu 11 ani - efectuaţi
Profilaxia tetanosului rapel, administrare de imunoglobuline specifice,
Bibliografie asociata temei: antibioterapie
Book Des ECN, editia in limba romana, redactor C. rapel antitetanic în urmă cu 8 ani - efectuaţi un
Laurent Karila, 2011 nou rapel şi instituiţi antibioterapie
pag. 367-369 * fara date epidemiologice D. pacient cu vaccinare incompletă - practicaţi
antibioterapie, rapel antitetanic, aducerea la zi a
*1 Care dintre următoarele afirmaţii referitoare vaccinării specifice, administrare de
la managementul unui caz de tetanos este imunoglobuline specifice
falsă?
E. vaccinare absentă sau nesigură - vaccinare
A. plaga se curăţă şi se debridează completă, imunoglobuline specifice, antibioterapie
B. plaga se acoperă cu unguente grase şi se (pag. 369)
pansează ocluziv
*5 Care dintre următoarele afirmaţii referitoare
C. se practică antibioterapie (Penicilina G la vaccinarea antitetanică este falsă?
administrată i.v.)
A. primovaccinarea se practică la sugar şi constă în
D. se practică seroterapie (administrarea de
trei administrări la interval de o lună (2, 3 şi 4 luni
anticorpi umani specifici, cu rol neutralizant al
de viaţă)
toxinei)
B. primul rapel se efectuază înainte de vârsta de 18
E. bolnavul va fi ulterior vaccinat antitetanic
luni
(pag. 368)
C. rapeluri se practică ulterior la 6 ani, 11-13 ani şi
*2 Care tip de complicaţie nu este specifică 16-18 ani
tetanosului? D. la adult se practică rapel odată la 25 de ani
A. complicaţii tromboembolice E. vaccinul conţine o toxină fără putere patogenă
B. suprainfecţii pulmonare (prin aspiraţie) (pag. 369)
C. nevralgii gambiere simetrice, cu caracter de
arsură
D. sindroame disautonome
E. decompensarea tarelor preexistente
(pag. 368)

Pag. 38 din 130


6 Care din următoarele afirmaţii referitoare la 9 Pentru diagnosticul tetanosului, care dintre
fiziopatologia şi profilaxia tetanosului sunt următoarele date clinice sunt adevărate?
adevărate?
A. incubaţia este în medie de 7 zile
A. tetanosul este o boală infecţioasă datorată B. simptomul inaugural este trismusul (permanent,
efectului neurogen al exotoxinei produse de ireductibil, fără febră asociată)
Clostridium difficile
C. simptomul inaugural este ptoza palpebrală
B. pătrunderea microbului în organism se face strict simetrică
prin intermediul unei plăgi cutanate
D. contracturile musculare cuprind iniţial membrele,
C. bacilul tetanic se dezvoltă în condiţii de apoi trunchiul şi faţa
anaerobioză
E. contracturile generalizate sunt responsabile de
D. toxina se deplasează pe cale axonală, în mod opistotonus
centripet şi blochează eliberarea de
neurotransmiţători la nivel sinaptic, ceea ce (pag. 368)
induce spasticitate musculară
10 Care dintre următoarele afirmaţii referitoare
E. profilaxia specifică apelează la un vaccin ce la profilaxia tetanosului sunt adevărate?
conţine anatoxina tetanică
A. vaccinarea antitetanică este obligatorie
(pag. 367-369)
B. vaccinul contine antitoxina tetanică
7 Care dintre următoarele afirmaţii referitoare C. nu există contraindicaţii de vaccinare
la toxina produsă de Clostridium tetani sunt
adevărate? D. primovaccinarea se efectuează la copilul mic sub
un an şi constă din trei administrări de produs
A. tetanospasmina nu are efect decât dacă biologic
subiectul nu este vaccinat (sau este vaccinat
incorect) şi dacă local sunt îndeplinite condiţii E. in faţa oricărei plăgi trebuie verificat statusul
propice (anaerobioză) vaccinal al pacientului

B. toxina se deplasează pe cale axonală în mod (pag. 369)


centripet
Tema nr. 33
C. paraliziile produse sunt flasce şi au caracter Hemoragia digestivă
asimetric, ascendent
Bibliografie asociata temei:
D. la nivel sinaptic, este blocată eliberarea unor
neurotransmiţători, ceea ce induce spasticitate Book Des ECN, editia in limba romana, redactor
musculară Laurent Karila, 2011
pag. 426-429
E. toxina este responsabilă de apariţia de deficite
senzoriale
*1 În clasificarea Forrest a hemoragiilor
(pag. 367) digestive superioare, scorul Forrest IB
semnifică
8 În faţa căror situaţii clinice trebuie avut în
vedere diagnosticul de tetanos? A. Hemoragie în pânză
A. plăgi cutanate sau mucoase B. Vas vizibil nehemoragic
B. plăgi ce crează condiţii locale de anaerobioză C. Cheag aderent
(ischemie, necroze) D. Pete pigmentare
C. paralizii simetrice, flasce, descendente E. Hemoragie în jet
D. trismus decelat la o persoană în vârstă provenită (pag. 427)
din Franţa
E. sugar provenit din ţări în curs de dezvoltare, ce *2 Hemoragiile diverticulare sunt favorizate de
prezintă dificultăţi de alimentare administrarea
(pag. 367-368) A. Aspirinei
B. Omeprazolului
C. Enalaprilului
D. Furosemidului
E. Metoclopramidului
(pag. 428)

Pag. 39 din 130


*3 Hemoragia digestivă superioară este definită 8 Producerea unei hemoragii digestive
ca hemoragie provenită din tubul digestiv inferioare la pacienţii cu cancer de colon
situat este favorizată de un tratament cu
A. În aval de unghiul duodenojejunal A. Antiagregante plachetare
B. În aval de flexura splenică B. Prokinetice
C. În aval de valvula ileocecală C. Anticoagulante
D. În amonte de unghiul duodenojejunal D. Antidiareice
E. În aval de flexura hepatică E. Hipolipemiante
(pag. 426) (pag. 428)

*4 Circumstanțele în care se efectuează 9 Conduita inițială în hemoragiile digestive


endoscopie digestivă superioară în urgență include
include
A. Tușeul rectal
A. Pacientul cu instabilitate hemodinamică marcată B. 2 căi venoase periferice sau o cale centrală în
B. La toți pacienții intubați și ventilați funcție de gravitate
C. Cu scop exclusiv diagnostic inițial C. Endoscopie digestivă chiar din momentul primirii
pacientului
D. La pacientul a jeun de 6 ore sau după golirea
stomacului prin administrarea unei perfuzii D. Administrare de octreotid în caz de ulcer gastric
intravenoase lente cu eritromicină hemoragic
E. La 72 ore de la debutul sângerării pentru a E. Oxigenoterapie
aștepta oprirea spontană a acesteia (pag. 426-427)
(pag. 427)
10 La pacienţii cu leziuni ulcerate
*5 La un pacient cu hemoragie digestivă gastroduodenale complicate cu hemoragie
inferioară cu colonoscopie normală și digestivă superioară, în caz de eşec repetat
hemoragie care persistă și este abundentă, al tratamentului endoscopic, se recomandă
se va avea în vedere efectuarea de urgență a
A. Embolizare arterială pe cale radiologică
A. Tomografiei computerizate B. Intervenţie chirurgicală
B. Rezonanței magnetice nucleare C. Tratament injectabil cu inhibitori de pompă de
C. Arteriografiei celiomezenterice protoni
D. Examinării cu videocapsula endoscopică D. Administrare intravenoasă de hemostatice
E. Laparoscopiei diagnostice E. Tratament cu sucralfat în doză de 4 grame pe zi
(pag. 428) (pag. 428)

6 Tratamentul hemoragiilor digestive Tema nr. 34


superioare cu inhibitori de pompă de protoni Hepatitele virale
(IPP) intravenos cu seringă electrică (IVSE)
este indicat în Bibliografie asociata temei:
Book Des ECN, editia in limba romana, redactor
A. Hemoragia în pânză
Laurent Karila, 2011
B. Vas vizibil nehemoragic pag. 450-458
C. Cheag aderent
*1 Care dintre următoarele afirmaţii referitoare
D. Pete pigmentare la hepatita virală tip A este adevărată ?
E. Fond alb al leziunii
A. este determinată de un virus ADN
(pag. 427)
B. calea de transmitere este parenterală
7 Diagnosticul diferențial în caz de hemoragie C. incubaţia este cuprinsă între 2-6 luni
digestivă superioară include
D. infecţia este cel mai adesea simptomatică şi
A. Hemopatii maligne cronicizează frecvent la imunodeprimaţi
B. Hemoptizia E. diagnosticul se stabileşte pe baza evidenţierii
C. Sângerarea de origine ORL IgM anti-VHA în faza acută, IgG anti VHA izolate
semnificând un contact vechi
D. Vărsăturile cu conținut de culoare roșie (vin,
(pag. 451)
sfeclă roșie)
E. Anevrism de arteră mezenterică
(pag. 426)

Pag. 40 din 130


*2 Referitor la hepatita cronică cu VHB, în care *5 Care dintre suferinţele următoare este cauză
dintre situaţiile următoare se indică frecventă a unui sindrom de citoliză hepatică
tratament antiviral? mai mare de 10 ori valoarea normală?
A. ADN VHB ≥ 2000 UI/ml, şi/sau creştere A. boala Wilson, hepatita autoimună
persistentă a ALAT cu viremie detectabilă şi scor B. sindromul Budd-Chiari, infiltraţie tumorală
Metavir ≥ A2 şi/sau F2 hepatică
B. ADN VHB ≥ 20000 UI/ml, şi/sau ALAT ≥ 10 N şi C. ciroza hepatică, carcinomul hepatocelular
scor Metavir ≥ A1 şi/sau F1
D. hepatitele acute virale A, B, C Delta, E, CMV,
C. ADN VHB detectabil şi/sau ALAT ≥ 10 N şi scor
EBV, virusuri din grupul Herpes, hepatita
Metavir ≥ A1 şi/sau F1
medicamentoasă sau toxică, litiaza veziculară cu
D. ADN VHB detectabil indiferent de valoarea ALAT migrare de calcul, ischemia hepatică
şi scorului Metavir
E. pancreatita acută, ulcerul gastroduodenal
E. ADN VHB > 20000 UI/ml indiferent de valoarea (pag. 456-457)
ALAT şi scorului Metavir
(pag. 452) 6 Care dintre următoarele date clinice,
biologice si evolutive caracterizează o
*3 Care dintre următoarele afirmaţii referitoare hepatită virală acută formă fulminantă?
la tratamentul antiviral în hepatita cronică tip
C este adevărată? A. apariţia unei encefalopatii hepatice la mai puţin
de 2 săptămâni de la apariţia icterului
A. este contraindicat în cazurile de ciroză
B. scăderea indicelului de protrombină
compensată
(frecvent<25%)
B. este indicat în principal în cazurile cu scor
C. mortalitate ridicată (>50% în absenţa
Metavir F2, F3 sau F4, independent de activitate
tratamentului prin insuficienţă hepatică sau
C. este indicat la pacienţii cu manifestări complicaţiile acesteia) impunând internarea în
extrahepatice VHC sau cu genotip 2 sau 3 doar secţii de terapie intensivă
în prezenţa fibrozei semnificative
D. creşterea indicelui de protrombină
D. se bazează pe monoterapie cu interferon (frecvent>75%)
alfapegilat
E. apariţia unor manifestări extrahepatice:
E. scăderea încărcăturii virale cu 1 log în poliradiculonevrită, anemie hemolitică
săptămâna 12 de tratament permite continuarea autoimună, glomerulopatie
tratamentului pe durata prevăzută iniţial (pag. 451)
(pag. 453)
7 Care dintre următoarele afirmaţii referitoare
*4 Care dintre suferinţele următoare este cauză la hepatita virală tip B sunt adevărate?
frecventă a unui sindrom de citoliză hepatică
A. etiologia este reprezentată de un virus ARN,
mai mică de 10 ori valoarea normală?
transmis pe cale sexuală, parenterală, verticală
A. hepatita autoimună, boala Wilson, sindromul sau orizontală
Budd-Chiari B. incubaţia este cuprinsă între 6-12 săptămâni
B. ficatul cardiac, boala celiacă, disfuncţia tiroidiană
C. diagnosticul de hepatită acută B se bazează pe
C. firoză, hepatite virale cronice B şi C, hepatită evidenţierea Ag HBs şi Ig M anti HBc
medicamentoasă, hemocromatoză, hepatopatie D. cronicizarea este frecvent întâlnită la adult
alcoolică, steatohepatită nonalcoolică
E. istoria naturală a hepatitei cronice B prezintă 3
D. carcinomul hepatocelular
faze: imunotoleranţa, imunoeliminarea şi faza
E. parazitoze, amiloidoză, tuberculoză non replicativă
(pag. 455-456) (pag. 451, 542)

Pag. 41 din 130


8 Care dintre următoarele afirmaţii referitoare Tema nr. 35
la hepatita virală de tip C sunt adevărate? Ulcerul gastric şi duodenal
A. etiologia este reprezentată de un virus ARN, cu Bibliografie asociata temei:
transmitere majoritar parenterală şi foarte rar
sexuală Book Des ECN, editia in limba romana, redactor
Laurent Karila, 2011
B. incubaţia este cuprinsă între 4-6 săptămâni pag. 433-435
C. infecţia acută este frecvent simptomatică şi
cronicizează rar *1 Ulcerul gastric
D. in caz de serologie pozitivă pentru VHC se A. presupune o pierdere de substanţă a peretelui
efectuează PCR VHC calitativ gastric cu afectarea musculoasei
E. in cazul în care PCR VHC pozitiv este necesară B. este similar eroziunilor şi exulceraţiilor peretelui
efectuarea unui bilanţ complet gastric
(pag. 453) C. este mai frecvent decât cel duodenal
9 Care dintre următoarele afirmaţii referitoare D. are o incidenţă în creştere în ultimii 20 de ani
la hepatita delta sunt adevărate? E. este o urgenţă chirurgicală
A. este cauzată de un virus ARN defectiv care (pag. 433)
necesită prezenţa VHB pentru a se multiplica
*2 După tripla terapie a ulcerulului gastric
B. se poate prezenta ca suprainfecţie sau coinfecţie Helicobacter pylori pozitiv, un control
a VHB
endoscopic este indispensabil după
C. virusul cauzal se transmite pe cale enterală
A. o săptămână
D. infecţia acută este frecvent simptomatică, nu
cronicizează şi nu influenţează istoria naturală a B. 4 săptămâni
infecţiei cu VHB C. 6 săptămâni
E. diagnosticul se bazează pe detectarea Ac anti D. 2 săptămâni
Delta în serul pacienţilor Ag HBs pozitivi.
E. 8 săptămâni
(pag. 454)
(pag. 434)
10 Care dintre următoarele afirmaţii referitoare
*3 Tratamentul de eradicare al Helicobacter
la hepatita de tip E sunt adevărate?
pylori presupune
A. este determinată de VHE, virus ARN cu
A. Inhibitori de pompă de protoni + amoxicilină +
transmitere parenterală
claritromicină – 7-10 zile
B. incubaţia este cuprinsă între 2-6 săptămâni
B. Inhibitori de pompă de protoni + amoxicilină +
C. infecţia acută este frecvent asimptomatică, dar claritromicină – 4-8 săptămâni
deseori mai gravă la vârstnici, femei însărcinate
C. Inhibitori de pompă de protoni + amoxicilină - 14
şi persoane cu hepatopatie cronică
zile
D. diagnosticul se bazează pe detectarea Ac anti
D. Inhibitori de pompă de protoni + claritromicină –
VHE în sânge şi PCR ARN VHE în sânge sau
14 zile
materii fecale
E. prescrierea unui tratament cu IPP în doză
E. cronicizarea a fost observată doar la persoanele
adaptată pacientului
imunodeprimate
(pag. 434)
(pag. 454)
*4 Precizaţi care dintre următoarele afirmaţii
referitoare la ulcerul gastro-duodenal este
adevărată
A. controlul cicatrizării este obligatoriu în ulcerul
duodenal necomplicat
B. controlul endoscopic cu biopsie este obligatoriu
în ulcerul gastric
C. biopsia endoscopică este obligatorie în orice tip
de ulcer pentru a exclude un cancer
D. tratamentul chirurgical este preferat celui cu
inhibitori de pompă de protoni
E. sunt preferate tratamentele cu AINS celor cu
inhibitori selectivi Cox-2
(pag. 434)

Pag. 42 din 130


*5 Perforaţia ulcerului 10 Complicaţiile ulcerului duodenal pot fi
A. este o urgenţă chirurgicală A. hemoragia
B. evoluează cu durere cronică în epigastru B. ştrangularea
C. tratamentul constă în administrarea de inhibitori C. perforaţia
de pompă de protoni în doză mare D. stenoza
D. intervenţia chirurgicală este rezervată cazurilor
E. malignizarea
care nu răspund la tratament medical
(pag. 435)
E. evoluează obligatoriu cu hematemeză şi melenă
(pag. 434) Tema nr. 36
Icterul
6 Factorii etiologici principali ai ulcerului sunt
Bibliografie asociata temei:
A. infecţia cu Helciobacter pylori
Book Des ECN, editia in limba romana, redactor
B. consumul de antibiotice Laurent Karila, 2011
C. consumul de aspirină pag. 445-446
D. consumul de AINS
*1 Colestaza extrahepatică se caracterizează
E. ingestia de substanţe caustice prin
(pag. 433) A. obstrucţia căilor biliare intrahepatice cu cale
biliară principală normală
7 Durerea ulceroasă tipică
B. obstrucţie a căii biliare principale
A. apare la aproximativ 1/3 dintre pacienţii cu ulcer
C. distrugerea canaliculelor biliare
B. are sediu epigastric
D. disfuncţie hepatocitară
C. este ritmată de mese
E. urină deschisă la culoare
D. apare în hipogastru
(pag. 445)
E. nu are legătură cu ingestia de alimente
(pag. 433) *2 Colangiocarcinomul
A. este o cauză de icter cu bilirubină neconjugată
8 Următoarele afirmaţii referitoare la rolul
endoscopiei digestive superioare în ulcerul B. este o tumoră benignă
gastric şi duodenal sunt adevărate C. poate produce icter cu bilirubină conjugată
A. permite vizualizarea directă a Helicobacter pylori D. este o tumoră cu origine în epiteliul veziculei
B. este examenul cheie pentru diagnostic biliare
C. precizează sediului ulcerului E. este cea mai frecventă formă de cancer hepatic
primitiv
D. se poate efectua în urgenţă la pacienţii cu ulcer
hemoragic instabili hemodinamic (pag. 446)
E. permite realizarea biopsiilor *3 Dintre examenele paraclinice de primă
(pag. 434) intenţie în icterul cu bilirubină conjugată fac
parte
9 Tratamentul ulcerului gastro-duodenal
A. tomografia computerizată
presupune
B. RMN
A. administrarea de cefalosporinde de generaţia a
II-a C. colangio-RMN
B. eradicarea H. Pylori dacă acesta este prezent D. ecoendoscopia
C. prescrierea unui tratament cu inhibitori de pompă E. ecografia abdominală
de protoni (pag. 446)
D. întreruperea administrării medicamentelor
gastrotoxice *4 În icterul cu bilirubină conjugată

E. administrarea AINS în doză de siguranţă pentru A. urina are culoare normală


ameliorarea durerii B. tomografia computerizată este o explorare de
(pag. 434) primă intenţie
C. scaunele sunt decolorate
D. scaunele au culoare normală
E. are loc o hemoliză intensă
(pag. 445)

Pag. 43 din 130


*5 Icterul manifest presupune creşterea 10 Dintre cauzele benigne ale icterului cu
nivelului plasmatic al bilirubinei mai mult de bilirubină conjugată fac parte
A. 5 µg/l A. litiaza biliară
B. 20 µg/l B. pancreatita cronică
C. 30 µg/l C. colangita sclerozantă primitivă
D. 25 µg/l D. hemoliza cronică
E. 50 µg/l E. hemoliza acută
(pag. 445) (pag. 446)

6 Icterul poate apărea în următoarele condiţii Tema nr. 37


A. creşterea producţiei de bilirubină de origine Ciroza hepatică şi complicaţiile cirozei
hematologică Bibliografie asociata temei:
B. deficit de eliminare a bilei Book Des ECN, editia in limba romana, redactor
C. stenoză pilorică Laurent Karila, 2011
pag. 459-466
D. infarct miocardic acut
E. enterocolită acută *1 Ciroza hepatică se caracterizează din punct
(pag. 445) de vedere histologic prin
A. Afectare parcelară a parenchimului hepatic
7 Icterul cu bilirubină neconjugată se
caracterizează prin B. Prezenţa hiperplaziei nodulare focale
A. distensia veziculei biliare C. Păstrarea arhitecturii normale a ficatului
B. urină deschisă la culoare D. Steatoză microveziculară
C. scaune normal colorate E. Fibroză mutilantă
D. creşterea enzimelor de colestază (pag. 459)
E. dilataţie de căi biliare intrahepatice *2 Între cauzele cele mai frecvente de ciroză
(pag. 445) hepatică se numară
A. Hepatita autoimună
8 Următoarele afirmaţii despre sindromul
Gilbert sunt adevărate B. Ciroza biliară primitivă
A. se transmite autozomal dominant C. Hepatita cronică virală
B. valoarea hemoglobinei este normală D. Tromboza venelor suprahepatice
C. nu necesită tratament medicamentos E. Ciroza biliară secundară
D. bilirubina neconjugată este crescută moderat (pag. 459)
E. transaminazele hepatice au valori crescute *3 Endoscopia digestivă superioară poate
(pag. 445) evidenţia în ciroza hepatică următoarele
semne
9 Principalele cauze maligne ale icterului cu
A. Dilatarea venei porte
bilirubină conjugată de origine extrahepatică
sunt B. Splenomegalie
A. cancer pancreatic cefalic C. Tromboză portală
B. ampulom vaterian D. Varice esofagiene
C. colangiocarcinom E. Inversarea fluxului portal
D. litiază biliară (pag. 460)
E. hepatită acută *4 Precizaţi care dintre următoarele semne
(pag. 446) clinice se datorează hipertensiunii portale
din ciroza hepatică
A. Angioame stelare
B. Hipocratism digital
C. Icter
D. Eritroză palmară
E. Splenomegalie
(pag. 460)
Pag. 44 din 130
*5 Scorul Child-Pugh 9 Precizaţi care dintre următoarele afirmaţii cu
privire la encefalopatia hepatică sunt
A. Se foloseşte pentru evaluarea gravităţii cirozei
adevărate
B. Gravitatea cirozei creşte invers proporţional cu
valoarea scorului A. În stadiul II apare asterixis şi sindrom confuzional

C. Endoscopia digestivă se foloseşte pentru B. Se impune excluderea tulburărilor neuropsihice


calcularea scorului de altă cauză
D. Diametrul venei porte este unul din parametrii C. Nu este necesară căutarea unui factor
utilizaţi pentru calcularea scorului declanşator
E. Varicele esofagiene mari cresc cu 3 puncte D. Coma apare în stadiul III
valoarea scorului E. Furosemidul şi propranololul au efecte benefice
(pag. 460-461) (pag. 464-465)
6 Complicaţiile cirozei hepatice sunt 10 Următoarele afirmaţii referitoare la
A. Hemoragia digestivă complicaţiile pleuropulmonare ale cirozei
hepatice sunt adevărate
B. Sindromul hepato-renal
A. Hidrotoracele este o acumulare de fluid pleural
C. Ascita neoplazică cu conţinut crescut de proteine
D. Sindromul hepato-pulmonar B. Hipoxemia este legată de prezenţa dilataţiilor
E. Infecţia spontană a lichidului de ascită vasculare pulmonare şi de shunturi
intrapulmonare
(pag. 462-466)
C. Tratamentul sindromului hepatopulmonar constă
7 Hemoragia digestivă din ciroza hepatică în oxigenoterapie şi transplant hepatic
A. Se produce cel mai frecvent prin erodarea arterei D. Hidrotoracele se localizează mai mult pe stânga
coronare gastrice
E. Platipneea reprezintă un element cheie de
B. Poate duce la decompensarea cirozei şi apariţia diagnostic al sindromului hepato-pulmonar
encefalopatiei (pag. 466)
C. Este o complicaţie rară
D. Necesită tratament diuretic Tema nr. 38
Boala Crohn şi rectocolita hemoragică
E. Endoscopia digestivă se face de urgenţă, în
primele 6 ore Bibliografie asociata temei:
(pag. 463-464) Book Des ECN, editia in limba romana, redactor
Laurent Karila, 2011
8 Ascita din ciroza hepatică are următoarele pag. 439-441 * fara date epidemiologice, fara
caracteristici denumiri comerciale (ex. Nexavar)
A. Fluidul de ascită are celularitate bogată şi
conţinut crescut în proteine *1 Rectocolita hemoragică se caracterizează
prin
B. Trebuie întotdeauna căutat un factor declanşator
A. Boală inflamatorie transmurală
C. Nu necesită puncţie exploratorie pentru
efectuarea bilanţului biochimic, citologic şi B. Afectează constant rectul
bacteriologic C. Afectează constant ileonul terminal
D. Se complică cu infecţia spontană a lichidului de D. Leziunile sunt discontinue
ascită
E. Stenoze şi fistule
E. Tratamentul presupune regim hiposodat,
(pag. 439)
administrarea de diuretice şi puncţie evacuatorie
(pag. 462-463) *2 Leziunea caracteristică histopatologică
pentru boala Crohn este
A. Infiltratul limfocitar
B. Infiltratul cu PMN
C. Necroza cazeoasă
D. Granulomul epiteloid și gigantocelular, fără
necroză cazeoasă
E. Fibroza intestinală
(pag. 440)

Pag. 45 din 130


*3 Simptomatologia în boala Crohn este 8 Diganosticul de colită acută gravă se
dominată de bazează pe prezența următoarelor criterii
A. Rectoragii abundente A. Număr de evacuări/24 ore > 6, cu sânge
B. Scaune diareice cu glere și sânge la majoritatea B. Hemoglobină peste 12 g%
pacienților C. Temperatură vesperală sub 37,5⁰C
C. Diareea acută
D. Frecvenţă cardiacă sub 90/min
D. Prurit
E. Ulcerații extinse, cu dezlipire și punți mucoase
E. Diaree cronică cu evoluție progresivă asociată cu (pag. 441)
dureri abdominale
(pag. 440) 9 Explorările morfologice efectuate în boală
Crohn, cu scopul de a căuta o afecţiune a
*4 Care dintre următoarele aspecte pledează intestinului subţire, dacă aceasta nu a fost
pentru rectocolita hemoragică în evidenţiată de colonoscopie, sunt
diagnosticul diferențial cu boala Crohn
A. Examenul baritat esogastroduodenal
A. Leziuni mucoase discontinue cu limita
B. Radiografie abdominală simplă
superioară imprecisă
C. Irigografia
B. Prezența de stenoze
D. Enterografia CT
C. Prezența de fistule
E. Enterografia-RMN
D. Leziuni continue omogene, fără interval de
mucoasă sănătoasă (pag. 440)
E. Ileon terminal cu mucoasa granulară și ulcerații 10 Localizările cele mai frecvente ale bolii
(pag. 439) Crohn, în practică, sunt

*5 Precizaţi care dintre următoarele A. Esofagul


manifestările extradigestive asociate cu B. Duodenul distal
bolile inflamatorii cronice intestinale (BICI)
C. Ileonul
evoluează independent de pusee
D. Colonul
A. Uveite
E. Anusul
B. Aftoza genitală
(pag. 440)
C. Eritem nodos
D. Colangita sclerozantă primitivă Tema nr. 39
E. Aftoza bucală Patologia hemoroidală
(pag. 441) Bibliografie asociata temei:
Book Des ECN, editia in limba romana, redactor
6 Pe termen lung, boala Crohn se poate Laurent Karila, 2011
complica cu pag. 473-474
A. Denutriţie
*1 Hemoroizii
B. Cancer colorectal
A. Sunt formaţiuni vasculare situate la nivelul
C. Tromboză venoasă profundă
colonului descendent
D. Amiloidoză AA
B. Tulburările de tranzit intestinal nu reprezintă un
E. Perforaţie factor favorizant
(pag. 440-441) C. Sunt mai frecvenţi la persoanele între 45 si 65 ani
7 Manifestările extradigestive asociate cu D. Antecedentele familiale sunt obligatorii pentru
bolile inflamatorii cronice intestinale (BICI), apariţia acestora
care evoluează în paralel cu puseele, cuprind E. Hemoroizii externi sunt situaţi deasupra liniei
pectinee
A. Uveite
(pag. 473)
B. Aftoza genitală
C. Eritem nodos
D. Pelvispondilita reumatismală
E. Colangita sclerozantă primitivă
(pag. 441)

Pag. 46 din 130


*2 Următoarele afirmaţii privind fiziopatologia 7 Tratamentul instrumental în patologia
hemoroizilor sunt adevărate hemoroidală internă constă în
A. Este pe deplin elucidată A. Scleroza hemoroizilor
B. Nu se datorează alterării ţesutului conjunctiv B. Hemoroidopexie
care ancorează hemoroizii de sfincterul extern C. Fotocoagulare cu infraroşii
C. Factorul vascular nu este implicat în apariţia
D. Ligatura elastică a hemoroizilor
hemoroizilor
E. Ligatura chirugicală
D. Poate fi implicată perturbarea întoarcerii venoase
(pag. 474)
E. Hipovascularizaţia arterială se întâlneşte frecvent
(pag. 473) 8 Următoarele afirmaţii privind patologia
hemoroidală sunt adevărate
*3 Tratamentul medicamentos de primă intenţie
a trombozei hemoroidale externe presupune A. Tromboza hemoroidală internă este rară
B. Tuşeul rectal poate evidenţia noduli fermi şi
A. Antispastice
dureroşi care corespund unui tromb la nivelul
B. Antalgice/AINS hemoroizilor interni
C. Probiotice C. Tuşeul rectal şi anuscopia trebuie evitate în
D. Cărbune medicinal urgenţă pentru că sunt inutile şi dureroase

E. Simeticonă D. Ligaturarea elastică se face numai în spital

(pag. 474) E. Ligaturarea elastică este mai riscantă decât


sclerozarea
*4 Stadializarea Goligher a hemoroizilor (pag. 474)
A. Se aplică hemoroizilor externi
9 Tratamentul chirurgical al hemoroizilor
B. În stadiul II se întalnesc hemoroizi interni fără constă în
prolaps
A. Hemoroidectomie
C. Prolapsul permanent şi ireductibil apare în
stadiul I B. Îndepărtarea hemoroizilor patologici grupaţi de
obicei în 3 pachete
D. Împarte hemoroizii în 5 stadii
C. Sutura plăgilor după hemoroidectomie
E. În stadiul III apare prolaps reductibil manual
D. Hemoroidopexie
(pag. 473-474)
E. Scleroterapie
*5 Următoarea afirmaţie privind hemoroizii (pag. 474)
interni este adevărată
10 Regulile igienicodietetice şi tratamentul
A. Se complică frecvent cu tromboză
medical în patologia hemoroidală include
B. Apar sângerări tipic nedureroase, cu sânge roşu,
în timpul sau imediat după defecaţie A. Creşterea raţiei zilnice de fibre alimentare

C. Regimul igienodietetic presupune reducerea B. Administrarea de laxative


raţiei zilnice de fibre alimentare C. Utilizarea de antispastice
D. Diagnosticul este exclusiv paraclinic D. Limitarea eforturilor excesive de defecaţie
E. Anuscopia se efectuează de urgenţă în caz de E. Utilizarea de medicamente topice
prolaps hemoroidal trombozat (pag. 474)
(pag. 473-474)

6 Despre tratamentul trombozei hemoroidale


sunt adevărate următoarele afirmaţii
A. Este medicamentos de primă intenţie
B. Intervenţia chirurgicală se efectuează în caz de
ineficienţă a tratamentului medicamentos
C. Are ca obiectiv reglarea tranzitului intestinal
D. Venotonicele au eficacitate certă
E. Nu se administreaza antialgicele şi AINS
(pag. 474)

Pag. 47 din 130


Tema nr. 40 *5 În caz de tumoră esofagiană metastatică cu
Patologia tumorală a tubului digestiv: tumorile deglutiţie prezentă este indicat:
colonului şi rectului, tumorile stomacului, tumorile A. esofagectomie
primitive şi secundare ale ficatului, tumorile
B. radiochimioterapie exclusivă
esofagului, tumorile pancreasului
Bibliografie asociata temei: C. gastrostoma de alimentaţie
Book Des ECN, editia in limba romana, redactor D. numai chimioterapie
Laurent Karila, 2011 E. instalarea unei proteze
pag. 475-478, 479-480, 481-484, 485-487, 488-490 (pag. 487)
* fara date epidemiologice
6 Factorii de risc în adenocarcinomul de
*1 În adenocarcinomul de colon şi rect, forma pancreas sunt:
necomplicată, una din trăsăturile clinice este
A. pancreatita cronică
reprezentată de:
B. chistele pancreatice
A. febra
C. diabetul
B. tulburări de tranzit/modificări de tranzit
D. adenomul pancreatic
C. apărare musculară abdominală
E. fumatul
D. distensia abdomenului
(pag. 488)
E. vărsături
(pag. 475) 7 Elementele de diagnostic clinic în cancerul
de esofag sunt:
*2 În tumorile de colon şi rect complicate
A. disfagia
ocluzia este cel mai frecvent întâlnită în:
B. disfonia
A. cancerul de cec
C. alterarea stării generale
B. cancerul de colon transvers
D. refluxul gastroesofagian: pirozisul
C. cancerul de colon stâng
E. eructaţiile
D. cancerul de rect
(pag. 486)
E. cancerul de unghi hepatic al colonului
(pag. 476) 8 Tratamentul curativ în carcinomul
hepatocelular constă în:
*3 În cancerul antral rezecabil intervenţia
A. chimioembolizare
chirurgicală indicată este:
B. Sorafenib
A. gastroenteroanastomoza
C. Transplantul hepatic
B. gastrostomie de alimentaţie
D. Rezecţie chirurgicală
C. gastrectomie totală
E. Distrugere locală (radiofrecvenţă, crioterapie,
D. gastrectomie 4/5 şi anastomoză gastrojejunală,
alcoolizare)
limfadenectomie
(pag. 483)
E. laparotomie exploratorie + biopsie tumorală +
biopsie ganglionară 9 Factorii de risc în cancerul gastric sunt:
(pag. 480) A. refluxul gastroesofagian
*4 Care din următoarele afirmaţii referitoare la B. infecţia cu HP
chistul biliar este adevarată: C. antecedentele familiale de cancer gastric
A. α-feto-proteina creşte peste 500 ng/ml D. ulcerul gastric
B. apare pe fondul unui ficat cirotic E. tumorile gastrice benigne
C. este indicată excizia chirurgicală (pag. 479)
D. este indicată distrucţia locală prin radiofrecvenţă
E. abţinere terapeutică, fără monitorizare
(pag. 482)

Pag. 48 din 130


10 Elementele clinice ale extensiei tumorale în 15 În cancerul de colon stâng ocluziv
cancerul gastric sunt: tratamentul constă în:
A. masă abdominală epigastrică A. rezolvarea de urgenţă a ocluziei (stomie sau
B. hepatomegalie endoproteză)
B. colectomia carcinologică în primul timp
C. carcinomatoza depistată la tuşeul rectal
C. reechilibrarea şi stabilizarea funcţiilor vitale
D. ganglion Troisier
D. chimioterapie paleativă
E. paloarea tegumentelor, inapetenţă, alterarea
stării generale E. hemicolectomie carcinologică stângă în al doilea
(pag. 480) timp cu anastomoză colorectală
(pag. 478)
11 Formele complicate ale cancerului de colon şi
rect sunt: 16 În cancerul de rect superior tratamentul
chirurgical constă în:
A. ocluzia intestinală
A. amputaţie de rect abdomino-perineală
B. hemoragiile oculte
B. colectomie stângă şi rezecţie parţială a rectului
C. tulburări de tranzit intestinal
(5 cm sub tumoră)
D. infecţia
C. limfadenectomie cu ligatura vaselor mezenterice
E. metastazele pulmonare inferioare
(pag. 476) D. excizia parţială a mezorectului
12 Confirmarea diagnostică în cancerul E. restabilirea continuităţii printr-o anastomoză
colorectal se face prin: colorectală joasă
(pag. 478)
A. depistarea hemoragiilor oculte
B. masă tumorală abdominală şi tulburări de tranzit * 17 Icterul într-un adenocarcinom de pancreas:
intestinal
A. are evoluţie oscilantă
C. ganglion Troisier
B. este progresiv
D. colonoscopie completă + biopsie + examen
C. este însoţit de durere, febră, frison
histologic
D. nu este însoţit de prurit
E. coloscaner/clismă cu substanţe de contrast
hidrosolubile în caz de colonoscopie incompletă E. este însoţit de scaune şi urini normal colorate
(pag. 475-476) (pag. 489)

13 În cancerul de colon drept tratamentul * 18 În caz de tumoră rezecabilă de cap de


chirurgical constă în: pancreas este indicată urmatoarea
intervenţie:
A. colostomie cutanată
A. duodenopancreatectomie cefalică
B. hemicolectomie dreaptă
B. splenopancreactectomie corporeocaudală
C. limfadenectomie pe vasele colice drepte şi la
originea lor C. pancreatectomie totală
D. restabilirea continuităţii printr-o anastomoză D. dubla derivaţie paleativă
ileocolică E. laparotomie exploratorie + biopsie ganglionară +
E. aplicarea de endoproteză biopsie pancreatică
(pag. 477) (pag. 489)

14 În cancerul de colon stâng non-ocluziv


tratamentul chirurgical constă în:
A. hemicolectomie stângă
B. colostomie
C. aplicare de endoproteză
D. limfadenectomie cu ligatura vaselor mezenterice
inferioare
E. restabilirea continuităţii printr-o anastomoză
colorectală
(pag. 477)

Pag. 49 din 130


Tema nr. 41 *5 Specificaţi care din următoarele metode de
Evaluarea severităţii şi identificarea complicaţiilor tratament reprezintă conduita corectă în
precoce la pacientul cu traumatism abdominal cazul plăgilor abdominale penetrante:

Bibliografie asociata temei: A. laparotomie, explorarea întregului tub digestiv şi


tratamentul leziunilor
Book Des ECN, editia in limba romana, redactor
Laurent Karila, 2011 B. pansamentul plăgii asociat cu antibioterapia
pag. 491-493 C. vaccinarea împotriva infecţiilor cu pneumococ
D. administrarea de analgezice şi supravegherea
pag. 491-493
plăgii
*1 Managementul unei contuzii abdominale la E. stimularea tranzitului intestinal
un pacient instabil hemodinamic obligă la: (pag. 493)
A. laparotomie de urgenţă
6 În cazul contuziilor abdominale, cele mai
B. administrarea de analgezice afectate organe sunt:
C. încălzirea pacientului A. vezicula biliară
D. profilaxia tetanosului B. ficatul
E. monitorizarea pacientului în unitatea de terapie C. duodenul
intensivă
D. stomacul
(pag. 491)
E. splina
*2 Managementul unui traumatism splenic la un (pag. 491)
pacient stabil hemodinamic presupune:
7 Explorările imagistice (ecografia, CT) folosite
A. splenectomie de urgenţă
în cazul unui traumatism splenic pot
B. antibioterapie cu amoxicilină evidenţia:
C. vaccinarea împotriva infecţiilor cu pneumococ A. pneumoperitoneu
D. tratament medical şi monitorizare B. hemoperitoneu
E. imobilizarea fracturilor costale C. hemotorax
(pag. 491) D. hematom al splinei
*3 Care din urmatoarele complicaţii poate apare E. nivele hidroaerice multiple
consecutiv unui traumatism hepatic? (pag. 492)
A. ocluzia intestinală
8 Diagnosticul de traumatism pancreatic este
B. coleperitoneul susţinut de:
C. pancreatita acută A. durere de pancreatită acută
D. abcesul fundului de sac Douglas B. scăderea amilazelor
E. supuraţia parietală C. creşterea lipazei
(pag. 492) D. febră persistentă şi frison
*4 Care din următoarele afecţiuni poate apare E. creşterea calcemiei
ca şi complicaţie consecutivă a unui (pag. 493)
traumatism pancreatic?
9 Complicaţiile traumatismului hepatic sunt:
A. peritonită generalizată
B. hemoperitoneu masiv A. embolia gazoasă în caz de plagă a venelor
suprahepatice
C. abces interhepatodiafragmatic drept
B. biliom
D. pancreatită acută
C. coleperitoneu
E. pancreatită cronică
D. ocluzia intestinală
(pag. 493)
E. şocul toxico-septic
(pag. 492)

Pag. 50 din 130


10 Examenul clinic al unei plăgi abdominale *5 Spuneţi ce simptom sau semn clinic nu face
trebuie să identifice: parte din tabloul clinic al ocluziei intestinale
joase:
A. punctul de intrare şi punctul de ieşire
B. traiectoria plăgii A. Oprirea precoce a tranzitului pentru materii şi
gaze
C. debridarea şi sutura plăgii
B. Vărsături tardive şi de aspect fecaloid
D. profilaxia tetanosului
C. Lipsa meteorismului
E. dacă plaga este penetrantă
D. Durere abdominală
(pag. 493)
E. Distensia abdominală
Tema nr. 42 (pag. 494)
Sindromul ocluziv
6 Pentru a stabili etiologia unui sindrom
Bibliografie asociata temei: ocluziv, în cadrul anamnezei punem accent
Book Des ECN, editia in limba romana, redactor pe următoarele:
Laurent Karila, 2011
A. Antecedentele chirurgicale (eventuale operaţii
pag. 494-498 anterioare)
*1 Ileus-ul biliar reprezintă un tip de ocluzie B. Medicaţia anterioară
intestinală care are ca mecanism: C. Semnele de impregnare neoplazică
A. Strangularea D. Antecedentele personale fiziologice
B. Obstrucţia E. Anamneza nu are un rol important în stabilirea
etiologiei, doar investigaţiile paraclinice singure
C. Funcţional
pot stabili etiologia sindromului ocluziv
D. Reflex
(pag. 494)
E. Necunoscut
7 Caracteristice pentru sindromul ocluziv sunt
(pag. 495)
următoarele semne şi simptome:
*2 Ce semn clinic este caracteristic la inspecţia A. Durerea abdominală
bolnavului cu volvulus de sigmoid?
B. Meteorismul abdominal
A. Abdomen uşor destins, cu echimoză
C. Hematemeza
periombilicală (semnul Cullen)
D. Vărsăturile
B. Steluţele vasculare
E. Oprirea tranzitului pentru materii şi gaze
C. Icter sclero-tegumentar
(pag. 494)
D. Meteorismul asimetric
E. Abdomen plat, imobil cu respiraţia 8 Următoarele patologii pot determina ocluzie
intestinală prin mecanism de obstrucţie:
(pag. 497)
A. Tumori intestinale
*3 Ce imagine radiologică este patognomonică
pentru ocluziile intestinale? B. Polipi
C. Ileus biliar
A. Pneumoperitoneul subdiafragmatic
D. Volvulus sigmoidian
B. Pleurezia bazală dreaptă
E. Ileus postoperator
C. Multiplele imagini de calculi radio-opaci
(pag. 495)
D. Nivelele hidro-aerice
E. Nu exista o imagine radiologică patognomonică 9 Tratamentul volvulusului de sigmoid poate fi
pentru ocluziile intestinale reprezentat de:
(pag. 495) A. Reducerea volvulusului cu o sonda Faucher +/-
colonoscopie
*4 Sindromul Ogilvie se poate intâlni în
următoarele situaţii, cu excepţia uneia: B. Scurtă sigmoidectomie cu anastomoză colo-
rectală
A. Context chirurgical postoperator
C. Rezecţia colonului volvulat urmat de stomie
B. În reanimare (intervenţia Hartmann)
C. Bolnavul cu hipopotasemie D. Doar tratament medical
D. Insuficienţa cardiacă E. Nu este nevoie de vreun tratament – sigmoidul
E. Hipertensiunea arterială se devolvulează spontan
(pag. 498) (pag. 497)

Pag. 51 din 130


10 Care dintre următoarele sunt considerate *4 În apendicita acută, explorările biologice
semne de severitate la un sindrom ocluziv? semnificative sunt reprezentate de:
A. Nivele hidro-aerice A. sindromul de citoliză hepatică
B. Sepsis sever B. sindromul inflamator
C. Apărarea abdominală C. sindromul de activitate imunologică
D. Meteorismul abdominal D. sindromul bilio-excretor
E. Pneumoperitoneul la radiografia abdominală E. sindromul de retenţie azotată
(pag. 494-495) (pag. 499)

Tema nr. 43 *5 Tratamentul etiologic în apendicita acută


Apendicita la copii şi adulţi este reprezentat de:

Bibliografie asociata temei: A. analgezice şi reechilibrare hidroelectrolitică


Book Des ECN, editia in limba romana, redactor B. antibioterapie intravenos 24-48 ore
Laurent Karila, 2011 C. apendicectomie
pag. 499-500
D. vaccinare profilactică
*1 Tratamentul plastronului apendicular E. regim alimentar absolut şi repaus la pat
presupune: (pag. 500)
A. apendicectomie de urgenţă
6 Diagnosticul clinic de apendicită acută este
B. apendicectomie de urgenţă şi administrarea de susţinut de:
antibiotice cu spectru larg
A. durere în fosa iliacă dreaptă
C. tratament medical iniţial şi apendicectomie la
distanţă B. vărsături, greţuri
D. tratament medical profilactic C. disurie, polakiurie
E. apendicectomie laparoscopică D. febră 37,5-38,5°C
(pag. 500) E. tuşeu pelvin dureros
(pag. 499)
*2 În cazul apendicitei mezoceliace, tabloul
clinic este cel de: 7 Complicaţiile apendicitei acute sunt:
A. peritonită generalizată A. plastronul apendicular
B. infecţie urinară B. peritonita generalizată
C. ocluzie febrilă C. abces apendicular
D. torsiune ovariană D. degenerarea malignă
E. colică renală E. hemoragie digestivă inferioară
(pag. 500) (pag. 500)
*3 În apendicita acută, infecţia apendicelui se 8 Diagnosticul diferenţial al apendicitei acute
produce cel mai adesea: se face cu:
A. pe cale endogenă, ca urmare a unei obstrucţii a A. invaginaţia intestinală acută
apendicelui
B. limfadenita mezenterică
B. pe cale hematogenă, de la un focar infecţios
C. orhiepididimita acută
aflat la distanţă
D. torsiunea ovariană
C. pe cale respiratorie, în cazul epidemiilorpe cale
respiratorie, în cazul epidemiilor E. colica renală dreaptă
D. pe cale limfatică, de la o infecţie de vecinătate (pag. 500)
E. pe cale digestivă, de la o toxiinfecţie alimentară 9 Tratamentul etiologic al apendicitei acute
(pag. 499) constă în:
A. apendicectomie
B. prelevarea de lichid pentru examen bacteriologic
C. omentectomie parţială
D. salpingectomie dreaptă
E. lavajul cavităţii abdominale
(pag. 500)
Pag. 52 din 130
10 Tratamentul peritonitei apendiculare constă *3 Care dintre următoarele afirmaţii legate de
în: arcada crurală (ligamentul inghinal) este
adevărată?
A. lavaj peritoneal abundent
B. profilaxia tetanosului A. Bandă fibroasă întinsă între spina iliacă antero-
superioară şi pubis (tuberculul pubian)
C. apendicectomie
B. Bandă fibroasă întinsă între spina iliacă antero-
D. antibioterapie timp de cel puţin 5 zile inferioară şi tuberculul pubian
E. repaus la pat timp de cel puţin 5 zile C. Bandă fibroasă întinsă între spina iliacă postero-
(pag. 500) superioară şi pube
D. Bandă fibroasă întinsă între creasta iliacă şi pube
Tema nr. 44
E. Bandă fibroasă întinsă între spina iliacă antero-
Hernia parietală la copil şi adult superioară şi ischion
Bibliografie asociata temei: (pag. 504)
Book Des ECN, editia in limba romana, redactor
Laurent Karila, 2011 *4 Herniile inghinale sunt acele hernii:
pag. 504-505 A. Al căror orificiu este situat mai sus de arcada
crurală
*1 Care dintre următoarele afirmaţii defineşte o
B. Al căror orificiu este situat sub arcada crurală
hernie?
C. Al căror orificiu este medial de vasele femurale
A. Corespunde unei exteriorizări a conţinutului
abdominal printr-un orificiu ne-natural D. Al căror orificiu este la nivelul zonei ombilicale
B. Corespunde unei exteriorizări a conţinutului E. Al căror orificiu este situat pe linia albă
abdominal printr-un orificiu din vecinătatea (pag. 504)
cicatricei unei laparotomii
C. Corespunde unei exteriorizări a conţinutului *5 Care dintre următoarele afirmaţii legate de
abdominal printr-un orificiu natural al peretelui reparaţia parietală într-o hernie inghinală
este adevărată?
D. Se distinge de eventraţii prin apariţia la scurt
timp scurt după intervenţia chirurgicală A. Sutura directă prin coborârea tendonului
conjunct la ligamentul Cooper
E. Se distinge de eventraţii prin absenţa peritoneului
din jurul viscerelor B. Poziţionarea unei plase nerezorbabile, fixată la
arcada crurală
(pag. 504)
C. Sutura simplă prin puncte separate pentru micile
*2 Care dintre următoarele afirmaţii legate de orificii
herniile femurale este adevărată?
D. Laparotomia sau laparoscopia
A. Sunt acele hernii al căror orificiu este situat mai E. Eliberarea sacului herniar şi reintegrarea
sus de arcada crurală conţinutului în cavitatea abdominală
B. Sunt acele hernii al căror orificiu este situat sub (pag. 505)
arcada crurală, medial de vasele femurale
C. Sunt acele hernii al căror orificiu se 6 Care dintre următoarele afirmaţii legate de
exteriorizează prin fascia transversalis herniile inghinale la copii sunt adevărate?
D. Sunt acele hernii al căror orificiu se A. Afectează mai ales băieţii
exteriorizează prin orificiul inghinal profund în B. Sunt congenitale
relaţie cu funiculul spermatic
C. De multe ori sunt chiar bilaterale
E. Sunt acele hernii al căror orificiu este situat la
nivelul inelului ombilical D. Sunt dobândite
(pag. 504) E. Uneori sunt asociate cu criptorhidia
(pag. 505)

7 Care dintre dintre următarele afirmaţii legate


de hernia inghinală necomplicată sunt false?
A. Masă inghinală nedureroasă, reductibilă
B. Masă inghinală, impulsivă la tuse, reductibilă
C. Poate fi situată în scrot, reductibilă
D. Hernia este dureroasă
E. Hernia este de volum mare, ireductibilă
(pag. 505)

Pag. 53 din 130


8 Principiile intervenţiei chirurgicale ale unei 2 Examenul ecografic in litiaza veziculară
hernii sunt: simptomatică necomplicată evidenţiază:
A. Laparotomia sau laparoscopia A. colecist mărit de volum
B. Eliberarea sacului herniar şi reintegrarea B. colecist cu perete ingroşat peste 2 mm
conţinutului în cavitatea peritoneală C. colecist cu perete neângroşat sub 2 mm
C. Reparaţia parietală
D. lichid pericolecistic
D. Enterectomia
E. calculi hiperecogeni cu con de umbră
E. Drenajul cavităţii peritoneale (pag. 509)
(pag. 505)
3 Colecistita acută litiazică:
9 În cazul în care conţinutul sacului de hernie
A. reprezintă inflamaţia peretelui colecistic
strangulată este reprezentat de intestin
subţire, simptomele digestive sunt: B. reprezintă inflamaţia căilor biliare extrahepatice
A. Durere abdominală majoră C. poate evolua spre constituirea piocolecistului
B. Ocluzie intestinală înaltă cu vărsături precoce D. poate evolua spre gangrena peretelui colecistului
C. Blocarea tranzitului pentru materii fecale şi gaze E. se poate rezorbi în totalitate prin tratament
în primă instanţă intensiv timp de 3 zile
D. Tablou clinic atenuat, în afara unui ileus reflex (pag. 510)
(paralitic)
4 Diagnosticul clinic de colecistită acută se
E. Constipaţie rebelă la tratamente cu purgative susţine pe următoarele simptome şi semne:
(pag. 505) A. durere epigastrică iradiată posterior şi vărsături
10 Care dintre următoarele afirmaţii legate de B. durere în hipocondrul drept cu iradiere în umărul
reparaţia parietală într-o hernie ombilicală drept şi apărare musculară
sunt false? C. prezenţa constantă a febrei
A. Sutură simplă prin puncte separate pentru micile D. prezenţa constantă a subicterului conjunctival
orificii
E. prezenţa meteorismului asimetric în hipocondrul
B. Poziţionarea unei plase nerezorbabile, în caz de drept sau flancul drept
orificiu mare
(pag. 510)
C. Poziţionarea unei plase nerezorbabile, fixată la
arcada crurală 5 Examenul ecografic în colecistita acută
litiazică poate evidenţia:
D. Sutura directă prin coborârea tendonului
conjunct la ligamentul Cooper A. dilataţia constantă a căilor biliare extrahepatice
E. Sutura tendonului conjunct la arcada crurală B. lichid liber perivezicular
(pag. 505) C. îngroşarea peste 4 mm a peretelui vezicular
D. perete vezicular în mare tensiune şi subţiat
Tema nr. 45
Litiaza biliară şi complicaţiile ei E. calculi în colecist
Bibliografie asociata temei: (pag. 510)
Book Des ECN, editia in limba romana, redactor *6 Colica biliară litiazică este durerea tipică în
Laurent Karila, 2011 litiaza veziculară, caracterizată prin:
pag. 508-512
A. acompanierea constantă de febră şi de icter
1 Factorii de risc în litiaza veziculară sunt: B. instalarea apărării musculare în hipocondrul drept
A. sexul masculin C. apariţia bruscă a icterului cu evoluţie progresivă
B. sedentarismul D. durere în hipocondrul drept cu iradiere în umărul
drept
C. stressul
E. durere în punctul cistic şi sindrom febril prelungit
D. regimul alimentar hipercaloric
(pag. 508)
E. dislipidemia
(pag. 508)

Pag. 54 din 130


*7 Ileusul biliar este: Tema nr. 46
A. consecinţa migrării spontane a calculilor din Panceatita acută si cronică
coledoc în duoden Bibliografie asociata temei:
B. o formă de ocluzie intestinală prin obstrucţie Book Des ECN, editia in limba romana, redactor
C. o pareză a intestinului subţire în contextul litiazei Laurent Karila, 2011
biliare simptomatice pag. 467-469, 513-516
D. un semn acompaniator al peritonitei biliare
*1 Care afirmaţie este falsă în cazul pancreatitei
E. o pareză a întregului intestin secundară unei cronice:
litiaze biliare asimptomatice
A. Cauza net dominantă este consumul de alcool
(pag. 511)
B. În evoluţia bolii se întâlnesc trei faze succesive
*8 Triada Charcot se caracterizează clinic prin C. Examenul clinic este sugestiv
următoarele elemente:
D. Pseudochistul poate apare ca şi complicaţie
A. durere biliară urmată de febră şi apoi de icter evolutivă
B. colică biliară, febră şi vărsături bilioase E. Ecografia abdominală este o metodă de
C. durere biliară, febră şi adenopatie diagnostic puţin sensibilă şi puţin specifică
supraclaviculară dreaptă (pag. 467, 468, 469)
D. durere în hipocondrul drept, icter şi prurit
tegumentar *2 În managementul terapeutic al pancreatitei
cronice intră:
E. colică biliară, hepatomegalie şi icter
A. Tratamentul disfuncţiei organice multiple
(pag. 512)
B. Colecistectomia laparoscopică de principiu
*9 Diagnosticul clinic de angiolangită se C. Anastomoza pancreatico-jejunală
susţine pe următoarele elemente clinice:
D. Splenectomia
A. colică biliară urmată de febră şi de instalarea
icterului E. Drenajul focarelor de necroză pancreatică şi
peripancreatică
B. ficat palpabil la rebord, icter şi vărsături
(pag. 469)
C. colecist palpabil, febră şi frisoane
D. durere în hipocondrul drept, icter şi scaune *3 Care afirmaţie este falsă cu privire la
decolorate pancreatita acută?

E. durere epigastrică, icter şi frisoane A. Durerea este localizată epigastric, transfixiantă,


iradiată spre posterior
(pag. 512)
B. Creşterea lipazemiei > 3N
* 10 Sfincterotomia endoscopică de urgenţă este C. Este prezent şocul septic în formele severe
indicată în:
D. Ecografia abdominală este cea mai sensibilă
A. colecistita acută gangrenoasă metodă imagistică de diagnostic
B. icterul mecanic însoţit de hepatomegalie E. Scorul Ranson şi Balthazar sunt scoruri de
C. angiolangita cu sepsis necontrolat gravitate
D. pancreatita acută de cauză biliară (pag. 513, 514, 515)
E. litiaza biliară de migrare *4 Tratamentul etiologic al pancreatitei acute
(pag. 512) constă în:
A. Pseudochisto-jejuno anastomoză
B. Drenajul necrozelor pancreatice şi
peripancreatice
C. Tratamentul disfuncţiei organice multiple
D. Colecistectomie de urgenţă în formele grave
E. ERCP în cazul persistenţei unui calcul la nivelul
căii biliare principale
(pag. 516)

Pag. 55 din 130


*5 Nu reprezintă cauze de pancreatită acută: 10 Care afirmaţii sunt adevărate cu privire la
tratamentul pancreatitei acute?
A. Litiaza biliară
B. Consumul de alcool A. Orice pacient cu pancreatită acută se internează

C. Traumatismele abdominale B. Formele grave de boală se tratează în secţiile de


terapie intensivă
D. Medicaţia antiinflamatorie nesteroidiană
C. La pacienţii ce prezintă şi angiocolită este
E. Hiperlipidemia indicată colecistectomia clasică de primă intenţie
(pag. 513) D. Orice formă de pancreatită acută necesită
intervenţie chirurgicală
6 Diagnosticul diferenţial al pancreatitei
cronice se face cu: E. Tratamentul antibiotic se face cu antibiotice cu
spectru larg de tipul Tienamului
A. Tumori pancreatice
(pag. 515)
B. Dilataţii chistice ale canalelor pancreatice
C. Angiocolita Tema nr. 47
Peritonita acută
D. Chistul hidatic hepatic
Bibliografie asociata temei:
E. Angorul mezenteric
Book Des ECN, editia in limba romana, redactor
(pag. 467)
Laurent Karila, 2011
7 Complicaţiile posibile în evoluţia pancreatitei pag. 517-518 * fara denumiri comerciale (ex.
cronice sunt: Tezocilin)
A. Diabetul insulino-necesitant *1 Peritonita corespunde:
B. Pseudochistul de pancreas A. Unei inflamaţii acute a peritoneului
C. Adenocarcinomul de pancreas B. Inflamaţiei acute a ţesutului gras retroperitoneal
D. Insuficienţă renală acută C. Torsiunii unui organ intraperitoneal
E. Compresiune duodenală D. Obstrucţiei acute a intestinului
(pag. 468)
E. Prezenţei sângelui în cavitatea peritoneală
8 Care afirmaţii sunt false cu privire la (pag. 517)
pancreatita cronică?
*2 Majoritatea peritonitelor sunt secundare:
A. Cauza cea mai frecventă este consumul
prelungit de alcool A. Infecţiei hematogene a lichidului de ascită
B. Poate apare ca urmare a unui obstacol la nivelul B. Tuberculozei abdominale
canalului pancreatic principal C. Perforaţiei de tub digestiv
C. Apare cel mai frecvent la pacienţii cu litiază biliară D. Extensiei unei leziuni pleurale de vecinătate
D. Ecoendoscopia este o metodă de diagnostic cu E. Ingestiei de substanţe caustice
sensibilitate redusă
(pag. 517)
E. Examenele biologice nu sunt relevante pentru
diagnosticul pozitiv *3 Care dintre următoarele afirmaţii este
(pag. 467, 469) adevărată în cazul peritonitei acute
secundare:
9 Care afirmaţii sunt corecte cu privire la
A. Infecţia este în majoritatea cazurilor
pancreatita acută?
monomicrobiană
A. Este o inflamaţie acută a pancreasului B. Bacilul Koch este responsabil de majoritatea
B. Cauzele cele mai frecvente sunt litiaza biliară şi infecţiilor peritoneale
consumul de alcool C. Agenţii infecţioşi sunt de cele mai multe ori
C. Apare frecvent în evoluţia tumorilor de cap de multiplii
pancreas D. Iritaţia peritoneală este indusă pe cale
D. Se poate complica cu insuficienţă multiorganică hematogenă
E. Nu este necesară internarea în formele uşoare E. Dializa peritoneală este principalul factor etiologic
de boală (pag. 517)
(pag. 513, 515)

Pag. 56 din 130


*4 Intervenţia Hartmann în peritonita prin 8 Tratamentul etiologic al peritonitei acute
cancer de colon stâng perforat constă în: constă în:
A. Sutura perforaţiei fără drenaj A. Laparotomie
B. Hemicolectomie stângă şi anastomoză B. Pleurostomie
colorectală termino-terminală C. Lavaj al cavităţii peritoneale cu 10-15 l de ser
C. Colectomie stângă, închiderea bontului rectal, fiziologic
colostomie terminală D. Tratamentul cauzei peritonitei
D. Colectomie stângă şi anastomoză colorectală
E. Corticoterapie
latero-terminală
(pag. 518)
E. Ileo-recto-anastomoză latero-laterală
(pag. 518) 9 Următoarele afirmaţii referitoare la peritonita
acută postoperatorie sunt adevărate:
*5 Următoarea afirmaţie referitoare la peritonita
acută secundară este adevărată: A. Uneori este dificil de diagnosticat
B. Poate fi determinată de dehiscenţa anastomotică
A. Principalul factor prognostic este intervalul de
timp scurs până la intervenţia chirurgicală C. CT nu este necesar, diagnosticul fiind adesea
evident
B. CT este obligatorie pentru stabilirea conduitei
terapeutice, indiferent de starea pacientului D. Este o formă de peritonită primitivă
C. Intervalul de timp debut – tratament chirurgical E. Va fi evocată de durerea toracică intensă însoţită
nu influenţează prognosticul de tuse şi dispnee
D. Tratamentul antibiotic singular este suficient în (pag. 518)
majoritatea peritonitelor secundare
10 Peritonita acută postoperatorie va fi evocată
E. Tratamentul chirurgical trebuie evitat în special în caz de:
(pag. 518) A. Febră
6 Următoarele circumstanţe etiologice B. Secreţii purulente prin cicatrice
corespund unei peritonite acute primitive:
C. Secreţii purulente pe tubul de dren
A. Tuberculoza abdominală D. Semn Babinski intens pozitiv
B. Perforaţia ulcerului duodenal E. Tulburări de cunoştinţă, agitaţie, insuficienţă
C. Dializa peritoneală renală acută, insuficienţă respiratorie acută
D. Peritonita cu fungi (pag. 518)
E. Infecţia ascitei
Tema nr. 48
(pag. 517) Sarcina normală si patologică: travaliul, naşterea şi
post-partumul normal, principalele complicaţii ale
7 Tabloul clinic al peritonitei acute
sarcinii, sarcina extrauterină
extrahospitaliere cuprinde:
Bibliografie asociata temei:
A. Durerea abdominală brutală, ce creşte tot mai
mult Book Des ECN, editia in limba romana, redactor
Laurent Karila, 2011
B. Contractura abdominală pag. 521-524, 525-532, 533-536, 549-551 * cu
C. Hemoptizia exceptia tabel 1 "concedii de materinitate" pag. 524
D. Oprirea tranzitului intestinal pentru materii fecale si paragraf 1 pag. 524, fara denumiri comerciale la
şi gaze tratamentul preeclampsiei, delivrentei dirijate, pag
551, subcapitol 7
E. Durere toracică accentuată de tuse şi inspir
profund *1 Factorii de risc pentru sarcina extrauterină
(pag. 517) sunt următorii, cu excepţia:
A. Tratamentul cu clomifen citrate
B. Endometrioza pelvină
C. Antecedente de salpingită
D. Tratament cu Metrotrexat
E. Antecedente de sarcină extrauterină
(pag. 533)

Pag. 57 din 130


*2 Sunt semne directe de sarcină extrauterină: 7 Scorul Fernandez in evaluarea oportunitatii
tratamentului chirurgical al sarcinii
A. Endometrul decidualizat
extrauterine cuprinde:
B. Evidenţierea ecografică a sacului gestaţional
extrauterin A. nivelul HCG

C. Hemoperitoneu B. nivelul estrogenilor

D. Sac gestaţional intrauterin absent C. nivelul progesteronului

E. Hematosalpinge D. dimensiunea hematosalpingelui


(pag. 534) E. prezenta hemoperitoneului
(pag. 535)
3 Semne indirecte de sarcină extrauterină sunt
urmatoarele: *8 Testul O’sullivan in dozarea glicemiei
A. Hematosalpinge venoase la cat timp dupa ingerarea a 50 g de
glucoza ?
B. Hemoperitoneu
A. 30’
C. Evidenţierea ecografică a sacului gestaţional
extrauterin B. 1h
D. Endometru gros, decidualizat C. 1.5h
E. Sac gestaţional intrauterin absent D. 2h
(pag. 534) E. 3h
(pag. 531)
4 Sarcina extruterină necomplicată (care nu s-
a rupt) se manifestă clinic prin: 9 Obiectivele conduitei dupa descoperirea
A. Dureri pelvine moderate si metroragie la diabetului gestational sunt:
începutul sarcinii A. Glicemia a jeun < 0,95 g/l
B. Durere violenta, sincopală B. Glicemia a jeun < 1,2g/l
C. Laterouterin, la tuşeul vaginal, se palpează masă C. Glicemia la 2 ore postprandial < 1,2 g/l
tumorală
D. Glicemia la 1 ora postprandial < 1,2 g/l
D. Tahicardie
E. Glicemia la 2 ore postprandial < 1,4 g/l
E. Durere ce iradiază la nivelul umerilor
(pag. 532)
(pag. 534)
* 10 Sarcina heterotopica reprezinta:
*5 Sarcina heterotopica reprezinta localizarea:
A. Sarcina extrauterina tubara
A. tubara
B. Sarcina extrauterina ovarian sau abdominala
B. interstitiala
C. Sarcina cervicala
C. peritoneala
D. Coexistenta unei sarcini intrauterine cu una
D. cervicala extrauterina
E. localizarea concomitenta intra si extrauterina E. Sarcina extrauterina tubara bilaterala
(pag. 533) (pag. 533)
6 Semnele indirecte ecografice ale sarcinii 11 Tabloul clinic al sarcinii extrauterine rupte
extrauterine sunt : cuprinde:
A. absenta sacului gestational intrauterin A. Aparare abdominala
B. vizualizarea unui sac gestational extrauterin B. Dureri pelvine moderate la debutul sarcinii
C. hematosalpinx C. Durere ce iradiaza in umeri
D. hemoperitoneu D. Durere violenta sincopala
E. endometru gros, decidualizat E. Semne de anemie
(pag. 534) (pag. 534)

Pag. 58 din 130


* 12 Care dintre urmatoarele examene paraclinice 17 Examene paraclinice, obligatorii, in timpul
nu sunt obligatorii sau recomandate in primei consultatii sunt:
timpul primei consultatii sunt:
A. VDRL si TPHA
A. Grup sangvin, Rh B. grupa de sange si factorul Rh
B. Testul de toleranta la glucoza C. proteinurie si glicozurie
C. VDRL D. serologie HIV, CMV
D. Proteinurie E. frotiu cervico-vaginal
E. Frotiu cervico-vaginal (pag. 522)
(pag. 522)
18 Preeclampsia severa se manifesta prin:
13 Sindromul HELLP se caracterizeaza prin:
A. Tas>160mmHg si Tad>110mmHg
A. hemoliza B. dureri in epigastru, greturi, varsaturi
B. citoliza hepatica C. proteinurie >2,5g/ zi
C. leucocitoza D. hemoliza
D. d.trombopenie E. trombocitopenie<80.000mm3
E. anemie (pag. 525)
(pag. 528)
19 Hematomul retroplacentar se caracterizeaza
* 14 In cazul unei HTA de novo,precizati semne prin:
asociate care justifica suspectarea unei
A. sangerare redusa cu singe negricios
preeclampsii:
B. sangerare abundenta
A. uricemia mai mica de 350micromol/l
C. ecografic imagine de hematom ce decoleaza
B. scaderea ASAT sub valoarea normala placenta
C. trombocite sub 150.000/mm3 D. anomalii ale RCF
D. absenta edemelor E. context de preeclampsie
E. macrosomie fetala (pag. 530)
(pag. 525)
* 20 Obiectivele primei consultaţii în sarcina
15 Diagnosticul diferential al unei sarcini normală sunt următoarele, cu excepţia:
extrauterine care nu s-a rupt se face cu: A. confirmarea stării sarcinii
A. avort B. evaluarea factorilor de risc
B. ruptura unui chist hemoragic C. evaluarea permeabilităţii colului uterin
C. torsiunea anexiala(daca durerile pelvine sunt D. informarea pacientei asupra depistării trisomiei
severe si fara metroragii) 21
D. ruptura unui anevrism al arterei splenice E. stabilirea unui plan de monitorizare adaptat
E. apendicita contextului clinic
(pag. 535) (pag. 521)

* 16 In prescrierea dietei diabetului gestational * 21 Semnele sugestive pentru diagnosticul de


sunt urmarite urmatoarele principii sarcină sunt următoarele, cu excepţia :
A. insulinoterapia va fi instaurata de la inceput daca A. creşterea tensiunii sânilor
glicemia a jeun este mai mica de 1,3g/l B. greţuri matinale
B. alimentatia va contine 50% lipide C. creşterea temperaturii bazale
C. ratia energetica totala va fi repartizata pe trei D. col violaceu la examenul cu speculul
mese si trei gustari
E. uter moale la tuşeul vaginal
D. insulinoterapia va fi instaurata de la inceput daca
glicemia a jeun este mai mare de 3,6g/l (pag. 521)
E. alimentatia va contine 50% proteine
(pag. 532)

Pag. 59 din 130


22 Depistarea trisomiei 21 în sarcină se face pe 27 Sindromul HELLP presupune:
baza următorilor parametri:
A. hemoliza
A. vârsta pacientei B. hematemeza
B. dimensiunea translucenţei nucale fetale în primul C. citoliza hepatica
trimestru de sarcină
D. trombocitoza
C. tensiunea arterială maternă
E. trombopenie
D. markeri serici în primul trimestru
(pag. 528)
E. diametrul mediu al sacului gestaţional
(pag. 523) * 28 Termenul teoretic al sarcinii este:
A. Data ultimului ciclu + 9 luni
23 Bilanţul paraclinic în luna a 6-a de sarcină
presupune: B. Data ultimului ciclu + 14 zile + 9 luni
A. AgHbs C. Data ultimului ciclu + 10 zile + 9 luni
B. Hemogramă D. Data ultimului ciclu + 7 zile + 9 luni
C. trensaminaze hepatice E. Data ultimului ciclu – 14 zile + 9 luni
D. serologia toxoplasmozei la nevoie (pag. 521)
E. identificarea albuminei şi a glucozei în urină * 29 La ce data se prescrie prima examinare
(pag. 523) ecografica in sarcina?
A. Imediat ce se instaleaza amenoreea, la 4
* 24 Consultaţia şi bilanţul preanestezic în
saptamani, pentru a confirma prezenta sarcinii in
sarcină se efectuează:
uter
A. în luna a 3-a de sarcină B. La 7-8 saptamani de sarcina, cand se poate
B. în luna a 5-a de sarcină aprecia viabilitatea embrionara
C. în luna a 8-a de sarcină C. La 11-13 saptamani, cand se poate face primul
examen de anatomie fetala si diagnosticul
D. în luna a 7-a de sarcină
malformatiilor severe, se poate aprecia riscul
E. la termen pentru cromozomopatii (masurarea translucentei
(pag. 523) nuchale), poate fi diagnosticata Gemelaritatea,
se poate confirma sau corecta datarea sarcinii
* 25 Scopurile primei ecografii în sarcină sunt D. La 22-24 de saptamani, varsta la care se poate
următoarele, cu excepţia: face examinarea completa a morfologiei fetale
A. diagnosticul precoce al sarcinilor multiple E. La 35-37 de saptamani, pentru a aprecia
B. depistarea precoce a unor malformaţii severe greutatea fatului si a estima prognosticul de
nastere naturala al pacientei
C. diagnosticul leziunilor de col uterin
(pag. 523)
D. depistarea precoce a trisomiei 21
E. confirmarea sau corectarea vârstei sarcinii 30 Intre modificarile fiziologice din timpul
sarcinii fac parte:
(pag. 523)
A. Proteinuria > 300 mg/l
26 Managementul cazurilor de preeclampsie B. Leucocitoza <14000-16000/mm3
presupune: (hiperleucocitoza fiziologica)
A. internarea de urgenţă C. Scaderea fosfatazei alcaline
B. transferul in utero la o maternitate adaptată D. Cresterea VSH
pentru aceste cazuri
E. Hb <10g/100 ml (hemodilutie fiziologica)
C. tratament ambulatoriu
(pag. 522)
D. supravegherea permanentă materno-fetală
E. odihna pacientei 31 Preeclampsia severa presupune:

(pag. 526) A. Trombocitopenia <300 000/mm3


B. Sindromul HELLP
C. Oliguria cu diureza <100 ml/h
D. Proteinuria >3,5g/zi
E. cefalee persistenta
(pag. 525)

Pag. 60 din 130


* 32 Următorul element se suplimentează pe 37 Printre factorii de risc ai sarcinii extrauterine
parcursul sarcinii pentru toate gravidele: fac parte :
A. fier A. Tabagismul
B. calciu B. Alcoolismul cronic
C. magneziu C. Tratamentul cu clomifen citrat
D. vitamina D D. Antecedente de interventii chirurgicale in sfera
E. vitamina E hepatica

(pag. 524) E. Varsta sub 35 de ani


(pag. 533)
33 Preeclampsia severă reprezintă asocierea
preeclampsiei cu unul din semnele * 38 Preeclampsia severa se caracterizeaza prin :
următoare: A. Dureri in epigastru , greturi , varsaturi
A. proteinuria > 3,5 g/zi B. Proteinurie < 3.5g / zi
B. hemoliza C. Poliurie
C. trombopenia < 150000/mm3 D. Leucopenie
D. tulburări neurologice E. ASAT normala
E. sindrom HELLP (pag. 525)
(pag. 525)
39 Explorarile paraclinice obligatorii in timpul
34 Monitorizarea maternă în caz de primei consultatii ginecologice sunt:
preeclampsie include:
A. Grupa sangvina si factorul Rh
A. monitorizare cardio-respiratorie B. Serologie CMV si HIV
B. pulsoximetrie C. Proteinuria si glicozuria
C. ecografie fetală D. Hemograma pentru depistarea anemiei
D. diureza E. Frotiu cervico-vaginal
E. edeme (pag. 522)
(pag. 527)
40 Bilantul biologic si ecografia in SEU
* 35 Metroragia în primul trimestru de sarcină nu evidentiaza:
poate fi determinată de: A. beta-hCG> 1500 ui/l si absenta ecografica a
A. sarcină oprită în evoluție sacului intrauterin
B. sarcină extrauterină B. beta-hCG< 1500 ui/l
C. molă hidatidiformă C. progesteronemia< 5 ng/ml
D. placenta praevia D. progesteronemia> 2,5 ng/ml
E. avort spontan E. hemoperitoneu
(pag. 529) (pag. 534)

* 36 Localizarile posibile ale sarcinii extrauterine * 41 Indicatiile de tratament chirurgical al SEU


sunt , cu exceptia : sunt urmatoarele cu exceptia:
A. Tubara A. beta-hCG plasmatic > 10.000 ui/l
B. Interstitiala B. dureri abdominele severe
C. Intrauterina C. scor Fernandez ≥ 13
D. Cervicala D. scor Fernandez < 12
E. Peritoneala E. hematosalpinge > 4cm ecografic
(pag. 533) (pag. 535)

Pag. 61 din 130


* 42 Factorii de risc in preeclampsie sunt 3 Sindromul ovarelor polichistice este asociat
urmatorii cu exceptia: cu urmatoarele anomalii ale ciclului
menstrual:
A. antecedentele de preeclampsie la mama sau
sora A. Metroragie
B. multiparitatea B. Spaniomenoree
C. varsta inaintata a mamei C. Menoragie
D. obezitatea D. Amenoree
E. insulinorezistenta E. Hipomenoree
(pag. 525) (pag. 559)
43 Indicatiile pentru inducerea travaliului sau *4 Care dintre urmatoarele simptome nu sunt
operatia cezariana in preeclampsie sunt: caracteristice sindromului premenstrual?
A. HTA controlata A. Meteorism abdominal
B. eclampsie B. Dureri pelvine
C. hematom retroplacentar C. Iritabilitate
D. trombocite > 300.000 D. Mastodinie
E. sindrom HELLP E. Metroragie
(pag. 527) (pag. 561)

Tema nr. 49 *5 Metroragia este:


Anomalii ale ciclului menstrual si hemoragia genitală A. menstruatie prea abundenta
la femei
B. sangerare in afara menstruatiei
Bibliografie asociata temei:
C. menstruatie survenita la interval > de 45 zile
Book Des ECN, editia in limba romana, redactor
Laurent Karila, 2011 D. menstruatie in cantitate mica
pag. 559-561, 583-586 * fara denumiri comerciale E. lipsa menstruatiei
ale medicamentelor (pag. 559)
1 Care dintre urmatoarele afirmatii sunt 6 Cauze ale metroragiei in timpul sarcinii sunt:
valabile, referitor la fazele ciclului menstrual?
A. placenta praevia
A. Faza foliculara se incheie cu ovulatia
B. mola hidatidiforma
B. Faza luteala se incheie cu ovulatia
C. avortul spontan
C. Faza foliculara corespunde transformarii
D. polipul endometrial
foliculului in corp galben
E. adenomioza
D. Faza luteala corespunde transformarii foliculului
in corp galben (pag. 529)
E. Faza luteala corespunde menstruatiei 7 Cauze ale hemoragiei genitale in afara
(pag. 559) sarcinii sunt:

*2 Care este semnificatia menoragiei? A. polipul endometrial

A. Sangerare in afara menstruatiei B. ectropionul


C. endometrita
B. Cicluri neregulate si rare
D. hiperplazia endometriala
C. Menstruatie abundenta cantitativ
D. Absenta menstruatiei pentru mai mult de trei luni E. ruptura uterina
(pag. 584)
E. Menstruatii cu frecventa crescuta
(pag. 559) *8 Tratamentul recomandat al fibromului
intracavitar submucos < 4 cm este:
A. rezectia histeroscopica
B. miometrectomie laparoscopica
C. histerctomie
D. anexectomie
E. chiuretaj uterin
(pag. 585)
Pag. 62 din 130
*9 Spaniomenoreea reprezintă: 14 In ce consta bilantul de sangerare genitala in
afara sarcinii:
A. sângerarea între menstruaţii
B. menstruaţie în cantitate redusă A. hemoleucograma si trombocitele

C. menstruaţie abundentă B. ecografia pelvina pe cale abdominala si


transvaginala
D. cicluri menstruale prea neregulate si prea rare (>
45 zile) C. histerosalpingografia

E. absenţa menarhei D. examinarea clinica a colului cu specul,TV


(pag. 559) E. frotiu cervicovaginal
(pag. 585)
* 10 Despre sindromul premenstrual sunt
adevarate următoarele afirmaţii, cu excepţia * 15 In cazul hemoragiei genitale la femeie,care
sunt semnele de anemie greu tolerabila:
A. Apare predominant în adolescenţă si
premenopauză A. bradicardie
B. Mastodinie ciclică secundară unei insuficienţe B. cresterea temperaturii
foliculare
C. dispnee
C. Meteorism abdominal
D. hipertensiune arteriala
D. Tulburări de somn
E. marmorare
E. Iritabilitate (pag. 583)
(pag. 561)
* 16 Ce este metroragia?
11 Care este etiologia metroragiei în timpul
sarcinii? A. menstruatia prea abundenta in cantitate sau ca
durata
A. Adenomioza
B. cicluri prea neregulate si prea rare
B. Risc de avort
C. menstruatia in cantitate mica
C. Sarcina molară
D. sangerarea in afara menstruatiei
D. Administrarea de anticoagulante
E. absenta menarhei
E. Sarcina extrauterină (pag. 559)
(pag. 560)
* 17 Cauze uterine de hemoragie genitala inafara
12 Semne de anemie greu tolerabilă sunt sarcinii sunt urmatoarele, cu exceptia:
urmatoarele:
A. cancer endometrial
A. Tahicardia
B. ectropion
B. Lipotimia
C. polip endometrial
C. Dispnee
D. fibrom uterin
D. Transpiratia
E. adenomioza
E. Extremităţile reci (pag. 584)
(pag. 583)
* 18 Tratamentul menometroragiei asociate cu
13 Enumerati cauze uterine ale hemoragiei adenomioza este:
genitale in afara sarcinii:
A. miomectomia prin laparoscopie
A. cancerul endometrial
B. agonisti de GnRH
B. adenomioza C. sterilet cu eliberare de progestativ
C. ectropion
D. embolizarea arterelor uterine
D. endometrita
E. mioliza cu ultrasunete focalizate sau cu
E. cervicita radiofrecventa
(pag. 584) (pag. 585)

Pag. 63 din 130


19 Semne de soc in hemoragie : 25 Sindromul ovarului polichistic se asociază
cu următoarele semne:
A. hipotensiune arteriala
B. dispnee A. alopecie

C. extremitati reci B. spaniomenoree

D. transpiratii C. amenoree

E. vertij D. scădere ponderală


(pag. 583) E. hiperandrogenism
(pag. 559)
20 La femeile cu metroragie, aflate la
menopauza , se va efectua: 26 În orice caz de sângerare a femeilor de
vârstă fertilă (<50 ani) , se impune:
A. frotiu Papanicolau
B. biopsie endometriala, insotita de examen A. dozarea transaminazelor hepatice
anatomopatologic B. examinare cu specul a colului uterin
C. histerosalpingografie C. tuşeu vaginal
D. laparoscopie D. test de sarcină
E. ecografie pelvina E. tuşeu rectal
(pag. 561) (pag. 559)
21 Hemoragia genitala de cauza uterina in afara * 27 Menarha defineşte:
sarcinii este data de:
A. debutul menopauzei
A. cancerul endometrial
B. prima menstruaţie
B. polipul endometrial
C. ultima menstruaţie
C. ectropion
D. menstruaţie abundentă
D. endometrita
E. absenţa menstruaţiei
E. chistul ovarian (pag. 559)
(pag. 584)
* 28 Menoragia reprezintă:
* 22 Sangerarea genitala intre perioadele
A. sângerarea în afara menstruaţiei
menstruale este definiția pentru:
B. menstruaţie în cantitate redusă
A. menoragie
C. menstruaţie prea abundentă, în cantitate sau ca
B. menometroragie durată
C. metroragie D. absenţa menstruaţiei
D. hemoragie E. cicluri foarte rare
E. spaniomenoree (pag. 559)
(pag. 559)
29 Sindromul premenstrual presupune:
* 23 Durata menstruaţiei la specia umană este:
A. mastodinia ciclica, secundară unei insuficienţe
A. maxim 10 zile luteale
B. maxim 7 zile B. durei pelvine
C. minim 3 zile C. dismenoree
D. minim 7 zile D. epistaxis
E. 28 zile E. spaniomenoree
(pag. 559) (pag. 561)

* 24 Metroragia reprezintă: 30 În contextul hemoragiei genitale la femeie,


semnele de anemie greu tolerabilă sunt:
A. sângerarea în afara menstruaţiei
A. tahicardia
B. menstruaţie în cantitate redusă
B. dispneea
C. menstruaţie foarte abundentă
C. astenia
D. absenţa menstruaţiei
D. vertijul
E. cicluri foarte rare
(pag. 559) E. bradicardia
(pag. 583)
Pag. 64 din 130
* 31 Metroragia este: * 36 Menoragia este
A. Sangerarea de cauza vaginala A. menstruatia în cantitate mică
B. Sangerarea prea abundenta de cauza uterina, in B. absența menstruației pentru mai mult de trei luni
momentul menstruatiei C. cicluri prea neregulate si prea rare
C. Sangerarea in afara menstruatiei
D. menstruatia prea abundentă, în cantitate sau ca
D. Sangerarea menstruala neregulata durată
E. Sangerarea menstruala in cantitate mica E. absența menarhei
(pag. 559) (pag. 559)

* 32 Selectati afirmatia adevarata despre 37 Bilantul în caz de sângerare genitală in afara


metroragia la femeile aflate la menopauza: sarcinii cuprinde:
A. Este necesar examenul citoexfoliativ Babes A. cuantificarea sângerării
Papanicolaou, chiar in timpul hemoragiei, pentru B. hemoleucograma
a exclude cancerul de col uterin
C. ecografia pelvină
B. Este indicat examenul ecografic, pentru a se
masura endometrul D. feritinemia
C. Este indicata histerosalpingografia, pentru a fi E. Histerosalpingografia
evaluata cavitatea uterina si a exclude prezenta (pag. 585)
unui polip de endometru
D. Este indicata completarea scorului Higham * 38 Sindromul ovarului polichistic se asociaza
cu urmatoarele semne:
E. Este indicata in toate cazurile histerectomia
A. Spaniomenoreea sau amenoreea
(pag. 561)
B. Hipomenoreea
33 Bilantul in cazul sangerarii de cauza genitala
C. Dureri lomboabdominale
in afara sarcinii se face astfel:
D. Menoragie
A. Nu se face examenul de specialitate cu speculul,
contraindicat in timpul sangerarii E. Mastodinie
B. Se indica excluderea unei sarcini, prin dozarea (pag. 559)
beta HCG plasmatic si urinar
* 39 In caz de metroragie, este contraindicat:
C. Se indica hemoleucograma si numararea de
trombocite A. Sterilet cu eliberare de progestativ

D. Este absolut contraindicata biopsierea B. Histerectomia, ca ultima solutie


endometrului, pana la oprirea sangerarii C. Distrugerea endometrului
E. Se indica feritinemia D. Frotiu Papanicolau
(pag. 585) E. Antifibrinolitice
34 Intre cauzele metroragiei in afara sarcinii (pag. 561)
intra:
40 Sindromul premenstrual se caracterizeaza
A. Cancerul de col uterin prin:
B. Fibromioamele subseroase A. Dureri pelvine, dismenoree
C. Endometrita B. Dureri lombare
D. Torsiunea de ovar C. Senzatie de meteorism abdominal
E. Cancerul de endometru D. Modificari ale caracterului : iritabilitate, tulburari
(pag. 584) de somn
E. Metroragie
35 În afara sarcinii în etiologia metroragiei pot fi
implicate: (pag. 561)

A. cancerul de col uterin


B. polipul endometrial
C. hematomul retroplacentar
D. avortul spontan
E. endometrita
(pag. 560)

Pag. 65 din 130


41 Hemoragia genitala in afara sarcinii este data * 46 Prima menstruatie poarta denumirea de :
de:
A. metroragie
A. Hematom retroplacentar B. menoragie
B. Cancerul ovarian C. menarha
C. Adenomioza D. mamografie
D. Endometrita E. amenoree
E. Chist ovarian (pag. 559)
(pag. 560)
47 Tratamentul in SOP poate consta in :
42 În sindromul ovarului polichistic apare:
A. reducerea greutatii
A. hiperandrogenism B. administrarea de metformin
B. chisturi ciocolatii C. sterilet ce elibereaza progestativ
C. infertilitate, obezitate, hiperpilozitate D. administrarea de clomifen citrat
D. hipermenoree, polimenoree E. decorticare sau drilling ovarian chirurgical
E. spaniomenoree sau amenoree (pag. 559)
(pag. 559)
48 Metroragia in afara sarcinii poate fi cauzata
* 43 Clasificarea sângerărilor genitale şi de ;
tulburărilor ciclului menstrual nu include:
A. adenomioza
A. meno- metroragia B. endometrita
B. spaniomenoreea C. hematomul retroplacentar
C. amenoreea primară sau secundară D. polip endometrial
D. hipomenoreea E. ectropion
E. menarha (pag. 560)
(pag. 559)
* 49 Ce investigatie este contraindicata in caz de
44 Metroragia la femeile în postmenopauză: metroragie la femeia in menopauza
A. este evocatoare pentru cancerul endometrial A. ecografia pelvina
B. examenul ecografic identifică un endometru B. histerosalpingografia
subţire, atrofic C. biopsia de endometru
C. tratamentul este hormonal, pastila combinată
D. examen anatomopatologic
D. histerosalpingografia este utilă pentru
E. examen de boli de nutritie si metabolism
evidenţierea unui endometru hipertrofic
(pag. 561)
E. diagnosticul anatomo- patologic este obligatoriu,
prin prelevarea unei biopsii endometriale
Tema nr. 50
(pag. 561) Infecţiile genitale la femei
* 45 Sindromul premenstrual Bibliografie asociata temei:
A. predominant la femeile active sexual Book Des ECN, editia in limba romana, redactor
Laurent Karila, 2011
B. simptomatologia predomină în perioada pag. 576-578 * fara denumiri comerciale ale
ovulatorie medicamentelor
C. mastodinia ciclică presupune o patologie
mamară asociată 1 Tratamentul caror infectii genitale joase este
D. biopsia de endometru este obligatorie pentru asigurat de Metronidazol?
stabilirea diagnosticului A. Trichomonas vaginalis
E. beneficiază de AINS şi terapie anxiolitică B. Streptococ grup B
(pag. 561) C. Candidoza vaginala
D. Gardnerella vaginalis
E. Herpes virus genital
(pag. 576)

Pag. 66 din 130


2 Care simptome sunt caracteristice infectiei 7 Vaginita cu Candida prezinta urmatoarele
vaginale cu Candida albicans? simptome:
A. Leucoree gri A. leucoree gri, urat mirositoare
B. Leucoree verde B. leucoree verde
C. Leucoree alba C. leucoree alba
D. Depozite vaginale albicioase D. prurit
E. Eritem vaginal E. dispareunie
(pag. 576) (pag. 576)

*3 Care dintre urmatoarele entitati patologice 8 Vaginita cu Trichomonas prezinta


nu reprezinta o complicatie a infectiilor urmatoarele simptome:
genitale inalte?
A. leucoree gri, urat mirositoare
A. Infertilitatea tubara B. leucoree verde
B. Sarcina extrauterina C. leucoree alba
C. Dureri pelvine cornice D. prurit
D. Vulvo-vaginite E. dispareunie
E. Recidive (pag. 576)
(pag. 578)
*9 Vulvo-vaginita cu Trichomonas vaginalis
*4 Testul pozitiv cu hidroxid de potasiu este prezintă urmatoarele simptome, cu excepţia:
diagnostic pentru infectia cu:
A. Leucoree verde
A. Candida albicans B. Prurit
B. Trichomonas vaginalis C. Dispareunie
C. Gardnerella vaginalis D. Aspect normal al colului uterin
D. Chlamidia trachomatis E. Arsuri vaginale
E. Papiloma virusuri (pag. 576)
(pag. 576)
* 10 Despre infecţiile genitale joase,
*5 Testul cu hidroxid de potasiu se face pentru vulvovaginitele, sunt adevarate următoarele
confirmarea infectiilor vaginale cu: afirmaţii:
A. Trichomonas A. Sunt insotite de febra
B. Candida B. Dureri pelviabdominale
C. Gardnerella C. Prezenţa abundentă a lactobacililor
D. Chlamidia D. Favorizate de un tratament antibiotic
E. Mycoplasma E. Dureri la mobilizarea uterina
(pag. 576) (pag. 576)

*6 Tratamentul infectiei cu Trichomonas se face 11 Despre endometrită sunt adevarate


prin administrarea de : următoarele afirmaţii:
A. Amoxicilina A. Este însoţită de febră
B. Ofloxacina B. Mobilizarea uterina fără durere
C. Metronidazol C. Leucoree purulentă
D. Antiinflamatoare D. Dureri abdomino-pelvine
E. Analgezice E. Apare post-partum sau post-avort
(pag. 576) (pag. 577)

Pag. 67 din 130


12 Tratamentul piosapinxului cuprinde: 18 Complicatiile pe termen lung ale infectiilor
genitale inalte pot fi:
A. Laparoscopie, drenaj si evacuare de puroi
B. Puncţie diagnostic A. aderente pelvine

C. Antiinflamatoare nesteroidiene şi corticoterapie B. sarcina extrauterina

D. Montare DIU C. endometrite

E. Antibioterapie D. distrugerea mucoasei ciliate a trompelor uterine


(pag. 577) E. piosalpinx
(pag. 578)
13 Trichomonas vaginalis se caracterizeaza
prin: 19 Candida albicans se manifesta clinic prin:
A. eritem vaginal A. leucoree gri
B. leucoree verde,urat mirositoare B. prurit
C. cervicita C. depozite albicioase
D. leucoree gri,urat mirositoare D. modificarea aspectului colului
E. disurie E. dispareunie
(pag. 576) (pag. 576)

* 14 Care sunt infectiile genitale inalte: * 20 Semnele endometritei sunt următoarele, cu


excepţia:
A. vaginita bacteriana
B. endometrita A. febra

C. micoza B. dureri abdominopelvine

D. Trichomonas vaginalis C. vărsături

E. vulvovaginita D. dureri la mobilizarea uterină


(pag. 576) E. leucoree
(pag. 577)
15 Piosalpingele se caracterizeaza clinic prin:
* 21 Diagnosticul salpingitei presupune
A. febra
următoarele semne, cu excepţia :
B. leucoree murdara sau purulenta
A. leucoree purulentă sau murdară
C. frison
B. hipermenoree
D. dureri abdominopelvine,uneori pulsatile si
insomniante C. dureri abdominopelvine

E. dispareunie D. febra
(pag. 576) E. dureri în hipocondrul drept în cadrul unui sindrom
Fitz-Hugh-Curtis
* 16 Vaginita bacteriana se caracterizeaza prin: (pag. 577)
A. leucoreea gri,urat mirositoare(de peste putred)
22 Piosalpingele presupune din punct de vedere
B. eritem vaginal clinic:
C. depozite albicioase(lapte batut) A. amenoree
D. leucoree verde,urat mirositoare B. febră
E. disurie C. dureri abdominopelvine
(pag. 576) D. leucoree murdară sau purulentă
* 17 Vaginita bacteriana se manifesta clinic prin: E. masă laterouterină dureroasă
A. febra (pag. 577)
B. dureri abdominopelvine
C. leucoree
D. modificarea aspectului colului
E. durere la mobilizarea uterina
(pag. 576)

Pag. 68 din 130


23 Complicaţiile pe termen mediu şi lung ale * 28 Trichomonasul este:
infecţiilor genitale înalte sunt:
A. O bacterie mobila
A. infertilitatea tubară B. O micelie mobila
B. sarcina extrauterină C. Un protozoar mobil
C. adenomioza D. Un germene ce nu poate fi transmis pe cale
D. aderenţe pelvine sexuala
E. recidive E. Un germene din categoria celor implicate in
(pag. 578) aparitia bolii inflamatorii pelvine cronice
(pag. 576)
* 24 Complicaţiile pe termen mediu şi lung ale
infecţiilor genitale înalte sunt următoarele, * 29 Complicatiile pe termen lung ale infectiilor
genitale inalte includ:
A. infertilitatea tubară
A. Cresterea riscului de sarcina multipla
B. sarcina extrauterină
B. Cresterea riscului de infectii renale
C. adenomioza
C. Cresterea riscului de infertilitate de cauza tubara
D. aderenţe pelvine
D. Cresterea riscului de cancer ovarian
E. recidive
(pag. 578) E. Cresterea riscului de cancer mamar
(pag. 578)
* 25 Tratamentul piosalpingelui presupune:
30 Vaginita bacteriana (Gardnerella vaginalis):
A. chimioterapie
A. Prezinta leucoree verde, urat mirositoare
B. laparoscopie cu drenaj şi evacuare de puroi din
piosalpinge B. Prezinta leucoree gri, urat mirositoare, de peste
putred
C. radioterapie
C. Prezinta leucoree alba, zgrunturoasa,
D. numai antibioterapie
nemirositoare
E. histeroscopie
D. Colul are aspect de cervicita
(pag. 578)
E. Colul are aspect normal
26 Simptomele în infecţia genitală cu (pag. 576)
Trichomonas vaginalis sunt:
31 Piosalpinxul:
A. leucoree verde, urât mirositoare
A. Da febra
B. spaniomenoree
B. Nu da febra
C. dispareunie
C. Da dureri importante pelvine
D. prurit vulvovaginal
D. Se insoteste de prezenta de masa laterouterina
E. cervicită
dureroasa la tuseul vaginal
(pag. 576)
E. Este o infectie torpida, paucisimptomatica
27 Simptomele în infecţia genitală cu Candida (pag. 576)
albicans sunt:
32 Piosalpingele se caracterizează prin:
A. prurit
A. febră
B. leucoree albă
B. cefalee
C. metroragie
C. dureri abdominopelvine
D. eritem vaginal
D. leucoree purulentă
E. dispareunie
E. masă latero-uterină dureroasă la tușeul vaginal
(pag. 576)
(pag. 577)

Pag. 69 din 130


33 Următoarele afirmații despre endometrită 39 Complicatiile pe termen mediu si lung ale
sunt adevărate: infectiilor genitale inalte sunt :
A. febră A. Infertilitate tubara
B. dureri abdomino-pelvine B. Litiaza renala
C. dureri la mobilizarea uterină C. Sarcina extrauterina
D. hidroree D. Cistita acuta hemoragica
E. leucoree purulentă E. Boala abortiva
(pag. 577) (pag. 578)

* 34 Diagnosticul diferențial al salpingitei nu se * 40 Endometrita post- avort nu se caracterizeaza


face cu: prin:
A. apendicita A. febra
B. diverticulita B. durere la mobilizarea uterina
C. sarcina ectopică C. dureri pelvine
D. sarcina intrauterină D. dureri in hipocondrul drept
E. chisturi ovariene E. leucoree purulenta, murdara
(pag. 577) (pag. 577)

* 35 Complicațiile pe termen mediu și lung ale 41 Diagnosticul diferential al salpingitei se face


infecțiilor genitale înalte nu includ: cu:
A. infertilitatea tubară A. chisturi ovariene
B. boala abortivă B. sarcina ectopica
C. sarcina extrauterină C. pancreatita
D. dureri pelvine cronice D. pielonefrita
E. recidive E. colecistita
(pag. 578) (pag. 577)

* 36 Vaginita bacteriana se caracterizeaza prin : 42 Complicatiile pe termen mediu si lung ale


infectiilor genitale inalte sunt:
A. Leucoree urat mirositoare
B. Eritem vaginal A. sarcina extrauterina

C. Prurit vulvovaginal B. fibromatoza uterina

D. Depozite albicioase ( lapte batut ) C. cancerul de endometru

E. Dispareunie D. infertilitatea tubara


(pag. 576) E. dureri pelvine cronice
(pag. 578)
* 37 Diagnosticul diferential al salpingitei include
urmatoarele , cu exceptia : * 43 Vulvo-vaginita cu Trichomonas vaginalis se
caracterizeaza prin urmatoarele, cu exceptia:
A. Apendicita
B. Diverticulita A. prurit vulvo-vaginal

C. Colecistita B. senzatia de arsura vaginala

D. Avort septic C. dispareunie

E. Hidronefroza D. leucoree de culoare gri, urat mirositoare

(pag. 577) E. necesita tratament per os


(pag. 576)
38 Urmatoarele simptome sunt specifice
endometritei :
A. Febra
B. Cefalee
C. Leucoree purulenta
D. Lombalgii
E. Prurit
(pag. 577)
Pag. 70 din 130
Tema nr. 51 5 Semnele ecografice evocatoare ale
Patologia tumorală genitală pelvină şi mamară la malignitatii unui chist organic ovarian sunt :
femei A. ascita
Bibliografie asociata temei: B. componenta solida importanta
Book Des ECN, editia in limba romana, redactor C. existenta vegetatiilor
Laurent Karila, 2011
D. vascularizatia centrala si un indice de rezistenta
pag. 587-589, 598-600, 601-604 * fara denumiri
Doppler scazut
comerciale ale medicamentelor
E. vascularizatia periferica si un indice de rezistenta
*1 Următoarele semne ecografice, pentru Doppler crescut
chistul organic, sunt evocatoare pentru (pag. 588)
malignitate, cu excepţia:
6 Markeri tumorali cuprinsi in bilantul
A. Ascita
preterapeutic al maselor ovariene suspecte
B. Componenta solidă sunt :
C. Indice Doppler crescut A. HER – 2
D. Prezenţa vegetaţiilor B. CA 125
E. Vascularizaţie centrală C. ACE
(pag. 588) D. CA 19-9
*2 La femeile aflate la menopauză şi care E. CA 15-3
prezintă chist de ovar benign, se va (pag. 600)
recomanda:
*7 Screeningul cancerului mamar presupune:
A. CA 125
A. mamografie incepand de la 50 de ani
B. CA 19-9
B. mamografie incepand de la 30 de ani
C. CA 153
C. mamografie incepand de la 40 de ani
D. CEA
D. ecografie incepand de la 50 de ani
E. Mutaţia BRCA1 sau BRCA2
(pag. 588) E. ecografie bianual de la 50 de ani
(pag. 603)
3 Despre adenomioză sunt adevarate
următoarele afirmaţii: *8 Imunoterapie (erceptine) se administreaza :
A. Dismenoreea debutează înainte de menstruaţie A. in caz de supraexpresie HER-2
şi continuă, uneori, şi după încheierea B. in caz de CA-125 crescut
menstruaţiei
C. in caz de CA-125 scazut
B. Uterul nu poate fi complet invadat de adenomioză
D. in caz de CA-15-3 crescut
C. Este o patologie rară
E. in caz de CA-15-3 scazut
D. Poate fi asociata cu fibromul uterin
(pag. 603)
E. Reprezintă infiltrarea zonei joncţiunii cu ţesut
endometrial în cadrul miometrului *9 Chistul dermoid este din punct de vedere
(pag. 589) histologic o tumora cu celule:
A. Mucinoase
4 Lipomul:
B. Endometroide
A. Este o proliferare malignă a ţesutului adipos
înconjurat de o capsulă C. Tranzitionale
B. Vârsta medie de apariţie este de 45 ani D. Germinale
C. Masă moale,mobilă, nedureroasă E. Clare
D. Impune tratament chirurgical (pag. 598)
E. Mamografie: zonă hiperclară, înconjurată de o
capsulă fină si opacă
(pag. 602)

Pag. 71 din 130


10 Care dintre urmatoarele situatii clinice * 15 Natura organica a chistului ovarian va fi
reprezinta complicatii ale chisturilor evocata de prezenta unuia dintre
ovariene? urmatoarele semne:
A. Necrobioza A. componenta lichida
B. Hemoragia intrachistica B. una sau mai multe vegetatii intrachistice
C. Ruptura hemoragica a chistului C. perete gros
D. Torsiunea D. diametrul chistului mai mare sau egal cu 3 cm
E. Sepsis indiferent de aspectul ecografic

(pag. 599) E. ascita


(pag. 588)
* 11 Analiza histologica a zonei de rezectie larga
in tumora mamara canceroasa palpabila, sub * 16 Tratamentul adjuvant al carcinomului ductal
3 cm, include urmatoarele, cu exceptia: mamar consta in:
A. Tipul histologic al tumorii A. chimioterapie
B. Dimensiunea tumorii B. hormonoterapie
C. Invadarea marginilor de rezectie C. radioterapie externa a sanului restant
D. Examenul citologic pe lama D. tratament chirurgical
E. Expresia receptorilor pentru hormonii sexuali E. suprimarea functiei ovariene cu GnRh
(pag. 603) (pag. 604)

12 Factorii de risc pentru cancerul mamar 17 Distrofia fibro-chistica a sanului se


includ: caracterizeaza prin:
A. Antecedente personale patologice de tumori A. prezenta unui nodul sau mai multi, chiar placard
benigne mamare B. nu creste riscul cancerului de san
B. Antecedente personale patologice de tumori C. nodul mamar care este constant indiferent de
maligne mamare momentul ciclului
C. Multigestitate D. mastodinie cronica cu agravare premenstrual
D. Pubertate tardiva E. pot fi prezente adenopatii axilare homolaterale
E. Prima sarcina precoce (pag. 601)
(pag. 602)
18 Chisturile dermoide:
13 Care sunt factorii de risc pentru cancerul
A. contin fanere, grasime, fragmente os
mamar:
B. sunt tumori benigne de ovar
A. conditiile socioeconomice favorabile
C. sunt tumori maligne de ovar
B. pubertatea precoce,nuligestitate
D. se impune rezectie(chistectomie) in momentul
C. mutatia genelor BRCA1 si BRCA2 diagnosticului indiferent de dimensiune si
D. antecedente personale de cancer mamar simptomatologie
E. prima sarcina precoce. E. diagnosticul este confirmat de CT/RMN
(pag. 602) (pag. 598)

14 Mastoza sau distrofia fibrochistica a sanului: * 19 Adenofibromul mamar:


A. conduita terapeutica consta in administrarea de A. are dimensiune variabila in functie de momentul
progestative per os sau topice timp de 20 de zile ciclului menstrual
pe luna B. tegumentul din jurul sau este afectat
B. marimea chisturilor este variabila in functie de C. este prezenta adenopatie axilara homolaterala
momentul ciclului
D. este un nodul mamar izolat, superficial, bine
C. este o tumora mamara maligna delimitat, nedureros
D. creste riscul cancerului de san E. apare la femei peste 35 ani
E. tratamentul este eminamente chirurgical (pag. 601)
(pag. 601)

Pag. 72 din 130


* 20 Natura organica a chistului ovarian va fi * 25 Adenomioza este:
evocata de prezenta unuia dintre
A. infiltrarea cu ţesut endometrial în cadrul
urmatoarele semne:
miometrului
A. perete subtire B. o afecţiune genitală malignă
B. componenta lichidiana C. sindrom aderenţial pelvin postoperator
C. caracter unilocular D. infiltrarea cu ţesut endometrial la nivelul ovarului
D. una sau mai multe vegetatii intrachistice E. infiltrarea cu ţesut endometrial la nivelul marelui
E. diametrul chistului mai mare sau egal cu 4 cm. epiploon
(pag. 588) (pag. 589)

* 21 Una dintre particularităţile chisturilor * 26 Următoarele afirmaţii cu privire la piosalpinx


dermoide este: sunt adevărate, cu excepţia:
A. este o tumoră malignă A. este un abces tubar sau tubo-ovarian
B. dispare sub tratament estroprogestativ B. reprezintă o complicaţie a salpingitei
C. conţine grăsime, fanere, dinţi, fragmente de os C. este localizarea tubară a sarcinii ectopice
D. are exclusiv conţinut lichidian D. este o colecţie dureroasă latero-uterină vizibilă la
ecografie
E. se remite sub tratament antibiotic
E. se asociaza cu sindrom biologic inflamator
(pag. 598)
(pag. 589)
* 22 Tratamentul adjuvant al carcinomului mamar
ductal presupune 27 Ruptura hemoragică a unui chist ovarian are
următorul tablou:
A. chimioterapie
A. hemoperitoneu cu dureri pelvine severe
B. hormonoterapie
B. apărare musculară abdominală
C. chirurgie radicală
C. dismenoree
D. radioterapie externă a sânului restant
D. şoc hemoragic dacă sângerarea este severă
E. chirurgie conservatoare
E. bradicardie
(pag. 604)
(pag. 599)
23 Natura organică a unui chist ovarian poate fi
evocată de următoarele semne: 28 Afirmaţiile adevărate cu privire la
adenofibromul mamar sunt:
A. una sau mai multe vegetaţii intrachistice
A. apare la femeile tinere cu vârste între 15-35 de
B. spaniomenoree ani
C. componentă solidă B. este o tumoră benignă
D. caracter multilocular C. este o tumoră malignă
E. hiperandrogenism D. este un nodul mamar izolat, superficial, bine
(pag. 588) delimitat, ferm
24 Semne ecografice evocatoare ale malignităţii E. există adenopatie asociată
pentru un chist ovarian organic sunt: (pag. 601)
A. ascita
B. importanţa componentei solide
C. existenţa vegetaţiilor
D. indice de rezistenţă Doppler scăzut
E. conul de umbră posterior
(pag. 588)

Pag. 73 din 130


* 29 In cazul unei formatiuni tumorale pelvine la * 33 Mastoza este:
femeie, din bilantul paraclinic, se recomanda
A. O afectiune eminamente benigna a sanului, nu
de prima intentie:
creste riscul de cancer mamar
A. Tomografia computerizata, datorita sensibilitati si B. O afectiune preneoplazica a sanului, ce asociaza
specificitatii foarte mari un interval mic de timp de la diagnostic pana la
B. Rezonanta magnetica nucleara, datorita aparitia cancerului mamar
eficacitatii crescute si datorita absentei riscului C. O afectiune care asociaza diferite tipuri de
de iradiere leziuni, non-inflamatorii, non-neoplazice, si care
C. Examenul ecografic pelvin, examinare ieftina, creste riscul de cancer de san
neiradianta, accesibila, fara risc, sensibilitate si D. O afectiune care necesita intotdeauna biopsiere
specificitate mare si cura chirurgicala.
D. Interventia chirurgicala, care ofera examenul E. O afectiune benigna, care nu asociaza niciodata
histopatologic al piesei adenopatie axilara
E. Markerii tumorali (CA12-5, CA 19-9, CEA), (pag. 601)
markeri specifici si utili in procesul de diagnostic
(pag. 587) * 34 Screening-ul populational pentru cancerul de
san se face:
* 30 Selectati afirmatia adevarata despre
A. Cu examene mamografice anuale de la 35 de ani
adenomioza:
B. Cu examene ecografice, imediat dupa inceperea
A. Este o patologie rar intalnita vietii sexuale, annual
B. Este o leziune preneoplazica a uterului C. Cu examen RMN de san, la toate pacientele cu
C. Este frecvent intalnita la femei tinere, in asociere risc, annual
cu endometrioza D. Cu mamografie la doi ani la pacientele cu varste
D. Se poate asocia cu cresterea de volum a uterului peste 50 de ani
si cu dismenoree E. Cu identificarea mutatiei BRCA 1 si 2
E. Este intotdeauna o afectiune izolata, nu se (pag. 603)
asociaza cu fibromul uterin.
(pag. 589) 35 Intre factorii de risc pentru cancerul mamar
intra:
31 Natura organica a unui chist ovarian este
A. Multiparitatea, cu pauze scurte intre alaptari
evocata de:
multiple
A. Prezenta unui perete subtire B. antecedente personale de patologie benigna a
B. Prezenta de vegetatii sanului, cu atipii
C. Prezenta de septuri fine la examinarea C. Pubertatea tardiva
ecografica, evocatoare pentru hemoragie D. Prima sarcina tardiva
intrachistica
E. Menopauza precoce
D. Dimensiuni mai mici de 6 cm
(pag. 602)
E. Caracterul multilocular
(pag. 588) 36 Principiul ganglionului santinela axilar:
A. Chiar daca ganglionul santinela nu este invadat,
32 Cancerul ovarian este: este frecventa invadarea celorlalti, de aceea se
A. O boala ce apare cel mai frecvent la femei in indica evidarea ganglionara completa
plina viata reproductiva chirurgicala in toate cazurile
B. O boala frecvent diagnosticata in stadiile B. Tehnica a fost validata doar pentru tumori mai
precoce, daca pacienta se prezinta la medicul mari de 3 cm
specialist anual, si i se practica examenul Babes C. Localizarea ganglioniilor santinela se face
Papanicolaou palpator, intraoperator
C. O boala depistata tardiv frecvent, la aparitia D. Se practica examenul HP extemporaneu din
complicatiilor (exemplu ascita) ganglionii santinela
D. O boala ce apare frecvent in climax E. In caz de ganglion santinela metastatic se
E. O boala ce poate da colectii pleurale practica limfadenectomie axilara
(pag. 599) (pag. 603)

Pag. 74 din 130


* 37 Poate fi semn ecografic evocator al 42 Factorii de risc pentru cancerul mamar sunt:
malignitătii unui chist ovarian:
A. prima sarcina tardiva
A. ascita B. Obezitatea
B. importanta componentei lichide C. Alcoolismul
C. absenta vegetatiilor D. Prezenta alaptarii
D. vascularizarea periferică E. Tabagismul
E. indice de rezistentă Doppler crescut (pag. 602)
(pag. 588)
* 43 Abcesul mamar:
* 38 Torsiunea ovariană nu se caracterizează prin:
A. Complica alaptarea materna
A. durere cu iradiere epigastrică B. Nu este favorizat de fumat
B. vărsături C. Reprezinta proliferarea benigna a tesutului
C. apărare abdomino-pelvină adipos inconjurat de capsula
D. durere de tip lovitură de pumnal D. Este o tumora benigna ce se dezvolta din
canalele galactofore
E. durere cu iradiere lombară
(pag. 599) E. Renuntarea la fumat nu influenteaza recidivele.
(pag. 602)
39 Natura organică a unui chist ovarian este
evocată de prezența unuia din următoarele * 44 Hemoragia intrachistica se caracterizeaza
semne: prin:
A. una sau mai multe vegetatii intrachistice A. Hemoperitoneu
B. diametrul chistului mare sau egal cu 1 cm B. Durere pelvina, de multe ori fara aparare
C. caracter multilocular C. Beta-HCG pozitiv
D. perete gros D. Metroragie
E. perete subțire E. Durere severa de tip ‘lovitura de pumnal’.
(pag. 588) (pag. 599)

40 Următoarele afirmații despre mastoză sunt 45 Clasificarea histologică a tumorilor ovariene


adevărate: cuprinde:
A. creste riscul cancerului de sân A. tumori epiteliale
B. după terminarea menstruației simptomele se B. tumori ale stromei gonadale şi ale cordoanelor
atenuează sexuale
C. pot fi prezente adenopatii axilare homolaterale C. tumori cu celule germinale
D. tratamentul este eminamente chirurgical D. tumori de “rete ovarii”
E. tratamentul constă în administrarea de E. tumori borderline de ovar
progestative (pag. 598)
(pag. 601)
46 Complicaţiile chisturilor ovariene includ:
41 Piosalpinxul se caracterizeaza prin: A. involuţia în menopauză
A. Dureri,febra sau subfebrilitate B. torsiunea ovarului
B. Sindrom biologic inflamator C. ruptura hemoragica
C. Dismenoree D. degenerescenţa malignă
D. Ascita E. hemoragia intrachistică
E. Metroragie (pag. 599)
(pag. 589)

Pag. 75 din 130


* 47 Diagnosticul imunologic al tumorilor * 52 Modificările mamografice în cancerul mamar
ovariene presupune: sunt:
A. alfa- fetoproteina A. îngroşarea sau retracţia tegumentelor
B. beta- HCG B. prezenţa unei opacităţi stelate, neregulate
C. CA 125 C. modificari de sensibilitate
D. CA 19.3 D. opacitate cu contur regulat
E. progesteronul E. modificări de temperatură la nivelul formaţiunii
(pag. 600) tumorale
(pag. 603)
* 48 Privind cancerul ovarian urmatoarele
afirmaţii sunt adevărate: 53 Natura organică a unui chist ovarian este
evocată de:
A. nu prezintă factori de risc
A. diametrul chistului mai mare sau egal cu 6 cm
B. vârsta medie de apariţie este 40- 45 ani
B. componenta lichidiană
C. existenţa unei colecţii pleurale
C. caracter multilocular
D. tulburările digestive sunt rare
D. perete subţire, nevizualizabil
E. tratament de elecţie: histerectomie totală cu
anexectomie bilaterală E. prezenţa vegetaţiilor intrachistice
(pag. 599) (pag. 588)

* 49 Adenofibromul mamar: 54 Managementul unei tumori anexiale


suspecte presupune:
A. este forma comună de tumoră malignă
B. apare de obicei la vârste înaintate A. RMN pelvin

C. diagnosticul de certitudine este dat de biopsia B. CA 125


sub ghidaj ecografic C. CT toracoabdominopelvin
D. are limite imprecise, plurilobulat, nedureros D. histerosalpingografie
E. excizia chirurgicală se face numai la pacientele E. aspiraţia transvaginală sub ghidaj ecografic
în postmenopauză (pag. 589)
(pag. 601)
* 55 Semnele ecografice evocatoare ale
* 50 Screeningul pentru cancerul de sân: malignităţii nu includ:
A. este indicat la toate femeile în perioada de A. ascita
activitate sexuală
B. existenţa vegetaţiilor
B. este indicat la femeile peste 70 de ani
C. vascularizarea periferică
C. include examinare echografică
D. indice de rezistenţă Doppler scăzut
D. include examinare clinică şi mamografică
E. componenta solidă
E. este indicat anual la femeile peste 50 de ani
(pag. 588)
(pag. 603)
56 Indicaţia operatorie ( chistectomie prin
51 În distrofia fibrochistică a sânului: laparoscopie) se recomandă în caz de:
A. se asociază leziuni de diferite tipuri A. chist cu aspect organic evidenţiat prin ecografie
B. unul sau mai mulţi noduli, uneori apar scurgeri B. creştere de volum a chistului
mamelonare
C. prezenţa ascitei în cantitate mare
C. acuzele sunt exacerbate în perioada
premenstruală D. prezenţa unui chist follicular sau de corp luteal

D. adenopatii homolaterale suspecte E. în caz de modificări morfologice ale chistului

E. aderă la piele şi la planurile subiacente (pag. 589)

(pag. 601)

Pag. 76 din 130


57 Semnele ecografice evocatoare pentru Tema nr. 52
tumora maligna ovariana sunt; Poliartrita reumatoidă
A. ascita Bibliografie asociata temei:
B. vegetatii intrachistice Book Des ECN, editia in limba romana, redactor
C. componenta solida Laurent Karila, 2011
pag. 628-631 * fara denumiri comerciale ale
D. diametrul chistului sub 2,5 cm
medicamentelor
E. continut intrachistic clar, anecogen
(pag. 588) *1 Identificaţi afirmaţia corectă privind afectarea
articulară în poliartrita reumatoidă
* 58 Hemoragia intrachistica ovariana se
A. respectă articulaţiile metacarpofalangiene si
manifesta clinic prin;
interfalangiene proximale
A. durere severa tip " lovitura de pumnal" B. implică preferenţial articulaţiile interfalangiene
B. tablou de hemoperitoneu distale
C. aparare abdomino-pelvina obligatorie C. asociază frecvent simptome/semne
extraarticulare
D. durere pelvina, de multe ori fara aparare
D. este o poliartrită acută cu durata sub şase
E. soc hemoragic săptămâni
(pag. 599)
E. se caracterizează prin prezenţa sinovitei şi
* 59 Factorii de risc pentru cancerul mamar sunt eroziunilor cu pattern poliarticular
urmatorii, cu exceptia; (pag. 629)
A. pubertate precoce *2 Precizaţi afirmaţia corectă referitor la
B. menopauza tardiva evaluarea radiografică a unui pacient cu
poliartrită reumatoidă
C. alaptarea
D. nuligestitate A. necesită expunerea mâinii şi a pumnului pentru
identificarea semnelor de condrocalcinoză
E. antecedente de radioterapie toracica
B. nu identifică leziuni radiologice în primii 2 ani de
(pag. 602) evoluţie
60 Conduita terapeutica in cancerul mamar C. este obligatorie expunerea radiografică a
subclinic consta in: articulaţiilor tibio-tarsiene

A. tumorectomie D. absenţa leziunilor radiologice exclude


diagnosticul
B. radioterapie externa a sanului restant
E. expunerea 3/4 picior bilateral faţă poate identifica
C. chimioterapie pensări de spaţiu articular şi eroziuni
D. fara hormonoterapie (pag. 629)
E. limfadenectomie axilara
*3 Care dintre complicaţiile enumerate nu este
(pag. 604) întâlnită în poliartrita reumatoidă
A. pericardita
B. amiloidoza AA
C. glomerulonefrita proliferativă difuză
D. multinevrita
E. episclerita
(pag. 630)

*4 Reacţiile adverse la methotrexat nu includ


A. citopenia
B. stomatita
C. hirsutismul
D. infecţiile
E. citoliza hepatică
(pag. 631)

Pag. 77 din 130


*5 Tratamentul de fond convenţional în 9 Monitorizarea pacientului cu poliartrită
poliartrita reumatoidă utilizează reumatoidă include
A. etanercept A. evaluarea clinică a rigidităţii matinale şi durerii
B. leflunomidă articulare
B. determinarea markerilor biologici: VSH, CRP,
C. tocilizumab
ANA, ANCA
D. infliximab
C. evaluarea radiografică a mâinilor la fiecare trei
E. abatacept luni
(pag. 631) D. scorul de activitate a bolii DAS28
6 Evaluarea complementară a pacientului cu E. identificarea comorbidităţii cardiovasculare
poliartrită reumatoidă presupune (pag. 631)
A. cercetarea sindromului inflamator, VSH, CRP 10 Factori de prognostic ai severităţii în
B. identificarea terenului genetic HLA-DR0401, poliartrita reumatoidă sunt
0404, 0405
A. vasculita reumatoidă
C. determinarea anticorpilor anti-CCP şi a factorului
B. prezenţa eroziunilor la diagnostic
reumatoid
C. teren genetic HLA-B27
D. radiografia de mâini şi ecografia articulară
D. persistenţa sinovitei la 3 luni de tratament
E. testarea funcţiei hepatice şi a creatininei
(pag. 629) E. debut acut monoarticular
(pag. 629,630)
7 Afirmaţiile incorecte privind anticorpii anti-
CCP sunt Tema nr. 53
A. apar la 15% din vârstnicii peste 70 de ani Spondilită anchilozantă
B. sunt identificaţi frecvent în lupusul eritematos Bibliografie asociata temei:
sistemic, sclerodermie şi spondiloartropatii Book Des ECN, editia in limba romana, redactor
C. sunt foarte sensibili şi specifici pentru PR Laurent Karila, 2011
pag. 634-635
D. prezintă semnificaţie diagnostică dar nu şi
prognostică, de severitate a bolii
*1 Care dintre urmatoarele afirmatii despre
E. se pozitivează în infecţii cronice (virus hepatitic spondiloartropatii nu este adevarata:
C, tuberculoză, mononucleoză).
A. asociaza afectare de tip inflamator axial, periferic
(pag. 628) si entezal
8 Tratamentul de fond în poliartrita reumatoidă B. sunt cert definite de pozitivitatea pentru HLA-B27
presupune C. reunesc spondilita anchilozanta, artrita
A. utilizarea de primă intenţie a terapiei anti-TNF în psoriazica, artritele reactive, artritele din boli
formele uşoare şi moderate de boală inflamatorii intestinale, sindromul SAPHO;
B. iniţial administrarea de metotrexat sau D. pot avea manifestari extraarticulare oculare,
leflunomidă gastrointestinale, cutanate
C. administrarea de rituximab de primă intenţie în E. au frecventa similara poliartritei reumatoide
formele severe de boală (pag. 634)
D. evitarea hidroxiclorochinei în monoterapie
*2 Alegeti afirmatia adevarata privind testul
E. utilizarea terapiei combinate metotrexat + Schober in spondilita anchilozanta
sulfasalazină
A. este manevra de provocare a durerii in
(pag. 630,631) articulatiile sacroiliace
B. evalueaza afectarea inflamatorie a articulatiei
soldului
C. este util in aprecierea redorii coloanei lombare
D. reprezinta metoda de electie pentru aprecierea
expansiunii toracice
E. este indispensabil in aprecierea afectarii
articulare periferice
(pag. 634)

Pag. 78 din 130


*3 Care dintre semnele radiologice enumerate 8 Examenul RMN in spondilita anchilozanta
nu este specific spondilitei anchilozante evidentiaza
A. semnul Romanus A. efuziunea articulara de sold
B. sindesmofitele B. afectarea sacroiliaca si vertebrala in cazurile cu
C. aspectul de “vertebre patrate” radiografii normale
C. hiposemnal T1 dupa administrarea de
D. osteofitele
gadolinium si hipersemnal T2 la nivelul spatiilor
E. sacroiliita sacroiliace
(pag. 635) D. aspectul de spondilodiscita infectioasa
*4 Care dintre urmatoarele manifestari E. sinovita articulatiilor periferice in cazurile cu
extraarticulare nu este caracteristica aspect ecografic normal
spondilitei anchilozante (pag. 635)
A. amiloidoza AA 9 Evaluarea complementara in
B. nefropatia IgA spondiloartropatii consta in
C. boala de reflux gastroesofagiana A. determinarea prezentei HLA-B27
D. insuficienta aortica B. testarea VSH, CPR, factor reumatoid
E. uveita anterioara C. ecografia articulara
(pag. 634,635) D. radiografii ale bazinului, coloanei vertebrale
toraco-lombare, picioarelor
*5 Tratamentul cu metotrexat in spondilita
anchilozanta este util in controlul E. determinari ale hemogramei, transaminazelor,
ureei, creatininei
A. artritei periferice
(pag. 635)
B. afectarilor entezelor
10 Care dintre urmatoarele afirmatii privind
C. progresiei axiale a bolii
diagnosticul clinic al spondilitei anchilozante
D. uveitei nu sunt adevarate
E. afectarii coxofemurale A. durerea de spate de tip inflamator raspunde
(pag. 635) prompt la administrarea AINS si reaparare dupa
48 de ore dupa oprirea tratamentului
6 Alegeti afirmatiile adevarate privind afectarea
B. afectarea fesiera este obiectivata prin durere
entezelor in spondiloartropatii fesiera
A. se exprima frecvent prin talalgii C. artritele periferice sunt simetrice
B. consta in inflamatia zonei de insertie pe os a D. antecendetele familiale de spondiloartropatie,
tendoanelor si ligamentelor psoriazis, boli inflamatorii intestinale – sunt
C. raspunde la administrarea de sulfasalazina semnificative pentru diagnostic
D. se trateaza cu AINS E. niciuna dintre afirmatiile de mai sus
E. consta in dureri trohanteriene, sternale (pag. 634,635)
(pag. 634,635)

7 Tratamentul anti-TNF-alfa in spondilita


anchilozanta este
A. indicat in cazurile cu afectare vertebrala dupa
esec succesiv la trei AINS
B. indicat in afectarea coxofemurala
C. prescris dupa excluderea istoricului de
tuberculoza primara
D. singura optiune terapeutica a uveitelor
E. util in cazurile cu afectare a entezelor care nu
raspund la administrarea corecta a AINS
(pag. 635)

Pag. 79 din 130


Tema nr. 54 *5 Fracturile osteoporotice
Osteoporoză A. nu apar niciodata consecutiv unui traumatism
Bibliografie asociata temei: minim
Book Des ECN, editia in limba romana, redactor B. sunt extrem de frecvente la nivelul extremitatii
Laurent Karila, 2011 proximale a antebratului
pag. 636-640 C. afecteaza preponderent segmentul cervical al
coloanei vertebrale
*1 Osteoporoza este D. sunt localizate uzual la nivel vertebral
A. boala extrem de dureroasa indiferent de E. prezinta simptome neurologice in majoritatea
instalarea unei fracturi cazurilor
B. maladie difuza a scheletului, caracterizata prin (pag. 637)
scaderea masei osoase
C. afectiune rara de tip primar 6 Care afirmatii referitoare la osteoporoza sunt
incorecte
D. boala grava cand valoarea scorului T se
regaseste in intervalul -1 si -2.5 A. Terapia osteoporozei in absenta unor fracturi
este intotdeauna de tip farmacologic sistematic
E. afectiune preponderent diagnosticata la varsta
tanara B. Osteoporoza severa nu se asociaza decat
rareori cu prezenta uneia/mai multor fracturi
(pag. 636)
C. Diagnosticul de osteoporoza secundara nu se
*2 Care dintre urmatoarele afirmatii referitoare regaseste niciodata la un pacient cunoscut cu
la osteoporoza este adevarata boala celiaca
A. Osteoporoza primara este boala osoasa D. Testele de laborator ce trebuie efectuate la
fragilizanta cea mai frecventa pacientul cu osteoporoza cuprind doar:
hemograma, calciu seric si calciuria
B. Osteoporoza nu necesita teste de laborator in
cazul unei fracturi cu aparenta osteoporotica E. Maladia Marfan este o osteopatie fragilizanta
non-osteoporotica
C. Tratamentul osteoporozei presupune doar un
regim igieno-dietetic (pag. 637, 638)
D. Terapia osteporozei este strict medicamentoasa 7 Osteoporoza primara
E. Oricare dintre variantele de mai sus A. exclude alte cauze de osteopatie fragilizanta
(pag. 636) B. presupune evaluarea vitezei de sedimentare a
*3 Diagnosticul de osteoporoza primara hematiilor, electroforeza proteinelor serice,
calciului seric, creatininuriei
A. nu consta in excluderea osteoporozei secundare
C. poate induce afectiuni de tip malign
B. poate fi obiectivat la un pacient cu mastocitoza
D. poate fi diagnosticata atat la femei cat si la
sistemica
barbati
C. nu include evaluarea imunofixarii urinare sau a
E. nu este dureroasa, simptomatologia algica fiind
TSH-ului
de fapt prezenta doar in cazul unei fracturi
D. presupune in primul rand decelarea unei cauze (pag. 636,637)
generale de osteopatie fragilizanta non-
osteoporotica 8 Absortiometria cu raze X
E. consta in efectuarea frecventa a densitometriei A. este utilizata pentru terapia osteoporozei
osoase
B. masoara densitatea minerala osoasa la nivelul
(pag. 637) coloanei vertebrale lombare
*4 Osteoporoza secundara C. reda un scor T mai mare de -2.5 pentru
osteopenie
A. nu are niciodata o cauza endocrinologica
D. reda un scor T normal mai mare de -1
B. nu trebuie exclusa in momentul diagnosticului
formei primare intrucat reprezinta un alt tip de E. reprezinta un instrument important in diagnosticul
patologie osteoporozei secundare
C. se poate regasi la un pacient cu rezectie (pag. 636)
intestinala extinsa in antecedentele patologice
D. reprezinta un stadiu evolutiv al osteoporozei
primare
E. niciuna dintre variantele de mai sus nu este
corecta
(pag. 637)
Pag. 80 din 130
9 Bisfosfonatii *2 Radiculalgia simptomatica
A. reprezinta substanta activa a ranelatului de A. poate releva o spondilodiscita infectioasa
strontiu B. are un caracter strict mecanic, fara asocierea
B. reduc riscul fracturilor vertebrale si de sold semnelor generale de tip febril sau alterarea
C. au ca efecte secundare: osteonecroza aseptica starii generale
de cap femural si stomatita C. nu reprezinta o urgenta diagnostica sau
D. se administreaza concomitent cu raloxifenul, terapeutica putand fi usor recunoscuta clinic
dimineata pe stomacul gol fara a manca cu un D. traduce o afectare tronculara a nervului sciatic
pahar mare de apa popliteu extern
E. pot fi administrati la pacienti cu osteoporoza, de E. cand este asociata cu un sindrom rahidian si in
sex feminin inainte de 70 de ani cu DMO foarte cazul herniei de disc rupte, cu migrare trebuie
scazut (scorul T mai mic de -3) chiar si in cautata o compresie extrarahidiana
absenta unei fracturi (pag. 625)
(pag. 638,639)
*3 Semnele Leri si Lasegue
10 Cand indicati efectuarea densitometriei
A. sunt pozitive in cazul herniei rupte
osoase
B. traduc sciatica L3-L4
A. La populatia generala cu antecedente personale
de fractura la nivel vertebral lombar fara o C. redau originea discala a unei afectari radiculare
trauma majora D. sunt echivalente semnului soneriei
B. In cazul descoperirii unei tumori osoase E. niciunul dintre raspunsurile de mai sus nu este
vertebrale cel corect
C. La femei in postmenopauza cu un IMC mai mic (pag. 627)
de 19kg/m², cu exceptia celor care se afla in
terapie de substitutie hormonala cu TSH in doze *4 Sindromul de tunel carpian
mai mici decat cele necesare pentru protectia
A. se insoteste clinic de hiperestezie in teritoriul
oaselor
nervului median
D. La femei aflate in postmenopauza cu istoric
B. va fi obiectivat prin semnul Tinel efectuat prin
concomitent de tratament cu corticosteroizi
hiperflexia incheieturii mainii
pentru o perioada de cel putin 3 luni consecutive
la o doza care sa depaseasca 7.5 mg/zi C. conduce la acroparestezie nocturna, cu
echivalent al prednisonului interesarea teritoriului nervului median –fata
palmara a primelor 2 degete si jumatatea
E. La subiecti de sex feminin cu menopauza inainte
exterioara din medius
de 40 de ani
(pag. 638) D. conduce la acroparestezie nocturna cu
interesarea teritoriului nervului median –fata
palmara a primelor 3 degete si jumatatea
Tema nr. 55
exterioara din inelar
Radiculalgia şi sindromul de comprensiune nervoasă
E. nu poate avea semne functionale de tip
Bibliografie asociata temei: acroparestezie noctura
Book Des ECN, editia in limba romana, redactor (pag. 627)
Laurent Karila, 2011
pag. 625-627 *5 Teritoriul senzitiv al afectiunii radiculare cu
topografie la nivelul C5 va fi evaluat
*1 Lomboradiculalgia comuna
A. prin abductia bratului
A. este intotdeauna bilaterala cu exceptia stenozei
B. prin determinarea reflexului bicipital
de canal verterbal lombar
C. pe fata interna a bratului
B. rareori este relationata cu o deformare rahidiana
congenitala sau dobandita D. prin manevre pasive de mobilizare ale articulatiei
scapulo-humerale
C. se caracterizeaza prin hernie de disc mai ales la
pacientii varstnici cu un factor declansator E. in partea de sus a umerilor
D. presupune efectuarea de rutina a unui examen (pag. 626)
computer-tomografic
E. toate variantele de mai sus sunt corecte
(pag. 625)

Pag. 81 din 130


6 Tratamentul radiculalgiei si al sindromului de 9 Indicatiile chirurgicale ale unei
compresie nervoasa lomboradiculalgii
A. include administrarea pe cale sistemica si pe A. sunt rezervate doar cazurilor extrem de grave,
termen lung a unui corticoid cu potential letal
B. presupune terapia cauzei principale B. includ deficitul motor recent si radiculalgia
C. poate fi de tip medicamentos local hiperalgica
C. se adreseaza cazurilor cu afectiuni
D. nu include cura chirurgicala in ceea ce priveste
pluriradiculare, tulburari sfincteriene, dar si celor
sindromul de tunel carpian
cu dureri radiculare de origine discala cu
E. permite infiltratia cu corticosteroizi si la pacientii insufcienta functionala majora
cu tulburari de coagulare
D. nu exista indicatie chirurgicala in cazul
(pag. 626,627) lomboradiculalgiilor
7 Investigatiile suplimentare in cazul unei E. nu se poate recurge la terapie chirurgicala in
radiculalgii necomplicate includ: primele 8 saptamani de la diagnosticul
sindromului de coada de cal
A. decat radiografia de coloana vertebrala indiferent
de durata simptomatologiei algice (pag. 627)

B. nimic suplimentar, cu exceptia radiografiei de 10 Examinarile complementare in cazul


coloana vertebrala, hemoleucograma, CRP si sindromului de tunel carpian
hemostaza in cazul duratei sub 6 saptamani a
A. pot releva semne de denervare cu indicatie
simptomatologiei si numai daca s-au prevazut
chirurgicala imediata
inflitratii rahidiene
B. nu aduc nici un beneficiu comparativ cu
C. evaluare extensiva indiferent de durata
examenul clinic
simptomatologiei sau de afectiunile preexistente
C. includ efectuarea de radiografii ale mainilor
D. radiografie de coloana vertebrala, CRP,
hemostaza dupa 6 - 8 saptamani de la debutul D. presupun inclusiv prelevarea proteinuriei de 24
simptomatologiei de ore
E. CT rahidian dupa 6-8 saptamani de la instalarea E. vor cuprinde si hemograma, VSH,CRP
simptomatologiei algice (pag. 627)
(pag. 626)
Tema nr. 56
8 Care dintre urmatoarele afirmatii sunt corecte
Prescrierea şi monitorizarea antiinflamatoarelor
A. Un fumator cu radiculalgie C8-D1, cu orar corticosteroidiene şi necorticosteroidiene
inflamator si asociere de sindrom Claude- Bibliografie asociata temei:
Bernard-Horner poate fi diagnosticat cu
afectiune maligna pulmonara Book Des ECN, editia in limba romana, redactor
Laurent Karila, 2011
B. Nementionarea posibilitatii unei amiloidoze AL la pag. 645-650 * fara denumiri comerciale ale
un pacient de peste 65 de ani cu sindrom de medicamentelor
tunel carpian – reprezinta o eroare de diagnostic
C. Afectarea radiculara L5 include lezarea *1 Care dintre urmatorii agenti farmacologici
tronculara a nervului sciatic popliteu extern sunt de tip AINS
D. Cruralgia poate fi definita la nivel L3 sau L4 A. Pirazolii
E. Sindromul de coada de cal reprezinta indicatie B. Prednisonul
chirurgicala in cazul unei lomboradiculalgii
C. Betametazona
(pag. 626)
D. Ketotifenul
E. Lorazepamul
(pag. 646)

*2 In care dintre afectiunile enumerate mai jos


este permisa infiltrarea de corticosteroizi?
A. Status infectios activ
B. Bursita olecraniana
C. Artrita septica
D. Osteocondrom
E. Toate raspunsurile de mai sus sunt eronate
(pag. 649)

Pag. 82 din 130


*3 Ce tipuri de afectiuni dintre cele redate mai 8 Care dintre urmatoarele afirmatii sunt
jos, presupun administrarea pe o perioada incorecte
lunga a AINS
A. AINS au si efecte de tip analgezic si antipiretic
A. Rahialgii B. Etoricoxibul este un inhibitor selectiv al
B. Tendinite si bursite ciclooxigenazei 1
C. Traume sportive C. Indometacinul face parte din clasa pirazolilor
D. Spondilita anchilozanta D. AINS pot induce crize de crize de diverticulita
sigmoidiana
E. Guta cronica tofacee
(pag. 646) E. Administrarea de AINS poate avea drept efect
secundar, nefrita acuta interstitiala imunoalergica
*4 Care sunt reactiile adverse dupa (pag. 645,646)
administrarea steroizilor pe cale generala
9 Ce indicati in prescriptia corticosteroizilor
A. Hipercorticismul iatrogen, retard de crestere la
copil, amenoree, osteonecroza aseptica A. scaderea treptata a dozelor fara necesitatea unei
terapii de fond in poliartrita reumatoida
B. Boala Crohn, complicatii cardio-vasculare,
hiperlipemie, sigmoidita, acnee B. contraindicatia acestor substante
medicamentoase in vasculite sistemice, tip boala
C. Diverticuloza, cicatrici cheloide, infectii
Horton
oportuniste
C. administrarea in polimialgia reumatica in doza
D. Diabet zaharat tip I insulinonecesitant
initiala de „atac” cu diminuarea ulterioara treptata
E. Feocromocitom, hipotiroidism, hipopotasemie D. riscul de insuficienta suprarenala consecutiv unei
(pag. 648,649) diminuari a dozei de steroid sub 0,1mg/kgc/zi
impune efectuarea unui test la Synacthen
*5 Interactiunile medicamentoase ale AINS
includ E. terapia pe termen scurt nu impune descrestere
progresiva
A. antihipertensivele de tip sartan
(pag. 648)
B. fenitoina-cu risc de supradozaj
10 AINS pot fi administrate in siguranta in
C. corticosteroizii –indiferent de diagnostic si de
urmatoarele circumstante
simptomatologia algica
A. topic, de exemplu ketoprofenul fara a induce
D. salicilatii – cu risc de reactie de hipersenibilizare
fenomene de fotosensibilizare
imediata
B. la pacientul cu osteoartrita de genunchi, in puseu
E. inhibitorii de pompa de protoni
congestiv
(pag. 647,648)
C. in patologii radiculare
6 Este permisa D. in doze mari la pacientii cu boli inflamatorii
A. prescrierea a doua antiinflamatoare simultan intestinale
B. prescrierea de AINS concomitent cu E. in colagenoze simultan cu antiinflamatoare
anticoagulante orale corticosteroidiene
C. continuarea terapiei cu AINS si in afara (pag. 646,647)
perioadelor nedureroase din artroze
D. administrarea de corticosteroid la un pacient cu
poliartrita reumatoida
E. infiltrarea locala de corticosteroizi la pacientul cu
sindrom de tunel carpian
(pag. 646,647,648)

7 Reprezinta complicatii specifice la infiltratii


de corticosteroizi
A. hemartroza
B. ruptura tendinoasa in cazul infiltrarii periarticulare
C. risartroza
D. artrita acuta gutoasa
E. artrita psoriazica
(pag. 650)

Pag. 83 din 130


Tema nr. 57 *5 Fractura-dezlipire epifizară Salter 3 este
Fractura extremităţii inferioare a radiusului şi a fractura cartilajului de creștere cu
extremităţii superioare a femurului la adult şi A. Traiect numai prin cartilajul de creștere
particularităţile fracturilor la copil
B. Prelungire metafizară
Bibliografie asociata temei:
C. Prelungire epifizară
Book Des ECN, editia in limba romana, redactor
Laurent Karila, 2011 D. Prelungire metafizară și epifizară
pag. 653-654, 655-656, 664 E. Compresiune axială
(pag. 664)
*1 Fractura extraarticulară cu deplasare
posterioară a extremității inferioare a 6 Deformarea membrului inferior într-o
radiusului se numește: fractură dezangrenată a extremității
superioare a femurului este în:
A. Pouteau-Colles
B. Goyrand-Smith A. Rotație externă

C. Gerard-Marchand B. Rotație internă

D. Destot C. Abducție

E. Galeazzi D. Adducție
(pag. 653) E. Flexie
(pag. 655)
*2 Fractura extraarticulară cu deplasare
anterioară a extremității inferioare a 7 Următoarele afirmații referitoare la fracturile
radiusului se numește: extremității superioare a femurului sunt
adevărate:
A. Pouteau-Colles
B. Goyrand-Smith A. Fracturile cervicale sunt fracturi extraarticulare

C. Gerard-Marchand B. Fracturile cervicale prezintă risc de necroză


secundară a capului femural
D. Destot
C. Fracturile capului femural sunt relativ frecvente
E. Galeazzi
D. Fracturile cervicale prezintă risc de
(pag. 653) pseudartroză
*3 Conform clasificării Garden, fractura E. Fracturile masivului trohanterian au risc de
cervicală a femurului cu ruptură completă consolidare vicioasă
este de tip: (pag. 655)
A. Garden I 8 O fractură a extremității inferioare a
B. Garden II radiusului se caracterizează clinic prin:
C. Garden III A. Durere
D. Garden IV B. Impotență funcțională
E. Garden V C. Deformarea pumnului „în dos de furculiță” dacă
(pag. 655) deplasarea este anterioară
D. Leziune a nervului radial
*4 Fracturile intertrohanteriene sunt fracturi cu
traiect de fractură: E. Edem

A. Parcelare ale marelui trohanter (pag. 653)

B. Ce separă un fragment diafizar 9 Din complicațiile posibile ale unei fracturi a


C. Oblic de la marele la micul trohanter extremității inferioare a radiusului fac parte:

D. Orizontal între marele și micul trohanter A. Leziunea nervului radial


E. Sub nivelul micului trohanter B. Deschidere cutanată
(pag. 655) C. Sindromul de compartiment
D. Infecția zonei operate
E. Necroza osoasă
(pag. 654)

Pag. 84 din 130


10 Cel mai frecvent tratament al fracturilor la 4 Complicațiile generale și la distanță ale
copil este tratamentul ortopedic din infecțiilor acute ale părților moi sunt:
următoarele motive
A. Artroza
A. Risc de epifiziodeză postoperatorie (dacă B. Endocardita
intervenția chirurgicală este localizată pe
cartilajul de creștere) C. Tetanosul
B. Tolerabilitate bună a fixării vicioase prin D. Decompensarea în cazul unui teren favorizant
remodelarea ulterioară E. Flegmonul digital
C. Epifize fertile: aproape de genunchi și departe de (pag. 669)
cot
D. Formarea unui punct osos la nivelul leziunii 5 Tratamentul chirurgical al flegmonului digital
cartilajului de creștere se poate face prin:

E. Absența complicațiilor de rigiditate sau a A. Ablație prin contraincizii la extremitățile degetului


tromboflebitei sub ghips B. Antibioterapie cu spectru larg pentru 21 de zile
(pag. 664) C. Deschidere în Z a degetului

Tema nr. 58 D. Sinovectomie digitală


Infecţiile acute ale părţilor moi (abces, panariţiu, E. Excizia tendonului
flegmon al tecii) (pag. 670)
Bibliografie asociata temei:
*6 Tratamentul flegmonului digital în stadiul 2
Book Des ECN, editia in limba romana, redactor se face prin:
Laurent Karila, 2011
pag. 669-670 A. Dacă nu a colectat, antibioterapie pentru 10
zile și băi antiseptice de 2 ori/zi
1 Unele din semnele clinice locale ale B. 2 contraincizii la nivelul extremităților degetului,
abcesului părților moi sunt: apoi antibioterapie pentru 21 de zile
A. Colecția fluctuentă C. Excizia tendonului interesat dacă acesta a
necrozat
B. Febra
D. Deschidere în Z a degetului și sinovectomie
C. Hiperleucocitoza
digitală
D. Durerea pulsatilă
E. Antibioterapie cu spectru larg pentru 45 de zile
E. Hipotermia locală (pag. 670)
(pag. 669)
*7 Tratamentul flegmonului digital în stadiul 3
2 Flegmonul digital este caracterizat clinic prin: se face prin:
A. Mărirea în volum a degetului afectat A. Dacă nu a colectat, antibioterapie pentru 10 zile
B. Localizarea infecției și băi antiseptice de 2 ori/zi

C. Iradierea durerii de-a lungul învelișului B. 2 contraincizii la nivelul extremităților degetului,


apoi antibioterapie pentru 21 de zile
D. Durere la extensia pasivă a degetului
C. Excizia tendonului interesat dacă acesta a
E. Deformarea degetului cu încovoierea lui necrozat și reconstrucție ulterioară
(pag. 669) D. Deschidere în Z a degetului și sinovectomie
digitală
3 Din factorii favorizanți ai infecțiilor părților
moi fac parte: E. Antibioterapie cu spectru larg pentru 45 de zile
A. Hipotiroidismul (pag. 670)

B. Artroza la nivelul degetelor mâinii *8 Durata antibioterapiei în artrita (complicație


C. Diabetul zaharat locală în infecțiile acute ale părților moi) este
de:
D. Deficiență a statusului imunologic
A. 7 zile
E. Prezența de corpi străini
B. 14 zile
(pag. 669)
C. 21 de zile
D. 45 de zile
E. 60 de zile
(pag. 669)

Pag. 85 din 130


*9 Investigațiile de laborator în diagnosticul 3 Ocluzia arterei centrale a retinei se
pozitiv al infecțiilor acute ale părților moi caracterizeazǎ prin:
arată:
A. congestia ocularǎ
A. Hipertermie locală B. scǎderea bruscǎ a vederii
B. Leucopenie C. precipitate pe faṭa posterioarǎ a corneei
C. Trombocitoză D. d.sinechii irido-cristaliniene
D. Creșterea proteinei C reactive E. edem macular cu aspect de cireaṣǎ macularǎ
E. Scăderea vitezei de sedimentare a hematiilor (pag. 679)
(pag. 669)
*4 Ocluzia venei centrale a retinei se
* 10 În flegmonul digital, durerea este caracterizeazǎ prin:
caracterizată de:
A. artere ṣi vene subṭiri ṣi filiforme la nivelul retinei
A. Intensificare la extensia pasivă a degetului B. hemoragii retiniene striate sau rorunde
B. Intensificare la flexia pasivă a degetului C. edem macular ischemic cu aspect de cireaṣǎ
C. Iradiere până la pliul palmar proximal pentru macularǎ
degetele II, III și IV D. d.Tyndall pozitiv la nivelul camerei anterioare
D. Iradiere până la pliul pumnului pentru degetele E. vene ṣi artere ȋntecuite sub formǎ de
II, III, IV manṣoane perivasculare
E. Iradiere până la pliul palmar distal pentru (pag. 680)
degetele I și V
(pag. 669) *5 Tratamentul glaucomului acut presupune
urmǎtoarele metode cu excepṭia:
Tema nr. 59 A. beta-blocante administrate local
Anomalii ale vederii cu debut brutal
B. inhibitor de anhidrazǎ carbonicǎ administrate
Bibliografie asociata temei: local ṣi general
Book Des ECN, editia in limba romana, redactor C. midriatice administrate local
Laurent Karila, 2011
D. d.iridotomie la nivelul ochiului afectat
pag. 678-685 * fara denumiri comerciale ale
medicamentelor E. iridotomie la nivelul ochiului congener, sǎnǎtos
(pag. 682)
*1 Care dintre urmǎtoarele afecṭiuni este
caracterizatǎ de scǎderea brutalǎ a vederii cu 6 Care dintre urmǎtorii factori favorizeazǎ
ochi roṣu, dureros? apariṭia glaucomului acut prin ȋnchiderea
unghiului?
A. dezlipirea de retinǎ
B. neuropatia opticǎ ischemicǎ acutǎ anterioarǎ A. hipermetropia

C. glaucom neovascular B. administrare simpaticolitice

D. d.degenerescenṭa macularǎ legatǎ de vȃrstǎ C. administrare parasimpaticomimetice

E. ocluzia venei centrale ale retinei D. d.stress


(pag. 678) E. expunere prelungitǎ la ȋntuneric
(pag. 682)
*2 Care dintre urmǎtoarele afecṭiuni este
caracterizatǎ de scǎderea brutalǎ a vederii cu 7 Care dintre urmǎtoarele afirmaṭii sunt
ochi alb, nedureros? valabile ȋn cazul neuropatiei oprice
ischemice acute anterioare?
A. glaucomul acut
B. keratita acutǎ A. scǎderea brutalǎ a vederii

C. endoftalmitǎ B. durere ocularǎ, congestie ocularǎ apǎrutǎ cu


24 de ore ȋnainte de scǎderea acuitǎṭii vizuale
D. d.neuropatia opticǎ ischemicǎ anterioarǎ
C. amputarea cȃmpului vizual
E. glaucom neovascular
D. d.edem papilar cu hemoragii peripapilare
(pag. 678)
E. poate reprezenta o manifestare a bolii Horton
(pag. 681)

Pag. 86 din 130


8 Uveita anterioarǎ acutǎ este caracterizatǎ *2 In care dintre urmǎtoarele afecṭiuni
prin: caracterizate de ochi roṣu ṣi dureros nu
apare scǎderea acuitǎṭii vizuale?
A. hemoftalmus
B. cerc pericheratic A. conjunctivita bacteriana

C. efect Tyndall B. traumatism ocular cu hemoragie subconjunctivalǎ

D. d.precipitate endocorneene C. plagǎ conjunctivalǎ cu retenṭie de corp strǎin

E. sinechii iridocristaliniene, care forṭeazǎ pupila D. d. keratita acutǎ


sǎ rǎmȃnǎ ȋn midriazǎ maximǎ E. episclerita acutǎ
(pag. 684) (pag. 686)
9 Conduita terapeuticǎ a complicaṭiilor 3 Urmǎtoarele semne ṣi simptome sunt
ocluziei venei centrale a retinei includ: caracteristice pentru sclerita acutǎ
A. mǎsurarea tensiunii oculare si tratament anterioarǎ?
hipotonizant pentru glaucomul secundar A. acuitate vizualǎ normalǎ
B. fotocoagulare panretinianǎ pentru prevenirea B. ochiul prezintǎ roṣeaṭǎ localizatǎ care dispare
complicaṭiilor neovasculare dupǎ aplicarea unui vasoconstrictor
C. injectare intravitreeanǎ de corticosteroizi pentru C. apare durere ocularǎ intensificatǎ la miṣcǎrile
tratamentul edemului macular cistoid globului ocular
D. d.antibioterapie masivǎ pentru evitarea D. d. cristalinul prezintǎ opacifiere nuclearǎ sau
panoftalmiei oculare corticalǎ posterioarǎ
E. lavaj al camerei anterioare pentru eliminarea E. edem papilar bilateral, cu hemoragii ṣi exsudate
hipopionului peripapilare
(pag. 680) (pag. 687)
* 10 Care dintre formele de keratitǎ acutǎ este 4 Care dintre urmǎtoarele afecṭiuni pot
caracterizatǎ de ulcer dendritic sau ȋn hartǎ determina scleritǎ?
geograficǎ:
A. boala Behcet
A. keratita bacterianǎ
B. tuberculoză
B. keratitǎ herpeticǎ
C. sarcinǎ extrauterinǎ
C. keratitǎ cu adenovirus
D. d. boalǎ Crohn
D. d.sindrom cu ochi uscat
E. poliartritǎ reumatoidǎ
E. sindrom de expunere prin malpoziṭie palpebralǎ (pag. 687)
(pag. 684)
5 Care dintre urmǎtoarele afecṭiuni oculare
Tema nr. 60 sunt caracterizate de scǎderea acuitǎṭii
Ochiul roşu şi/sau dureros vizuale?
Bibliografie asociata temei: A. conjunctivita acutǎ
Book Des ECN, editia in limba romana, redactor B. plagǎ ocularǎ cu retenṭie de corp strǎin
Laurent Karila, 2011 intraocular
pag. 686-688 * fara denumiri comerciale ale C. keratitǎ acutǎ
medicamentelor
D. d. scleritǎ acutǎ anterioarǎ
1 Care dintre urmǎtoarele afecṭiuni sunt E. uveitǎ anterioarǎ acutǎ
caracterizate de scǎderea acuitǎṭii vizuale? (pag. 687)
A. conjunctivita acutǎ
*6 Glaucomul acut se caracterizeazǎ prin
B. Keratita acută urmǎtoarele semne ṣi simptome cu excepṭia:
C. Sclerita anterioară acută A. congestie ocularǎ
D. d. Glaucomul acut B. ochi dur datoritǎ creṣterii tensiunii intraoculare
E. Endoftalmita C. durere ocularǎ intensǎ
(pag. 687) D. d.miozǎ, cu pupilǎ fixǎ areflexivǎ
E. stare generalǎ alteratǎ, greṭuri, vǎrsǎturi
(pag. 682)

Pag. 87 din 130


*7 Urmǎtoarele semne ṣi simptome sunt Tema nr. 61
caracterizate pentru keratitǎ epitalialǎ acutǎ Otalgii şi otite la copii şi adulţi
cu excepṭia:
Bibliografie asociata temei:
A. congestie ocularǎ
Book Des ECN, editia in limba romana, redactor
B. scǎderea acuitǎṭii vizuale Laurent Karila, 2011
C. midriazǎ pag. 716-721 * fara denumiri comerciale ale
medicamentelor
D. d.reflex corneean modificat cu hipoestezie
corneeanǎ
1 Inervaţia senzitivă a urechii externe este
E. coloraṭie cu fluoresceinǎ pozitivǎ asigurată de:
(pag. 683) A. Nervul auriculotemporal
*8 Urmǎtoarele semne clinice sunt B. Nervul Wrisberg
caracteristice uveitei anterioare acute cu C. Ram auricular al nervului vag
excepṭia:
D. Nervul timpanic
A. acuitate vizualǎ normalǎ
E. Plex cervical superior
B. congestie ocularǎ
(pag. 716)
C. durere ocularǎ
2 Stadiile otitei medii acute sunt:
D. d.pupilǎ deformatǎe.apare cu recurenṭe
identice ȋn cadrul unei boli generale A. Stadiul de colectare
E. apare cu recurenṭe identice ȋn cadrul unei boli B. Stadiul de cicatrizare
generale C. Stadiul perforat
(pag. 684)
D. Stadiul congestiv
*9 Lentilele de contact pot predispune la una E. Stadiul cataral
sau mai multe dintre urmǎtoarele afecṭiuni:
(pag. 717)
A. glaucom acut
3 Otita externă se manifestă prin:
B. keratita epitelialǎ acutǎ
A. Hipoacuzie neurosenzorială
C. episcleritǎ acutǎ
B. Otalgie
D. d.cataractǎ traumaticǎ
C. Edem al conductului auditiv extern
E. corioretinitǎ
D. Congestia timpanului
(pag. 688)
E. Ameţeli
* 10 Uveita acutǎ este caracterizatǎ de
(pag. 718)
urmǎtoarele aspecte clinice cu excepṭia:
A. precipitate pe endoteliul corneean 4 Semnele de atenţionare ale unei posibile
otite seromucoase la copil sunt:
B. efect Tyndall al umorii apoase
A. Tulburări de atenţie
C. sinechii iridocristaliniene
B. Otite externe repetate
D. d.focar inflamator corioretinian localizat
peripapilar sau la nivelul retinei periferice C. Întârzierea vorbirii
E. focar inflamator localizat la 2-3 mm de limbul D. Tulburări de comportament
sclerocorneean E. Somnolenţa
(pag. 684) (pag. 719)

5 La impedanţmetrie în cadrul explorării otitei


seromucoase găsim:
A. Curba tip A
B. Curbă tip B
C. Curbă tip C
D. Curba tip D
E. Curbă tip A şi C
(pag. 720)

Pag. 88 din 130


*6 Suspiciunea de otita medie acută cu Tema nr. 62
Haemophilus influenzae se pune când este Angine şi faringite ale adultului
asociată otitei:
Bibliografie asociata temei:
A. Otoragie
Book Des ECN, editia in limba romana, redactor
B. Conjunctivita Laurent Karila, 2011
C. Somnolenţă pag. 722-727 * fara denumiri comerciale ale
medicamentelor,fara tratamentul anginei pag. 723 -
D. Disfagie
recomandarea AFSSAPS
E. Meningita
(pag. 718) 1 Angina ulcero-necrotică se întâlneşte în:
A. Mononucleoza infecţioasă
*7 Complicaţiile otitei medii acute nu includ:
B. Difterie
A. Mastoidita
C. Angina Vincent
B. Tromboflebita sinusului lateral
D. Şancru sifilitic
C. Nevralgia facială
E. Herpes
D. Menigita
(pag. 722)
E. Abcesul cerebral
(pag. 718) 2 Angina veziculară se întâlneşte în:
A. Herpangină
*8 Otita seromucoasă cronică are o durată a
evoluţiei simptomelor de: B. Gripă
A. 7 zile C. Herpes
B. 3 săptămâni D. Difterie
C. 4 săptămâni E. Zona nervului IX
D. 2 luni (pag. 722)

E. 3 luni 3 Angina eritemato-pultaceee se întâlneşte în:


(pag. 719) A. Angina Vincent
*9 Otita seromucoasă cronică este secundară: B. Angina streptococică
A. otitelor furunculoase C. Difterie
B. disfunctiei trompei lui Eustachio D. Herpangina
C. mastoiditei E. Mononucleoza infecţioasă
D. otitelor externe virale (pag. 722)

E. traumatismelor otice 4 În angina eritemato-pultacee depozitele


(pag. 719) pultacee sunt:
A. Aderente pe amigdale
* 10 Durata tratamentului antibiotic per os
recomandat la copii peste 2 ani în otita medie B. Neaderente pe amigdale
acută este: C. Albicioase
A. 2 zile D. Cenuşii
B. 5 zile E. Punctiforme
C. 9 zile (pag. 722)
D. o zi 5 Diagnosticul pozitiv în mononucleoza
E. 14 zile infecţioasă se stabileşte pe baza:
(pag. 720) A. Reacţia VDRL
B. Hemogramei
C. Reacţia Paul-Bunnel-Davidsohn
D. Serologia HIV
E. Serologia EBV
(pag. 725)

Pag. 89 din 130


*6 Difteria are ca agent etiologic : Tema nr. 63
A. Bacil Gram + Epistaxisul şi tratamentul acestuia
B. Bacil Gram – Bibliografie asociata temei:
C. Spirochet Book Des ECN, editia in limba romana, redactor
Laurent Karila, 2011
D. Adenovirus
pag. 737-739
E. Virusul parainfluenza
(pag. 725) 1 Factorii implicaţi în aprecierea abundenţei
epistaxisului sunt:
*7 Angina lui Vincent este cauzată de:
A. vârsta pacientului
A. Bacil gram pozitiv B. durata sângerării
B. Corynebacterium C. voma
C. Spirochetă + Bacil gram negativ D. debitul sângerării
D. Streptococ E. ameţelile
E. Stafilococ (pag. 737)
(pag. 726)
2 În aprecierea abundenţei unui epistaxis este
*8 Mononucleoza infecţioasă prezintă: primordială investigarea:
A. Uneori o purpură a vălului palatin A. paloarei tegumentare
B. Rash cutanat B. hipotensiunii
C. Paralizie velopalatină C. temperaturii
D. Trismus D. tahicardiei
E. Erupţie veziculară E. tahipneei
(pag. 725) (pag. 737)

*9 Angina pseudomembranoasă se întâlneşte în: 3 Oprirea sângerării într-un epistaxis


presupune:
A. Difterie
B. Angina Vincent A. antialgice

C. Herpangină B. tamponament nazal anterior

D. Herpes C. tamponament nazal posterior

E. Zona nervului IX D. embolizarea vaselor incriminate


(pag. 722) E. antibioterapie
(pag. 737,738)
* 10 Amigdalectomia are indicaţie doar în cazul:
4 Examinările complementare în epistaxis
A. anginelor care se repetă (2/iarnă)
presupun:
B. anginelor care se repetă (1/an)
A. bilanţul coagulării
C. anginelor care se repetă (o dată/ 2 ani)
B. bilanţul factorilor de risc cardio-vascular
D. recidivei flegmonului periamigdalian
C. bilanţ de imagistică (CT, RMN)
E. recidivei anginelor veziculare
D. examinarea otică
(pag. 727)
E. bilanţ de imagistică (angiografie)
(pag. 738,739)

5 Rinitele şi sinuzitele pot acţiona în mod


sistemic cu:
A. purpura (febră tifoidă)
B. tahicardie
C. fragilitate capilară (gripă)
D. disfagie
E. dispnee inspiratorie
(pag. 739)

Pag. 90 din 130


*6 Care din afirmaţiile următoare nu este Tema nr. 64
specifică fibromului naso-faringian: Pacientul vârstnic: îmbătrânirea normală şi
A. tumoră bogat vascularizată particularităţi semiologice, psihologice, terapeutice
B. frecventă la adolescenţii de sex masculin Bibliografie asociata temei:
C. frecventă la adolescenţii de sex feminin Book Des ECN, editia in limba romana, redactor
Laurent Karila, 2011
D. are risc de sângerare crescut
pag. 776-778, 779-782
E. este o cauză locală de epistaxis
(pag. 739) 1 Care sunt formele de imbatranire?
A. Imbatranire datorita unui accident vascular
*7 Dintre cauzele epistaxisului enumerate mai cerebral
jos care nu este cauză generală:
B. Imbatranire obisnuita fara patologii evolutive, dar
A. HTA cu o limitare relativa a capacitatilor functionale
B. Anomalie vasculară Rendu Osler C. Imbatranire genetica
C. Rinosinuzită D. Imbatranire prematura
D. Hemofilie E. Imbatranire patologica cu patologii cronice
E. Insuficienţa hepatică evolutive provocand handicapuri si dependenta
care poate ajunge pana la dependenta totala
(pag. 739)
pentru toate activitatile vietii cotidiene
*8 Dintre cauzele epistaxisului enumerate mai (pag. 776)
jos care nu este cauză locală:
2 Care sunt consecintele imbatranirii functiei
A. Rinosinuzita renale?
B. Boala Rendu Osler A. Scaderea volumului rinichilor bilateral
C. Tumori nazale B. Scaderea volumului unui singur rinichi
D. Epistaxis esenţial C. Frecventa complicatiilor renale iatrogene
E. Traumatisme nazale D. Frecventa deshidratarilor extracelulare si
(pag. 739) toleranta scazuta la regimurile desodate

*9 Dintre cauzele epistaxisului enumerate mai E. Frecventa hipo- si hipernatremiei


jos care nu este cauză generală: (pag. 777)
A. Carenţa de vitamină K 3 Imbatranirea neurocognitiva determina:
B. Anomalie vasculară Rendu Osler A. dificultati in coordonarea miscarilor
C. Epistaxis esenţial B. diminuarea capacitatii de invatare
D. Hemofilie C. diminuarea timpului de reactie
E. Trombopenie D. cresterea timpului de reactie
(pag. 739) E. diminuarea atentiei
* 10 Din afirmaţiile următoare care nu corespunde (pag. 777)
pentru boala Rendu Osler:
4 Managementul pacientului varstnic trebuie
A. Este o angiomatoză autozomal dominantă sa cuprinda intotdeauna
B. Prezintă malformaţii vasculare mucoase A. anularea datelor anamnestice
C. Prezintă malformaţii vasculare extra-mucoase B. reconstituirea anamnezei
D. Nu este responsabilă de sângerări recidivante C. tratament de urgenta indiferent de tratamentele
E. Tratamentul său vizează utilizarea anticorpilor urmate anterior
anti VEGF D. cunoasterea tratamentului pacientului, fara a
(pag. 739) omite automedicatia
E. examen clinic complet, inclusiv tuseu rectal si
ECG
(pag. 780)

Pag. 91 din 130


5 Inainte de a face prescrierea medicamentelor *9 Imbatranirea vasculara are urmatoarele
in geriatrie trebuie realizate consecinte clinice
A. ierarhizarea patologiilor, tinandu-se cont de A. tendinta la hipotensiune ortostatica accentuata
parerea pacientului de anumite probleme venoase, deshidratare sau
B. tratamentul suferintei vasculare cerebrale medicament

C. se va determina daca pacientul poate sa isi B. aparitia anevrismelor aortice


administreze singur tratamentul C. edem pulmonar
D. se va determina calitatea venelor pentru D. pierderea acuitatii vizuale
administrarea perfuziilor
E. cefalee de tip migrenoid
E. se va estima nevoia de oxigenoterapie (pag. 777)
(pag. 781)
* 10 Imbatranirea sistemului musculare determina
*6 Care din urmatoarele reguli de management
A. neuropatii periferice
pentru un pacient varstnic bolnav sunt
valide? B. sarcopenia = scaderea masei musculare
A. prezenta aparaturii necesare neuroimagisticii C. crampe musculare
B. capacitatea si independenta de miscare a D. fracturi ale colului femural
pacientului pastrate E. rupturi musculare repetate
C. identificarea rapida a patologiilor acute si (pag. 777)
tratarea lor pentru a evita decompensarile in
cascada Tema nr. 65
D. evidentierea lipsei in antecedente a tulburarii Tulburări anxioase, tulburări fobice,tulburări obsesiv-
depresiva compulsive, tulburări convertive, starea de stres
E. evidentierea prezentei in antecedente a tulburarii posttraumatic şi tulburările de adaptare
depresiva Bibliografie asociata temei:
(pag. 780) Book Des ECN, editia in limba romana, redactor
Laurent Karila, 2011
*7 Riscul iatrogen este mai crescut la pag. 913-921 * fara denumiri comerciale ale
persoanele in varsta datorita medicamentelor
A. culorii identice a tabletelor unor medicamente
diferite 1 In cadrul tulburarilor anxioase sunt cuprinse:
B. medicatiei administrate sub forma de solutie A. tulburarea anxioasa provocata de o boala sau
orala substanta
C. nerespectararii regimului igieno-dietetic B. pierderea controlului emotional
D. modificari farmacocinetice datorate varstei C. agitatia psihomotorie
E. existentei hipertermiei D. fobia specifica
(pag. 781) E. starea de stres acut

*8 Imbatranirea aparatului respirator poate fi (pag. 913)


determina 2 Tulburarea anxioasa generalizata poate avea
A. algii intercostale bilaterale urmatoarea evolutie
B. algii intercostale unilaterale A. complicatii: tulburari depresive, tulburari de
panica, dependente, izolare sociala
C. cresterea suprafetei schimburilor gazoase
B. deterioararea cognitiva
D. lipsa posibilitatii de efectuare a schimburilor
gazoase C. remisii
E. diminuarea suprafetei schimburilor gazoase D. cronicizare
(pag. 777) E. frecvente accidente vasculare cerebrale
(pag. 914)

Pag. 92 din 130


3 Diagnosticul diferential pentru tulburarea *8 Tulburarea obsesiv-compulsiva poate avea
obsesiva compulsiva se va face cu: urmatoarele comorbiditati:
A. patologia organica A. episod maniacal acut
B. tulburari anxioase B. dementa Alzheimer
C. tulburari de dispozitie C. tulburari de limbaj
D. dementa vasculara D. schizofrenia
E. tulburarea deliranta E. tulburari de coordonare
(pag. 917) (pag. 917)

4 Care din urmatoarele argumente clinice sunt *9 Simptomele tulburarilor de conversie pot fi:
in favoarea diagnosticului de tulburare
A. hipoglicemii
conversiva?
B. hiperglicemii
A. traumatisme craniene in antecedentele recente
C. cresterea creatininei
B. doliu patologic
D. transpiratii profuze nocturne
C. bilant somatic perfect normal
E. manifestari acute: agitatia psihomotorie, sincope,
D. personalitate histrionica sau pasiv-dependenta indispozitie, lipotimie, crize pseudo-convulsive,
E. relatie comprehensiva intre tulburarile prezentate miscari anormale, tremuraturi
si contextul patologic (pag. 919)
(pag. 918)
* 10 Care din urmatoarele caracteristici sunt
5 Formele clinice ale tulburarii de adaptare specifice diagnosticului pentru tulburarea de
sunt: adaptare
A. tulburarea de adaptare cu dispozitie depresiva A. tulburare permanenta
B. tulburarea de adaptare cu anxietate B. tulburare paroxistica
C. tulburarea de adaptare antisociala C. tulburare tranzitorie
D. tulburarea de adaptare cu perturbari de conduita D. tulburare cu evolutie progredienta
E. tulburarea de adaptare mixta (depresiva, E. tulburare dependenta de factorii vasculari
anxioasa) cerebrali
(pag. 921) (pag. 920)

*6 Care din urmatoarele afirmatii privind Tema nr. 66


diagnosticul tulburarii de panica sunt Tulburari psihice in perioada sarcinii si tulburari ale
advarate? post-partumului
A. tulburarea este mai frecventa la femei Bibliografie asociata temei:
B. tulburarea este mai frecventa la barbati Book Des ECN, editia in limba romana, redactor
C. tulburarea este mai frecventa la batrani Laurent Karila, 2011
pag. 922-925 * fara denumiri comerciale ale
D. tulburarea este mai frecventa la copii medicamentelor
E. tulburarea este mai frecventa dupa accidentele
rutiere 1 Managementul tulburarilor psihice in
(pag. 913) perioada sarcinii si tulburari ale post-
partumului necesita:
*7 In managementul terapeutic al tulburarii
A. spitalizarea in caz de simptomatologie deliranta
anxioase generalizate trebuie urmarite tintele sau risc suicidar
urmatoare:
B. tratamente medicamentoase
A. tratarea bolilor somatice asociate
C. terapie electroconvulsivanta
B. tratarea crizei de angoasa acuta
D. interventie obstetricala in regim de urgenta
C. instituirea unui regim dietetic adecvat
E. scaderea riscului pentru diabet zaharat tip II
D. tratamentul contracturilor musculare
(pag. 923)
E. tratamentul tulburarilor de atentie
(pag. 915)

Pag. 93 din 130


2 Factorii de risc pentru depresia post-partum *7 Managementul tulburarilor psihice in
sunt: perioada sarcinii in primul trimestru va
respecta urmatoarele reguli:
A. antecedente psihiatrice de depresie
B. sarcini multiple A. utilizarea in doze mici a antipsihoticelor

C. evenimente stresante B. utilizarea in doze mici a antidepresivelor

D. complicatii obstetricale C. utilizarea benzodiazepinelor

E. statut socio-economic scazut D. utilizarea anticonvulsivantelor cu efect


timostabilizator
(pag. 924)
E. evitarea prescrierii medicamentelor psihotrope
3 Care din elementele diagnostice sunt (pag. 923)
valabile pentru post-partum blues sau
sindromul celei de-a treia zile sau baby blues *8 Tratamentul post-partum blues (sau
sindromul celei de-a treia zile sau baby
A. halucinatii auditive persistente
blues) impune:
B. delir de persecutie
A. terapie electroconvulsivanta
C. tuloburari de somn
B. utilizarea medicamentelor psihotrope injectabile
D. anxietate, neliniste
C. utilizarea medicamentelor psihotrope sub forma
E. astenie de solutie orala
(pag. 924) D. utilizarea medicamentelor psihotrope cu actiune
prelungita
4 Psihoza puerperala (bufeu delirant acut
confuzo-oniric) poate avea urmatoarea E. nu se administreaza tratament medicamentos
evolutie: (pag. 924)
A. favorabila *9 Melancolia deliranta post-partum are
B. recidive in post-partum in 50% din cazuri sau in urmatoarele riscuri:
cursul unei sarcini ulterioare la 30% din cazuri
A. risc de accident vascular cerebral
C. deteriorare cognitiva de tip demential
B. risc suicidar si de infanticid
D. tulburare de conversie
C. risc de infarct miocardic
E. poate evolua spre o tulburare bipolara sau
D. risc pentru dezvoltarea unei stari dementiale
schizofrenie
(pag. 925) E. risc neurodegenerativ
(pag. 925)
5 In faza prodromala a psihozei puerperale pot
aparea urmatoarele simptome: * 10 Negarea sarcinii are drept consecinta:
A. comportament bizar A. debutul schizofreniei
B. insomnie B. debutul tulburarii bipolare
C. manifestari depresive in cursul ultimelor C. debutul unei tulburari obsesivo-compulsive
saptamani de sarcina D. rata mare a mortalitatii perinatale
D. manifestari anxioase E. reactia de stres post-traumatic
E. deteriorare cognitiva (pag. 923)
(pag. 925)

*6 Depresia in perioada sarcinii se


caracterizeaza prin:
A. varsaturi matinale
B. varsaturi incoercibile
C. varsaturi postprandiale
D. varsaturi nocturne
E. intoleranta la fructoza
(pag. 922)

Pag. 94 din 130


Tema nr. 67 5 Dificultatile de calcul matematic afecteaza
Dezvoltarea psihomotorie a sugarului şi a copilului: urmatoarele componente:
aspecte normale şi patologice A. lingvistice (neintelegerea termenilor aritmetici si
Bibliografie asociata temei: conversia problemelor in simboluri aritmetice)
Book Des ECN, editia in limba romana, redactor B. perceptive (recunoasterea si intelegerea
Laurent Karila, 2011 simbolurilor)
pag. 995-1006 * fara denumiri comerciale ale C. motorie (incapacitatea de deplasare)
medicamentelor
D. atentionale (copierea cifrelor si a simbolurilor)
1 Dezvoltarea normala a copilului de la nastere E. cognitiva (deficit cognitiv sever)
pana la varsta de 6 ani evaluata prin (pag. 1001)
monitorizarea motricitatii se caracterizeaza
prin: *6 Managementul dificultatilor de calcul
matematic se va face prin:
A. merge singur la 12-18 luni
A. antidepresive
B. dobandirea progresiva a echilibrului perfect
B. antipsihotice
C. atonii musculare tranzitorii
C. reeducarea adaptata copilului
D. activitatea motrica este subordonata reflexelor
primare D. psihostimulente
E. motricitatea spontana arhaica asimetrica E. neuroprotectoare
(pag. 995) (pag. 1001)

2 Enurezisul se caracterizeaza prin: *7 Managementul dificultatilor de citire ale


copilului va beneficia de:
A. aparitia predominent la baieti
B. aparitia predominent la fete A. antidepresive

C. aparitie nocturna B. antipsihotice

D. evolutie defavorabila neurodegenerativa C. psihostimulente

E. evolutie favorabila spontana D. neuroprotective

(pag. 998) E. reeducare logopedica


(pag. 1001)
3 Tulburarile psihotice din perioada copilariei
*8 Autismul infantil Kanner este definit ca si:
A. autismul Kanner
B. sindromul Rett A. o tulburare neurodegenerativa precoce

C. sindromul Apserger B. o tulburare metabolica precoce

D. tulburarea psihotica bipolara cu debut precoce C. o tulburarea de dezvoltare precoce si severa


care afecteaza limbajul si abilitatile de
E. schizofrenia infantila cu debut precoce comunicare
(pag. 999) D. o tulburare post-traumatica
4 Autismul Kanner impune un diagnostic E. o tulburare de tip epileptic
diferential cu: (pag. 999)
A. manifestari epileptice
*9 Examinari complementare recomandate in
B. afectiuni metabolice diagnosticul diferential al autismului Kanner:
C. sindromul X fragil A. examen ORL adaptat varstei
D. surditate B. evaluarea perimetrului abdominal
E. traumatismele obstetricale severe C. evaluarea nivelului trigliceridelor
(pag. 999) D. evaluarea colesterolemiei
E. examen coproparazitologic
(pag. 999)

Pag. 95 din 130


* 10 Initierea precoce a relatiei parinti-copil se 15 Enurezisul la copil este caracterizat prin:
bazeaza pe urmatoarele nivele de
A. Mictiuni complete, involuntare la copilul de cel
interactiune:
putin 5 ani
A. interactiuni motorii B. Apare mai frecvent la fete
B. interactiuni operationale C. Apare in majoritatea cazurilor diurn
C. interactiuni competitionale D. Evolutie nefavorabila in lipsa tratamentului
D. interactiuni afective E. Forma primara e cea mai frecventa
E. interactiuni de tip managerial (pag. -)
(pag. 1000)
16 Care din urmatoarele afirmatii in legatura cu
* 11 Care dintre urmatoarele este caracteristica encoprezisul la copil sunt adevarate?
motricitatii normale la nastere? A. Apare mai frecvent la baieti
A. Motricitate spontana asimetrica B. Sunt mai ales forme secundare
B. Ridicarea capului atanci cand este intins pe C. Nu se asociaza cu constipatia
spate
D. Mai putin grav decat enurezisul
C. Tonus axial al membrelor este crescut
E. Dispare adesea in timpul separarii de familie
D. Hipotonie generalizata
(pag. -)
E. Lipsa reflexelor primare
(pag. -) * 17 Care este perioada caracteristica de aparitie
a mericismului?
* 12 Varsta la care sugarul poate sta in pozitie
A. La nastere
sezanda cu sprijin este de obicei:
B. In primele 3 luni de viata
A. 3 luni
C. Intre 3 si 12 luni
B. o luna
D. Intre 1 si 2 ani
C. 6-7 luni
E. Dupa varsta de 5 ani
D. 9 luni
(pag. -)
E. 1 an
(pag. -) 18 Sindromul Münchhausen prin transfer la
copil se caracterizeaza prin:
* 13 Inceputul controlului sfincterian la copil are
A. Patologia copilului provocata sau simulata de un
loc in jurul varstei de:
parinte
A. 4 luni B. Examene medicale foarte rare prin opozitia
B. 8 luni parintilor
C. 1 an C. Negarea cauzei simptomelor de catre parinti
D. 2 ani D. Regresia simptomelor atunci cand copilul este
despartit de parinte
E. 4 ani
E. Riscuri de sechele fizice, psihologice
(pag. -)
(pag. -)
14 Intarzierea achizitiilor corespunzatoare
varstei la copil se poate manifesta prin: 19 Care din urmatoarele afirmatii in legatura cu
dezvoltarea normala a vorbirii la copil sunt
A. dupa 5 luni nu-si poate sustine capul adevarate?
B. la o luna hipertonia membrelor
A. La sfarsitul celei de-a doua luni gangureste
C. dupa 10 luni nu poate sta in sezut
B. La 4 luni: lalalizare
D. dupa 6 luni persistenta reflexelor arhaice
C. La 6 luni articuleaza primele silabe
E. dupa 3 luni nu poate sta in sezut
D. La 7 luni pronunta primele cuvinte
(pag. -)
E. In jurul varstei de 3 ani construieste propozitii
corecte
(pag. -)

Pag. 96 din 130


* 20 Encoprezisul se caracterizeaza prin defecatie *4 Care din urmatoarele afirmatii referitoare la
involuntara in absenta unor afectiuni alimentatia sugarului pana la varsta de 4 – 6
organice la copilul de cel putin: luni este adevarata:
A. 6 luni A. este exclusiv lactata
B. 1 an B. se recomanda diversificarea precoce
C. 2 ani C. formulele de lapte praf sunt obligatorii
D. 4 ani D. se face dupa un program strict al meselor
E. 6 ani E. nu este necesar nici un supliment vitaminic
(pag. -) (pag. -)

Tema nr. 68 *5 Laptele hipoalergenic are urmatoarea


Nutriţie şi alimentaţie pediatrică particularitate:

Bibliografie asociata temei: A. Nu se utilizeaza in caz de atopie familiala


Book Des ECN, editia in limba romana, redactor B. Este obtinut prin hidroliza partiala a proteinelor
Laurent Karila, 2011 din laptele de vaca
pag. 1028-1030 * fara denumiri comerciale ale C. Este obtinut prin hidroliza totala a proteinelor din
medicamentelor laptele de vaca
D. Se utilizeaza in caz de intoleranta la lactoza
1 Principiile alaptarii naturale sunt:
E. Se utilizeaza in realimentare dupa
A. precoce, din primele ore de viata gastroenterocolita acuta
B. la cererea sugarului (pag. -)
C. de 2-3 ori pe zi
6 Laptele de soia:
D. durata de sub 20 de minute a suptului, din
fiecare san A. Nu se poate folosi la sugar
E. dupa fiecare masa se poate suplimenta cu o B. este alimentul de electie in primele luni de viata
formula de lapte adaptata varstei C. Prezinta risc de alergii incrucişate
(pag. -) D. Este o alternativa posibila in caz de alergie la
proteinele din laptele de vaca (PLV)
2 Printre beneficiile alaptarii materne se pot
mentiona: E. Are repercursiuni asupra dezvoltarii organelor
genitale (fitoestrogeni)
A. Contine acizi graşi esentiali indispensabili pentru
maturizarea cerebrala (pag. -)

B. Nu contine Ig A *7 Referitor la diversificare este adevarata


C. Nu contine proteine alergizante (lactoglobulina afirmatia:
din laptele de vaca) A. Numarul de mese creste progresiv
D. Avantaje psihoafective in relatia mama-copil B. Cerealele cu gluten se introduc la 4 luni
E. Compozitie perfect adaptata nevoilor copilului C. Introducerea proteinelor animale de la 5 luni
(pag. -) D. Nu se incepe inainte de 4 luni
*3 Care e formula de lapte utilizata in caz de E. Introducerea fructelor fierte de la 3 luni
atopie familiala? (pag. -)
A. lapte praf integral
8 Perioada de alimentatie de la 1 la 3 ani are
B. formule de start urmatoarele caracteristici:
C. formule de soia A. Este necesara mentinerea aportului lactat de
D. formule fara lactoza 500 ml/zi
E. formule hipoalergenice B. Apa este singura bautura recomandata
(pag. -) C. Şase mese principale pe zi
D. Se pot folosi bauturi carbogazoase
E. Se va opri diversificarea
(pag. -)

Pag. 97 din 130


*9 Care dintre urmatoarele este contraindicatie 4 Managementul febrei acute la copilul cu
exceptionala pentru laptele matern: varsta peste 3 ani implica urmatoarele:
A. Galactozemie A. Masurarea temperaturii auricular sau axilar
B. Ragade mamare B. Corticoterapie
C. Diabet zaharat C. Cautarea semnelor de gravitate
D. Gastroenterocolita acuta D. Cautarea unui punct de plecare
E. Intoleranta la gluten E. Tripla antibioterapie de prima intentie
(pag. -) (pag. -)

10 Perioada de tranzitie si diversificare al 5 Orientarea diagnostica in functie de


sugarului se caracterizeaza prin: examenul clinic in meningita la copilul cu
febra acuta este sustinuta de:
A. dureaza de la 2 la 4 luni de viata
B. alimentatia lactata ramane necesara: mimim A. Fontanela deprimata
500ml/zi B. Tulburari de conştienta
C. se trece la formulele de start C. Fontanela bombata
D. scaderea progresiva a numarului de mese D. Redoare de ceafa
E. continua suplimentarea cu vitamina D E. Stare de rau epileptic
(pag. -) (pag. -)

Tema nr. 69 6 Orientarea diagnostica in functie de


Febra la copil examenul clinic in infectia urinara la copilul
cu varsta peste 3 luni cu febra acuta se
Bibliografie asociata temei: bazeaza pe:
Book Des ECN, editia in limba romana, redactor
A. Apetit crescut
Laurent Karila, 2011
pag. 1039-1041 B. Iritabilitate
C. Urina tulbure sau hematurie
1 Care sunt germenii cei mai frecventi in cazul
infectiilor maternofetale la copilul sub varsta D. Polakiurie
sub 3 luni cu febra acuta? E. Disurie
A. Streptococ grup B (pag. -)
B. E. Coli *7 La copilul cu febra acuta, purpura orienteaza
C. Enterococi diagnosticul catre:
D. Streptococ β hemolitic de grup A A. encefalita
E. Listeria monocytogenes B. meningococemie
(pag. -) C. hemofilie

2 Managementul tipic al febrei acute la copilul D. infectie urinara


cu varsta sub 6 saptamani presupune: E. artrita
A. Urmarire la domiciliu (pag. -)
B. Hemograma, PCR, examen de urina *8 Care este tratamentul de prima intentie al
C. Administrarea de betalactamine per os febrei acute la copil?
D. Punctie lombara A. Aspirina
E. Hemocultura, radiografia toracelui B. Ibuprofen
(pag. -) C. Paracetamol

*3 Care este cea mai frecventa cauza de febra D. Diclofenac


acuta la copilul cu varsta intre 3 şi 36 luni? E. Ampicilina
A. Endocardita (pag. -)
B. Infectie ORL sau respiratorie
C. Pielonefrita
D. Meningita
E. Boala diareica acuta
(pag. -)
Pag. 98 din 130
*9 La copilul cu febra acuta, conjunctivita *3 Diagnosticul de deshidratare acuta la copil
aseptica, enantem, edem al extremitatilor, se face in principal prin urmatoarea
eruptie polimorfa si adenopatie, cel mai interventie:
probabil diagnostic este:
A. anamneza
A. Septicemie B. identificarea unui teren predispozant
B. Sindrom nefrotic impur C. examenul clinic
C. Meningita D. coprocultura
D. SIDA E. ionograma sanguina şi urinara
E. Boala Kawasaki (pag. -)
(pag. -)
*4 Varsaturile sugarului cu agravare progresiva,
* 10 Antibioticul de prima intentie in tratamentul aparute dupa un interval liber si cu pastrarea
febrei acute la copilul intre 3 si 36 de luni aperitului sunt caracteristice:
este:
A. alergiei alimentare
A. Macrolid B. refluxului gastroesofagian
B. Gentamicina C. hipertensiunii intracraniene
C. Flourochinolona D. epilepsiei
D. Cefalosporina de generatia a III-a E. stenozei pilorice
E. Biseptol (pag. -)
(pag. -)
*5 Stenoza pilorica la sugar se manifesta dupa
Tema nr. 70 un interval liber de:
Diareea acută la copil, Vărsăturile la sugar şi copil A. 2-3 ore
(şi tratament)
B. 2-3 zile
Bibliografie asociata temei:
C. 7 zile
Book Des ECN, editia in limba romana, redactor
Laurent Karila, 2011 D. 2-8 saptamani
pag. 1049-1051, 1062-1064 * fara denumiri E. 6-8 luni
comerciale ale medicamentelor (pag. -)

*1 Care este cea mai frecventa cauza de diaree 6 Deshidratarea extracelulara la copil se
acuta la copil? caracterizeaza prin:
A. Bacteriana A. Scadere importanta in greutate
B. Virala B. Pliu cutanat persistent
C. Parazitara C. Hipotonia globilor oculari
D. Greşeli de alimentatie D. Colaps
E. Fungica E. Sete vie
(pag. -) (pag. -)

*2 Care este microorganismul implicat cel mai 7 Managementul deshidratarii acute la copil
frecvent in diareea acuta la copil? implica:
A. Salmonella A. Urgenta terapeutica in cazul deshidratarii peste
B. E. coli patogen 10%

C. Adenovirusul B. Internare obligatorie

D. Rotavirusul C. Rehidratare intravenoasa chiar expansiune


volemica in caz de hipovolemie
E. Giardia lamblia
D. Solutii de rehidratare orala in cantitati mici in
(pag. -) orice tip de deshidratare
E. Rehidratare intravenoasa in cazul eşecului
rehidratarii pe cale orala in deshidratarea 5-10%.
(pag. -)

Pag. 99 din 130


8 Criteriile de spitalizare in diareea acuta la 3 Tabloul clinic in convulsiile simple ale
copil sunt: copilului cuprinde:
A. Varsta sub 3 luni A. Criza tonico-clonica generalizata
B. Necesitatea rehidratarii orale B. Hipotermie
C. Deshidratare peste 10% C. Durata sub 15min
D. Intoleranta digestiva totala D. Sugar cu dezvoltare psiho-motorie normala
E. Prematuritate E. Sugar cu deficit neurologic
(pag. -) (pag. -)

9 Atitudinea terapeutica in diareea acuta la 4 Pentru encefalita post-infectioasa la copil


copil implica urmatoarele: pledeaza:
A. Realimentare precoce A. Varsta peste 5 ani
B. Antibioticoterapie in fiecare caz de diaree acuta B. Anomalii neurologice severe focale
C. Antisecretorii (racecadotril) C. Debut progresiv
D. Lapte delactozat in caz de teren fragil sau diaree D. CT cu hipodensitati focale
persistenta E. Febra moderata
E. Realimentare cat mai tardiva (pag. -)
(pag. -)
5 Tratamentul unei convulsii simple la copil
10 Varsaturile cronice la sugar şi copil mic au cuprinde:
urmatoarele cauze digestive:
A. Antipiretice
A. Stenoza pilorica
B. Plasarea copilului in pozitie laterala de siguranta
B. Apendicita acuta C. Antibioterapie, la nevoie
C. Alergia alimentara
D. Bronhodilatatoare
D. Imperforatia anala
E. Antitusive
E. Refluxul gastro-esofagian (pag. -)
(pag. -)
*6 Care este situatia in care RMN cerebrala este
Tema nr. 71 utila in diagnosticul copilului cu epilepsie?
Convulsiile şi epilepsia la copil A. Numai la sugar
Bibliografie asociata temei: B. Numai inainte de interventia chirurgicala
Book Des ECN, editia in limba romana, redactor C. In epilepsiile generalizate
Laurent Karila, 2011
D. Pentru decelarea unei origini focale de epilepsie
pag. 1079-1085 * fara denumiri comerciale ale
partiala sau simptomatica
medicamentelor
E. Numai in cazurile in care se efectueaza RMN
1 In encefalita herpetica la copil, urmatoarele functionala.
afirmatii sunt adevarate: (pag. -)
A. Varsta de debut este sub 1 an
*7 Forma de epilepsie a copilului cu EEG cu
B. Alterarea constientei este progresiva aspect de varf-unda de 3 Hz si prognostic
C. Apar convulsii intarziate la 48-72h bun este:
D. Debut brusc A. Epilepsia grand mal
E. Apare in afebrilitate B. Epilepsia petit mal
(pag. -) C. Sindromul West
D. Sindromul Dravet
2 Convulsiile simple la copil sunt caracterizate
de: E. Sindromul Lenox-Gastaut
A. Asociere cu febra (pag. -)
B. Antecedente neurologice
C. Durata peste 30 minute
D. Aparitie la sugar sau la copilul mic (3 luni-5ani)
E. Fara semne de infectie intracraniana
(pag. -)
Pag. 100 din 130
*8 In epilepsia copilului, triada : spasme in *2 In anginele si faringitele copilului, etiologia
flexie, hipsaritmie la EEG si regresia virala se manifesta mai degraba prin:
achizitiilor este caracteristica:
A. simpla congestie eritematoasa a amigdalelor
A. Sindromului West B. aspect eritemato-pultaceu
B. Sindromului Dravet C. abces retrofaringian
C. Sindromului Doose D. angina pseudomembranoasa
D. Epilepsiei grand mal E. angina veziculoasa
E. Epilepsiei petit mal (pag. -)
(pag. -)
*3 Strategia antibiotica in pleuropneumopatiile
*9 La copilul cu epilepsie, anomaliile bifrontale la copil se face prin urmatoarea asociere:
aparute la EEG sunt caracteristice
A. ampicilina plus gentamicina
sindromului:
B. cefalosporine de generatia III asociate cu
A. West vancomicina sau rifampicina
B. Dravet C. penicilina plus doxiciclina
C. Lennox-Gastaut D. cefalosporine plus gentamicina
D. Doose E. penicilina plus flourochinolone
E. Down (pag. -)
(pag. -)
*4 In diagnosticul pneumoniei la copil,
* 10 In epilepsia grand mal la copil, crizele determinarea PCR in secretiile nazale este
epileptice au urmatoarea caracteristica: utila in identificarea urmatorului germen:
A. Sunt atone A. Streptococ betahemolitic
B. Apar mai ales la sugar B. E. coli
C. Sunt localizate pe un hemicorp C. Proteus
D. Prognosticul este infaust D. Mycoplasma
E. Sunt tonico-clonice generalizate E. Chlamydia.
(pag. -) (pag. -)

Tema nr. 72 *5 Tratamentul bronşitei acute la sugar trebuie


sa cuprinda urmatoarea interventie:
Infecţii ale tractului respirator la copil (Angine şi
faringite, Infecţii bronho-pulmonare la sugar şi copil) A. obligatoriu antibioterapie
Bibliografie asociata temei: B. perfuzii endovenoase cu anticolinergice
Book Des ECN, editia in limba romana, redactor C. antibioterapie in caz de febra persistenta
Laurent Karila, 2011
D. badijonaj lingual cu solutii de Nistatin
pag. 1068-1069, 1099-1101 * fara denumiri
comerciale ale medicamentelor E. efort fizic sustinut
(pag. -)
*1 Care din urmatoarele afirmatii legate de
rinofaringita acuta este adevarata: 6 Tabloul clinic al rinofaringitei acute
presupune:
A. Aproape exclusiv de origine bacteriana
B. Afecteaza mai ales copiii mici A. rinoree anterioara/posterioara

C. Evolutie prelungita 2-3 saptamani B. tuse iritativa

D. Necesita obligatoriu antibioterapie C. raluri bronşice

E. Este rara la copii D. modificari Rx

(pag. -) E. obstructie nazala


(pag. -)

Pag. 101 din 130


7 Care din urmatoarele afirmatii despre Tema nr. 73
anginele acute sunt adevarate? Alergii respiratorii la copii: Rinita alergica, astmul la
A. anginele sunt afectiuni inflamatorii ale copil
amigdalelor palatine Bibliografie asociata temei:
B. cel mai adesea sunt virale Book Des ECN, editia in limba romana, redactor
C. formele streptococice sunt caracteristice sugarilor Laurent Karila, 2011
pag. 1092-1094, 1095-1098 * fara denumiri
D. febra este adesea ridicata
comerciale ale medicamentelor
E. tratamentul antibiotic se foloseşte de rutina
(pag. -) 1 Care sunt examinarile complementare
necesare in astmul bronsic la copil?
8 Urmatoarele sunt caracteristice pentru
A. Ancheta alergologica
bronşiolitele acute ale sugarului:
B. Prick test cutanat
A. varsta sub 2 ani
C. IgE serice totale la copilul sub 36 de luni
B. apar in epidemii vara
D. Testare alergeni alimentari
C. sunt frecvent virale
E. Testare alergeni respiratori
D. examinarile complementare sunt obligatorii
(pag. -)
E. pot fi precedate de rinofaringita acuta
(pag. -) 2 Factorii de risc endogeni pentru alergiile
respiratorii la copil sunt:
9 Care sunt indicatiile pentru efectuarea
A. Teren genetic
radiografiei toracice in bronşita acuta?
B. Factori emotionali
A. febra inexplicabila, mai ales la sugar
C. Infectiile respiratorii
B. etiologia virala
D. Poluarea atmosferica
C. tuse persistenta insotita de tahipnee
E. Obezitatea
D. pneumonii recidivante
(pag. -)
E. suspiciune de inhalare de corp strain
(pag. -) 3 Care din urmatoarele sunt semne de atopie
personala utile in diagnosticul astmului
10 Tratamentul in pneumoniile acute presupune: alergic?
A. antibioterapie de urgenta şi de prima intentie A. Psoriazis
B. spitalizarea cazurilor grave B. Eczema
C. tratament cu penicilina pe o durata de 5 zile la C. Rinita atrofica
cei cu varsta mai mica de 3 ani
D. Rinita sezoniera
D. cefalosporine daca tabloul clinic evoca
E. Alergie alimentara
mycoplasma
(pag. -)
E. evaluare la 48-72 de ore in toate cazurile
(pag. -) *4 Care dintre urmatoarele reprezinta
tratamentul de fond folosit in astmul
intermitent?
A. Corticoizii inhalatori
B. Corticoizii per os
C. Inhibitorii de leucotriene
D. Beta 2 adrenoimimetice cu durata lunga de
actiune
E. Astmul intermitent nu are tratament de fond
(pag. -)

Pag. 102 din 130


*5 In astmul alergic sever se poate folosi ca 10 Dupa GINA 2006, astmul bronsic controlat se
tratament asociat urmatoarea substanta: poate defini prin urmatorii parametrii:
A. Omalizumab A. Fara limitarea activitatilor
B. Propranolol B. Fara simptome nocturne
C. Histamina C. Valoarea VEMS/PEF sub 80%
D. Aspirina D. Fara exacerbari
E. Ranitidina E. Folosirea zilnica a beta 2 agonistilor
(pag. -) (pag. -)

6 Tratamentul rinitei alergice intermitente Tema nr. 74


consta in: Infecţiile urinare la copil. Leucocituria
A. Evictiune alergenica Bibliografie asociata temei:
B. Corticoizi inhalator Book Des ECN, editia in limba romana, redactor
C. Histamina Laurent Karila, 2011
pag. 1105-1106 * fara denumiri comerciale ale
D. Cromone
medicamentelor
E. Decongestionante
(pag. -) 1 In pielonefrita acuta la copil, bandeleta
urinara este utila pentru determinarea:
*7 Care este criteriul de gravitate care poate
A. Cilindrilor leucocitari
anunta stop respirator iminent in astmul
bronsic la copil? B. Leucocituriei
A. Dispneea la effort C. Nitritilor
B. Bradicardia D. Hematuriei
C. Ralurile sibilante E. Cristalelor birefringente
D. PEF peste 80% (pag. -)

E. SaO2 90-95% *2 Care este examinarea complementara de


(pag. -) prima intentie in pielonefrita la copil?
A. Hemograma
*8 Care este tratamentul de fond de prima
intentie in nivelul 1 din astmul bronsic dupa B. Urocultura
GINA 2006? C. Bandeleta urinara
A. Corticoizi inhalatori in doze slabe D. Ecografia renala
B. Antileucotriene E. Hemocultura
C. Betaadrenergice cu actiune prelungita (pag. -)
D. Teofiline retard *3 Entitatea patologica depistata prin
E. Anti IgE cistografie retrograda efectuata la copil cu
(pag. -) pielonefrita este:
A. Insuficienta renala cronica
*9 Care este tramentul crizei de astm bronsic
dupa GINA2006? B. Refluxul vezico-ureteral
A. Corticostreroizi in doze slabe C. Ptoza renala
B. Histamina D. Malformatiile vasculare renale
C. Adrenalina E. Cicatricile renale
D. Antileucotriene (pag. -)

E. Beta 2 adrenergice cu actiune rapida la nevoie


(pag. -)

Pag. 103 din 130


4 Asocierea unui aminoglicozid la tratamentul *8 Care din urmatoarele afirmatii referitoare la
de atac al pielonefritei copilului este diagnosticul de pielonefrita la copil este
necesara in caz de: adevarata?
A. Uropatie malformativa cunoscuta A. Se confirma in caz de leucociturie de peste
B. Varsta < 3 luni 104/ml chiar fara bacteriurie
B. Se confirma in caz de prezenta a unei bacteriurii
C. Sindrom septicemic
unice peste 105/ml
D. Imunodepresie
C. Se confirma in caz de leucociturie de peste
E. Urocultura pozitiva pentru enterococ 104/ml şi prezenta unei bacteriurii unice peste
(pag. -) 105/ml
D. Se confirma in caz de hematurie microscopica
*5 Referitor la tratamentul antibiotic al
pielonefritei copilului, care din afirmatii este E. Se confirma prin prezenta cristalelor birefringente
adevarata? (pag. -)
A. Dureaza 10 zile in total 9 Tratamentul cistitei copilului consta in:
B. Este obligatoriu intravenos
A. Ceftriaxon (i.v sau i.m)
C. Tratamentul de atac dureaza 12 ore
B. Cotrimoxazol sau cefixim pentru 3-5 zile
D. Aminoglicozidele sunt contraindicate
C. Nu este necesara prevenirea recidivelor
E. Se efectueaza intotdeauna in ambulator D. tratamentul diareei, frecvent asociate in aceste
(pag. -) cazuri
*6 Care din urmatoarele afirmatii in legatura cu E. tratamentul instabilitatii vezicale.
diagnosticul cistitei la fetite este adevarata? (pag. -)
A. Bandeleta urinara este negativa (leucociturie, 10 Principiile terapeutice in pielonefrita
nitriti) copilului includ:
B. Prezenta febrei
A. spitalizare daca copilul are varsta sub 3 luni
C. Prezenta durerii lombare
B. spitalizare daca sunt semne de infectie severa
D. Prezenta sindromului inflamator C. evitarea initiala a tratamentului antibiotic
E. In sumarul de urina se identifica flora bacteriana
D. tratament antibiotic timp de 3-4 saptamani
unica
E. initial tratament antibiotic intravenos, apoi oral
(pag. -)
(pag. -)
7 Tratamentul oral de intretinere in pielonefrita
copilului consta in: Tema nr. 75
A. Cotrimoxazol sau Cefixim, in functie de Boli de nutriţie la copil (Diabetul zaharat tip 1 şi 2 la
rezultatele antibiogramei copil, Obezitatea la copil)
B. In caz de rezistenta la alte familii de antibiotice, Bibliografie asociata temei:
ciprofloxacina poate fi luata in considerare in Book Des ECN, editia in limba romana, redactor
prepubertate Laurent Karila, 2011
C. Reevaluare clinica dupa 10 -14 zile pag. 1109-1112, 1121-1122 * fara denumiri
comerciale ale medicamentelor
D. Prevenirea si tratamentul costipatiei
E. Cotrimoxazolul se poate administra de la nastere 1 Sindromul cardinal in diabetul zaharat la
(pag. -) copil este compus din:
A. Poliurie osmotica
B. Transpiratii profuze
C. Polidipsie
D. Polifagie
E. Scadere in greutate
(pag. -)

Pag. 104 din 130


2 Cetoacidoza diabetica la copil se manifesta 7 Diagnosticul sigur de diabet zaharat la copil
prin: se poate pune in urmatoarele situatii:
A. Bradipnee A. Glicozurie plus cetonurie
B. Deshidratare globala B. Glicemie plasmatica peste 11 mmol/l
C. Greturi, varsaturi C. 2 glicemii a jeun peste 7 mmol/l
D. Dureri abdominale D. crestere in greutate
E. Astenie E. apetit capricios
(pag. -) (pag. -)

3 Semnele clinice neuroglicopenice in *8 In diabetul la copil, determinarea anticorpilor


hipoglicemia din diabetul zaharat la copil anticelule insulare si anti GAD este utila in
sunt: urmatoarea situatie:
A. Hipotonia generalizata A. Calcularea riscului de hipoglicemie
B. Tulburarile de vedere B. Diagnosticul pozitiv al bolii
C. Tuburarile de echilibru C. Confirmarea caracterului autoimun
D. Tulburarile de constienta D. Confirmarea cetoacidozei
E. Convulsiile, astenia E. Confirmarea caracterului insulinodependent
(pag. -) (pag. -)

*4 Obezitatea de gradul I la copil se defineste *9 In managementul diabetului la copil, Hb


prin urmatorul element: glicata trebuie sa aiba o valoare:
A. IMC peste percentila 50 A. Sub 3%
B. IMC peste percentila 75 B. Sub 5%
C. IMC peste percentila 97 C. Sub 7,5%
D. IMC peste 30 kg/mp suprafata corporala D. Peste 9%
E. IMC peste 40 kg/mp suprafata corporala E. Peste 10%
(pag. -) (pag. -)

5 Tratamentul medicamentos sau chirurgical * 10 Care este masura terapeutica ce se poate


in obezitatea la copil este indicat la: aplica la un copil autonom cu diabet zaharat?
A. Copiii cu obezitate comuna A. Insulina cu actiune rapida inainte de mese
B. Adolescent cu obezitate morbida B. Insulina cu actiune lenta dimineata
C. Esecul masurilor dietetice C. Gustare la ora 10
D. Esecul masurilor de suport psihologic D. Maxim 2 injectii pe zi
E. Esecul masurilor de adaptare a activitatii fizice E. 2/3 din doza totala dimineata si 1/3 seara
(pag. -) (pag. -)

*6 Care din afirmatiile urmatoare, in legatura cu Tema nr. 76


managementul chirurgical al obezitatii la Evaluarea şi îngrijirea nou-născutului la termen
copil este adevarata?
Bibliografie asociata temei:
A. Este o masura de prima intentie
Book Des ECN, editia in limba romana, redactor
B. Se indica mai ales in obezitatea comuna Laurent Karila, 2011
C. Este contraindicat la adolescentul mare pag. 1031-1034 * fara denumiri comerciale ale
medicamentelor
D. Consta in aplicarea unui inel gastric
E. Este contraindicat in obezitatea morbida 1 Scorul Apgar utilizat pentru evaluarea nou
(pag. -) nascutului este constituit din:
A. Activitate cardiaca
B. Respiratie
C. Culoare
D. Tonus, reactivitate
E. Eliminarea meconiului
(pag. -)
Pag. 105 din 130
2 In evaluarea nou nascutului, serologia 7 Scorul Silverman din sindromul de detresa
materna in sarcina trebuie sa cuprinda: respiratorie a nou nascutului este compus
din:
A. HIV
B. HBV, HCV A. Bataia aripilor nazale

C. Rujeola B. Balans toraco-abdominal

D. Rubeola C. Tiraj intercostal, geamat respirator

E. Sifilis, toxoplasmoza D. Deprimare xifoidiana


(pag. -) E. Cianoza generalizata
(pag. -)
3 Ingrijirea nou nascutului la termen implica
urmatoarele interventii care se efectueaza *8 La nou nascutul normal primul meconiu
sistematic la nastere: trebuie emis la:
A. Administrarea de vitamina D A. O ora de la nastere
B. Administrarea de vitamina K B. 6 ore de la nastere
C. Administrarea de colir oftalmologic antibiotic C. 24 de ore de la nastere
D. Administrarea de cefalosporine iv in primele 24 D. 48 de ore de la nastere
de ore de viata E. 72 de ore de la nastere
E. Punerea la san sau primul biberon in primele ore (pag. -)
dupa nastere
(pag. -) *9 Metoda recomandata pentru depistarea
precoce a icterului neonatal este:
*4 Varsta gestationala in cazul prematuritatii
este: A. Dozarea sangvina a bilirubinei
B. Bilirubinometria transcutanata
A. Sub 40 de saptamani
C. Examenul sumar de urina
B. Peste 42 de saptamani
D. Determinarea grupului sangvin al mamei si
C. Sub 37 de saptamani
copilului
D. Sub 30 de saptamani
E. Dozarea bilirubinei materne in ultimul trimestru
E. Sub 28 de saptamani de sarcina
(pag. -) (pag. -)

*5 Care este afirmatia ce defineste retardul de * 10 Care dintre urmatoarele afirmatii in legatura
crestere intrauterina a nou nascutului? cu curba ponderala a nou-nascutului este
adevarata?
A. Greutate la nastere sub 2500 g
B. Greutate la nastere sub 2000 g A. Nou nascutul nu trebuie sa piarda mai mult de
25% din greutatea de la nastere
C. Greutate la nastere sub 1500 g
B. Nou nascutul nu trebuie sa piarda mai mult de
D. Greutatea la nastere raportata la varsta 10% din greutatea de la nastere
gestationala este inferioara percentilei 10
C. Revenirea la greutatea de la nastere trebuie sa
E. Greutatea la nastere raportata la varsta se faca in prima luna de viata
gestationala este inferioara percentilei 50
D. Nou nascutii la termen nu au pierdere ponderala
(pag. -) dupa nastere
6 Care sunt cei mai frecventi germeni implicati E. Formulele lactate permit cresterea ponderala mai
in infectia materno-fetala? rapida comparativ cu alimentatia cu lapte matern
A. Streptococ B (pag. -)

B. E. Coli
C. Pneumococ
D. Enterococi
E. Listeria monocitogenes
(pag. -)

Pag. 106 din 130


Tema nr. 77 *5 Afirmaţia corectă privind afectarea renală în
Lupusul eritematos diseminat. Sindromul lupus
antifosfolipidic (SAFL) A. afectarea renală este de tip interstiţial
Bibliografie asociata temei: B. nu necesită monitorizarea creatininei
Book Des ECN, editia in limba romana, redactor C. necesită obligatoriu examen citobacteriologic
Laurent Karila, 2011 pentru diagnostic
pag. 1138-1142
D. sindromul nefrotic impur impune puncţie biopsie
renală
*1 Diagnosticul în lupusul eritematos discoid
presupune existenţa E. nu determină glomerulonefrită proliferativă
A. rash-ului malar (pag. 1139)
B. pericarditei constrictive 6 Următoarele criterii definesc sindromul
C. artritelor erozive antifosfolipidic

D. ulceraţiilor mucoasei gastrice A. cel puţin un episod de tromboză venoasă


profundă sau arterială
E. ac anti-RNP
B. cel puţin o moarte fetală după săptămâna 10 de
(pag. 1138) gestaţie
*2 Lupusul eritematos diseminat C. cel puţin o naştere prematură înainte de
săptămâna a 34-a de sarcină
A. este o boală autoimună organ-specifică
D. absenţa anticoagulantului lupic la două
B. asociază autoimunitate îndreptată împotriva
determinări succesive
componentelor citoplasmatice
E. detectarea ac anticardiolipină la două determinări
C. afectează predilect femeile post-menopauză
succesive la valori < 40U
D. prezintă antigenitate a componentelor nucleare (pag. 1142)
celulare
E. survine doar la vârstă tânără (15 – 45 ani) 7 Care din următoarele afirmaţii referitoare la
autoimunitate din lupusul eritematos
(pag. 1138)
diseminat sunt corecte
*3 Afectarea hematologică în lupusul A. AAN sunt prezenţi în 95% din LED dar înalt
eritematos diseminat este reprezentată în specifici + +
principal de
B. ac anti-ADN reflectă activitatea bolii
A. limfocitoză constatată la 2 determinări
C. ac anti-Sm sunt specifici LED
B. trombopenie > 100G/L
D. C3 şi ac anti-ADN sunt factori predictivi ai
C. leucopenie < 4G/L constatată la 2 determinări puseelor în LED
D. anemie feriprivă E. Anticardiolipinele IgG şi IgM sunt specifice LED
E. trombocitoză (pag. 1140)
(pag. 1138) 8 Următoarele explorări nu sunt necesare în
*4 Identificaţi manifestarea clinică posibilă în bilanţul unui LED
LED A. dozarea ac anti-ADN
A. fotosensibilitate normală B. deteminarea ac anti-limfocitari
B. alopecie C. determinarea C1q
C. endocardită septică D. testul Coombs granular
D. artrite erozive E. dozarea anti-ECT
E. ulceraţii genitale (pag. 1140)
(pag. 1138) 9 Asocierea LED – sarcină presupune
A. risc de declanşare a unui puseu evolutiv al bolii
B. risc de eclampsie
C. rar lupus neonatal
D. boală stabilizată de cel puţin 3 luni
E. risc de retard de creştere a fătului
(pag. 1141)

Pag. 107 din 130


10 Bilanţul iniţial în LED presupune *4 Precizaţi afirmaţia corectă privind ac anti-
ENA
A. ac anti-AAN, anti-ECT, ANCA
B. factor reumatoid A. ac anti-histone sunt specifici pentru lupusul
eritematos sistemic
C. test Coombs globular
B. ac anti-SCL-70 sunt asociaţi sclerodermiei
D. ionogramă sanguină limitate
E. puncţie biopsie renală C. ac anti-RNP sunt identificaţi în lupusul indus
(pag. 1140) medicamentos
D. ac anti-SSA şi SSB pot fi prezenţi în poliartrita
Tema nr. 78 reumatoidă
Patologii autoimune: aspecte epidemiologice, E. ac anti-Jo1 sunt prezenţi în sindromul CREST
diagnostice şi principii de tratament
(pag. 1136)
Bibliografie asociata temei:
Book Des ECN, editia in limba romana, redactor *5 Alegeţi afirmaţia falsă referitoare la
Laurent Karila, 2011 anticorpii antinucleari
pag. 1136-1137 A. prezintă specificitate înaltă pentru lupusul
eritematos discoid
*1 Identificaţi anticorpii cu specificitate înaltă
B. sunt prezenţi la 95% din cazurile de LED
pentru poliartrita reumatoidă
C. includ ac anti-ADN şi anti-ENA
A. anticorpii antinucleari (AAN);
D. sunt de interes diagnostic în colagenoze
B. ac anti-CCP;
E. anti-RNP se asociază bolii mixte de ţesut
C. factorul reumatoid
conjunctiv
D. ac anti-centromer (pag. 1136)
E. ac anti-ADN
6 Anticorpii asociaţi sindromului
(pag. 1136) antifosfolipidic sunt:
*2 Anticorpii specifici, de interes diagnostic şi A. anticoagulantul lupic;
prognostic în lupusul eritematos sistemic
B. anti-proteinaza 3
sunt reprezentaţi de
C. anti-beta2-glicoproteina 1
A. anticorpii antinucleari (AAN)
D. anti-mieloperoxidază
B. ac anti-histone
E. anti-cardiolipină
C. ac anti-ADN
(pag. 1137)
D. anticoagulantul lupic
E. ac anti-antigene nucleare solubile 7 Care din următoarele afirmaţii privind
anticorpii anti-ENA nu sunt adevarate
(pag. 1136)
A. ac anti-Sm sunt foarte specifici pentru lupusul
*3 Anticorpii anti-antigene nucleare solubile indus medicamentos
includ, cu excepţia
B. sclerodermia sistemică poate pozitiva ac anti-
A. ac anti-RNP SCL-70
B. ac anti-centromer C. ac anti-PM1 sunt asociaţi miozitelor inflmatorii
C. ac anti-histone D. ac anti-SSA şi anti-SSB sunt specifici lupusului
eritematos sistemic
D. ac anti-ADN
E. sindromul CREST nu se asociază cu ac anti-
E. ac anti-PM1
centromer
(pag. 1136)
(pag. 1136)

Pag. 108 din 130


8 Principiile de tratament şi monitorizare pe Tema nr. 79
termen lung a unei boli autoimune vizează Indicaţii şi strategii de utilizare a principalelor
A. corticoterapia adesea nu este necesară, fiind examene imagistice
suficient tratamentul simptomatic Bibliografie asociata temei:
B. imunosupresoarele se adresează formelor Book Des ECN, editia in limba romana, redactor
corticodependente Laurent Karila, 2011
C. scăderea dozelor de cortizon necesită pag. 1149-1154 * fara pag. 1150 Tabelul 2, pag.
administrarea imunosupresiei 1151 Tabelul 3
D. imunoterapia cu anti-TNF-α este un tratament
simptomatic obligatoriu în poliartrita reumatoidă 1 Care dintre următorele afecţiuni constituie
contraindicaţie în cazul efectuării unei
E. suprimarea factorilor declanşatori radiografii
(pag. 1137) A. sarcina
9 Principalele patologii autoimune B. disecţia de aortă
A. prezintă o frecvenţă crescută la femei C. apendicita acută
B. sunt îndreptate împotriva antigenelor ubicuitare D. colecistita
C. nu prezintă factori favorizanţi E. insuficienţa renală severă în caz de utilizare a
substanţelor de contrast vasculare
D. survin pe fondul predispoziţiei genetice
(pag. 1149)
E. nu recunosc etiopatogenic implicarea factorilor
de mediu 2 Care sunt avantajele utilizării ecografiei
(pag. 1136) A. rapiditatea
10 Vasculitele necrozante primitive B. costurile reduse
A. definesc o bolă autoimună organ-specifică C. este neiradiantă
B. în boala Wegener ac anti-cANCA prezintă D. neinvazivitatea
specificitate înaltă E. este operator dependentă
C. ac anti-MPO sunt asociaţi cu micropoliangeita (pag. 1149)
D. sindromul Churg-Strauss pozitivează ac anti-p-
ANCA *3 Următoarele afirmaţii sunt adevărate în cazul
utilizării computer tomografiei ca metodă de
E. anti-proteinază-3 sunt de tip p-ANCA diagnostic
(pag. 1137)
A. foloseşte radiaţii X cu posibilitatea injectării de
11 Imunoglobulinele monoclonale produse de contrast vasculare

A. sunt tipate prin imunofixare B. utilizează rezonanţa magnetică nucleară

B. peak-uri IgM evocă boala Waldenstrom C. utilizează radiaţii radioactive

C. peak-uri IgE, IgA sau IgM ne orientează spre un D. se bazează pe propagarea ultrasunetelor
mielom E. este neiradiantă
D. nu pot fi identificate în limfoame şi colagenoze (pag. 1149)
E. se identifică prin electroforeza protidelor serice 4 Printre contraindicaţiile computer
(pag. 1137) tomografiei se numără
A. femei gravide
B. alergia
C. insuficienţa renală terminală
D. malformaţii fetale
E. epilepsia
(pag. 1150)

Pag. 109 din 130


5 Contraindicaţiile absolute în cazul folosirii * 10 10.În cazul unui traumatism cerebral, ce
rezonanţei magnetice ca metodă de metodă de diagnostic este preferată la un
diagnostic sunt pacient simptomatic
A. fibroza nefrogenică A. radiografia craniană
B. pacemaker B. CT cerebral
C. corpii străini metalici C. rezonanţă magnetică
D. claustrofobia D. scintigrafia
E. costul ridicat E. tomografia cu emisie de pozitroni (PET-CT)
(pag. 1150) (pag. 1154)

6 6.Radiografia simplă se foloseşte în cazul Tema nr. 80


următoarelor urgenţe diagnostice Infecţii cutaneo-mucoase bacteriene şi micotice
A. insuficienţă renală acută obstructivă Bibliografie asociata temei:
B. embolie pulmonară Book Des ECN, editia in limba romana, redactor
C. pneumoperitoneu Laurent Karila, 2011
pag. 1168-1172
D. ocluzie intestinală
E. compresiune medulară *1 Precizati care afirmatie este falsa
(pag. 1151) A. Pielea nu este sterila
*7 Care din următoarele urgenţe diagnostice B. Infectiile cutanate bacteriene sunt foliculare si
necesită utilizarea CT cerebral nefoliculare
A. malformaţia arterio-venoasă C. Diagnosticul infectiilor bacteriene este clinic in
majoritatea cazurilor
B. accident vascular cerebral sub anticoagulant
D. Exista imunizare impotriva infectiilor bacteriene
C. pneumocefalie
E. Majoritatea infectiilor cutanate bacteriene sunt
D. tulburări ale stării de conştienţă produse de cocii Gram +
E. epilepsie (pag. 1168)
(pag. 1152)
*2 Care dintre caracteristicile foliculitei nu este
*8 În cazul uni politraumatism al întregului adevarata
corp, care este metoda de urgenţă A. Reprezinta inflamatia foliculului pilosebaceu
diagnostică folosită
B. Face parte dintre infectiile cutanate bacteriene
A. computer tomografică nefoliculare
B. rezonanţă magnetică C. Are ca leziune elementara papulo-pustula
C. radiografia D. Se poate localiza la barba
D. ecografia E. In majoritatea cazurilor este produsa de
E. ultrasonografie stafilococul aureu.
(pag. 1153) (pag. 1168)

*9 În cazul cărei afecţiuni se efectuează RMN *3 Care dintre urmatoarele caracteristici ale
medulară de urgenţă furunculului nu este corecta

A. spondilodiscită A. Este o infectie profunda a foliculului pilosebaceu


produsa de streptococul beta-hemolitic
B. hernie de disc
B. Este un nodul centrat de fir de par
C. tumori intrarahidiene
C. Are evolutie spre necroza si eliminarea
D. compresiune medulară burbionului
E. rahialgii D. Stafilococia maligna a fetei reprezinta o
(pag. 1154) complicatie a furunculului.
E. Tratamentul furunculului se face cu antiseptice
sau antibiotice locale.
(pag. 1169)

Pag. 110 din 130


4 Impetigo se caracterizeaza prin 8 Candidozele cutaneo-mucoase au
urmatoarele caracteristici
A. Stare generala alterata, febra si frisoane
B. Prezenta unor vezicule si bule superficiale A. Se pot localiza atat la nivelul marilor pliuri cat si
la nivelul pliurilor mici cutanate.
C. Localizare periorificiala
B. Apar doar la persoanele cu imunodepresii severe
D. Forma buloasa este intalnita mai ales la copii si
varstnici C. Se trateaza cu antifungice generale timp de 6 luni

E. Contagiozitate mare D. Diagnosticul de certitudine se pune pe baza


examenului clinic
(pag. 1169)
E. Candidozele genitale nu apar niciodata la barbat
*5 Care dintre afirmatiile referitoare la (pag. 1171)
tratamentul infectiilor cutanate stafilococice
este adevarata 9 Care dintre urmatoarele afirmatii
caracterizeaza dermatofitoza pielii glabre
A. In formele localizate este suficienta toaleta cu
apa si sapun, antiseptice si antibiotice topice A. Se localizeaza in special la nivelul scalpului
B. Indiferent de forma clinica este necesara B. Se poate confunda cu eczema numulara sau
antibioterapie per os psoriazisul inversat
C. Indiferent de forma clinica este necesara C. Este foarte dureroasa
spitalizarea si antibioterapie iv D. Se prezinta sub forma unor placi inelare cu
D. Penicilina este unul dintre antibioticele de electie extindere centrifuga
E. Tratamentul antibiotic trebuie efectuat timp de 4 E. Se poate confunda cu candidoza marilor pliuri.
saptamani. (pag. 1171)
(pag. 1169)
10 Kerionul este
*6 Erizipelul se caracterizeaza prin
A. O candidoza cutanata suprainfectata
A. Placard inflamator eritematos, cald, dureros.
B. O tricofitie supurata cu inflamatie majora
B. Debuteaza sub forma unei pustule foliculare
C. O infectie dermatofitica produsa de dermatofiti
centrata de fir de par
patogeni
C. Prezenta unei porti de intrare nu este obligatorie
D. Infectie dermatofitica a pielii glabre insotita de
D. Este necesar intodeauna examenul adenopatii satelite frecvente
histopatologic pentru stabilirea diagnosticului. E. Afectiune cauzata de Malassezia furfur.
E. Stare generala alterata, febra, frisoane. (pag. 1171-1172)
(pag. 1170)
Tema nr. 81
7 Care dintre urmatoarele afirmatii legate de
Guşa şi nodulul tiroidian, hipotiroidismul si
erizipel sunt adevarate
hipertiroidismul
A. Fasceita necrotizanta este o complicatie Bibliografie asociata temei:
generala a erizipelului
Book Des ECN, editia in limba romana, redactor
B. Fasceita necrotizanta este favorizata de Laurent Karila, 2011
administrarea antiinflamatoarelor nesteroidiene pag. 1215-1220, 1221-1223, 1224-1226 * fara
C. Indiferent de forma clinica de erizipel este subcapitolul III pag 1225
necesara spitalizarea de urgenta
D. Penicilina G in perfuzie este tratamentul de 1 Care sunt examinările biologice în gușa
electie simplă:

E. Nu este obligatorie tratarea portii de intrare. A. TSH


(pag. 1170) B. Dozarea ATPO și antiTG
C. În caz de hipertrofie dozarea anticorpilor
antireceptor TSH
D. Ioduria pe 24 de ore
E. Dozarea tireoglobulinei
(pag. 1216)

Pag. 111 din 130


*2 În hipertiroidismul indus de amiodaronă: *8 În insuficiența corticotropă (insuficiența
suprarenală centrală) există:
A. Ecografia Doppler tiroidiană arată
hipervascularizație tiroidiană A. Melanodermie
B. Scintigrafia este fixantă B. Paloare
C. Există o patologie tiroidiană preexistentă C. Hipertensiune arterială
D. Ecografia Doppler tiroidiană este normală D. Surplus ponderal
E. Tratamentul se bazează pe anxiolitice, repaus E. Semne de virilizare
(pag. 1223) (pag. 1227)
*3 Diagnosticul de hipertiroidism frust *9 Diagnosticul de hipertiroidism frust
presupune: presupune:
A. TSH scăzut cu FT4 crescut A. TSH scăzut cu FT4 crescut
B. TSH scăzut cu FT4 scăzut B. TSH scăzut cu FT4 scăzut
C. TSH scăzut cu FT4 normal C. TSH scăzut cu FT4 normal
D. TSH crescut cu FT4 crescut D. TSH crescut cu FT4 crescut
E. TSH crescut cu FT4 normal E. TSH crescut cu FT4 normal
(pag. 1221) (pag. 1221)
*4 Boala Basedow este confirmată de: 10 Cauzele bolii Addison sunt:
A. Anticorpi antitireoglobulină crescuți A. Atrofia corticală autoimună
B. Anticorpi antireceptori TSH (TRAb) crescuți B. Tuberculoza suprarenală
C. TSH crescut C. Boala Biermer
D. Proteina C reactivă crescută D. Cauze iatrogene
E. Anticorpi antiperoxidază crescuți E. Cauze genetice
(pag. 1221) (pag. 1228)

5 Cum se monitorizează toleranța la Tema nr. 82


tratamentul cu antitiroidiene de sinteză:
Diabetul zaharat tip 1 şi 2 la adult – definiţie,
A. GOT, GPT la intervale regulate diagnostic, complicaţii acute, complicaţii cronice,
algoritm terapeutic, monitorizare, principii,
B. Hemoleucogramă la intervale regulate
management
C. Proteina C reactivă bilunar
Bibliografie asociata temei:
D. Bilirubina totală lunar Book Des ECN, editia in limba romana, redactor
E. Tireoglobulina la 3 luni Laurent Karila, 2011
(pag. 1223) pag. 1236-1254 * fara date epidemiologice, fara
denumiri comerciale din tratamentul cu insulina,
*6 În insuficiența tireotropă există: pag. 1250.
A. TSH crescut cu FT4 crescut
*1 Diabetul zaharat se defineşte prin:
B. TSH normal cu FT4 crescut
A. Glicemie a jeun în sânge venos mai mare de
C. TSH scăzut cu FT4 crescut 1,16g/l;
D. TSH normal sau scăzut, cu FT4 scăzut B. Glicemie postprandială mai mare de 1,30g/l;
E. TSH crescut cu FT4 scăzut C. Glicemie a jeun în sânge venos mai mare de
(pag. 1224) 1,26g/l, în două ocazii diferite sau mai mare de
2g/l în orice moment al zilei;
7 Complicațiile hipotiroidismului sunt: D. Glicemie a jeun în sânge venos mai mare de
A. Insuficiență coronariană 1,20g/l, în două ocazii diferite sau mai mare de
1,6g/l în orice moment al zilei;
B. Blocuri de ramură
E. Glicemie postprandială mai mare de 1,40g/l;
C. Coma mixedematoasă
(pag. 1236)
D. Depresie
E. Stenoză mitrală
(pag. 1225)

Pag. 112 din 130


*2 Diabetul zaharat tip 1 are ca substrat 7 Nefropatia diabetică incipientă se
patogenic: caracterizează prin:
A. Insulinorezistenţa musculară şi hepatică; A. Microalbuminurie anormală cuprinsă între
B. Citoliza hepatică; 30mg/24 ore şi 299mg/24 ore (sau 30mg şi
299mg/g creatinină);
C. Pancreatita cronică alcoolică;
B. Macroalbuminurie (albumină mai mare de
D. Distrugerea autoimună (în 95% din cazuri) a 300mg/24 ore sau mai mare de 300mg/g
celulelor beta din insulele Langerhans creatinină);
pancreatice, responsabilă de un deficit de
C. Tensiune arterială normală;
insulină;
D. Filtrare glomerulară normală;
E. Amiloidoza pancreatică.
(pag. 1236) E. Sindrom edematos;
(pag. 1245)
*3 Coma diabetică hiperosmolară se
caracterizează prin: 8 Efectele secundare ale tratamentului cu
insulină sunt reprezentate de:
A. Glicemie plasmatică mai mică de 2g/l şi pH
arterial mai mic de 7,30; A. Hipoglicemie;
B. Glicemie plasmatică mai mică de 2g/l şi B. Câştig în greutate în caz de supradozare;
bicarbonat plasmatic mai mic de 15mmol/l; C. Alergie;
C. Bicarbonat plasmatic mai mic de 15mmol/l şi pH
D. Lipodistrofii hipertrofice;
arterial mai mic de 7,30;
E. Hirsutism;
D. Hiperglicemie mai mare de 33mmol/l şi
osmolaritate mai mare de 350mmol/l; (pag. 1250)
E. Osmolaritate cuprinsă între 310 mOsm/kg şi 9 Efectele diabetului zaharat asupra sarcinii
315mOsm/kg; sunt reprezentate de:
(pag. 1241)
A. Malformaţii, avort;
*4 Tratamentul hipoglicemiant în diabetul B. Microsomie;
zaharat tip 1 se face cu:
C. Retard de maturare;
A. Metformin; D. Hipoglicemie şi hipocalcemie neonatale;
B. Metformin şi sulfamide hipoglicemiante; E. Hipertensiune gravidică şi preeclampsie;
C. Insulină; (pag. 1251)
D. Glinide;
10 Monitorizarea biologică a pacientului cu
E. Sulfamide hipoglicemiante; diabet zaharat presupune:
(pag. 1250)
A. HbA1c, monitorizare sistematică, de 4 ori pe an;
*5 Hemoglobina glicată (HbA1c) reflectă B. Glicemie venoasă a jeun, 1 dată pe lună;
echilibrul metabolic:
C. Bilanţ lipidic (CT, HDL-C, TG, calculul LDL-C) 1
A. Zilnic; dată pe an;
B. Săptămânal; D. Microalbuminurie, 1 dată pe an;
C. Lunar; E. Creatinemie a jeun 1 dată pe an.
D. Din cele 2-3 luni precedente; (pag. 1253)
E. Anual;
(pag. 1249)

6 Retinopatia diabetică neproliferativă se


caracterizează prin:
A. Dilatare capilară;
B. Microanevrisme;
C. Exudate;
D. Hemoragii;
E. Neovase.
(pag. 1244)

Pag. 113 din 130


Tema nr. 83 *6 Contraindicațiile chirurgiei bariatrice sunt, cu
Obezitatea la adulţi excepția:

Bibliografie asociata temei: A. Tulburări mentale severe;


Book Des ECN, editia in limba romana, redactor B. Dependența de alcool;
Laurent Karila, 2011 C. Pacient bine informat, cu spijin psihologic;
pag. 1276-1280 * fara denumiri comerciale ale
D. Dependența de substanțe psihoactive;
medicamentelor
E. Tulburări severe ale comportamentului alimentar.
1 Sindromul metabolic are următoarele (pag. 1279)
elemente:
*7 Complicațiile respiratorii ale obezității sunt,
A. Insulinorezistență;
cu excepția:
B. Diabet zaharat;
A. Sindromul metabolic;
C. Hipertensiune arterială;
B. Astmul;
D. Obezitate ginoidă;
C. Dispneea de efort;
E. Hiper HDL colesterolemie.
D. Sindromul de apnee în som;
(pag. 1276)
E. Sindromul restrictiv pulmonar.
2 Obezitatea secundară este dată de: (pag. 1276)
A. Hipotiroidism;
*8 Chirurgia bariatrică reversibilă constă în:
B. Hipertiroidism;
A. By-pass gastric;
C. Factori genetici;
B. Gastro-duodeno anastomoză;
D. Hipocorticism;
C. Gastrectomie totală;
E. Hipercorticism;
D. Esofagoplastie;
(pag. 1276)
E. Gastroplastie cu inel gastric.
3 Care dintre următoarele afirmații despre (pag. 1279)
Indicele de masă corporală sunt corecte:
*9 Obezitatea poate fi secundară:
A. IMC peste 40 kg/m2 semnifică obezitate morbidă;
B. IMC peste 18,5 kg/m2 semnifică A. Hipertiroidismului;
subponderabilitate; B. Hipercorticismului;
C. IMC între 18,5 și 24,9 semnifică greutate C. Hipocorticismului;
normală;
D. Gutei;
D. IMC între 25 și 29,9 semnifică
E. Dislipidemiei.
supraponderabilitate;
(pag. 1276)
E. IMC între 30 și 34,5 inseamnă obezitate.
(pag. 1276) * 10 Indicele de masă corporală are la o
persoană normoponderală următoarea
4 Complicațiile cardio-vasculare ale obezitații valoare:
sunt:
A. Sub 18,5;
A. Hipertensiunea arterială;
B. Peste 40;
B. Insuficiența cardiacă;
C. Între 18,5 și 24,9;
C. Hipotensiune arterială;
D. Sub 40;
D. Accident vascular cerebral;
E. Peste 18,5.
E. Hipertrofie ventriculară;
(pag. 1276)
(pag. 1276)

5 Complicațiile endocrine ale obezității sunt:


A. Litiaza biliară;
B. Sindromul ovarelor polichistice;
C. Hipofertilitate;
D. Hipogonadism;
E. Boală de reflux gastroesofagian.
(pag. 1277)

Pag. 114 din 130


Tema nr. 84 *5 Care dintre următoarele afirmaţii referitoare
Infecţiile urinare la adult. Leucocituria la infecţiile urinare nosocomiale sunt
adevărate?
Bibliografie asociata temei:
A. sunt infecţiile nosocomiale cele mai rar întâlnite
Book Des ECN, editia in limba romana, redactor
Laurent Karila, 2011 B. sunt cu germeni sensibili la antibiotice uzuale
pag. 1286-1292 * fara denumiri comerciale ale C. sunt definite ca infecţii dobândite după
medicamentelor externarea dintr-un centru de îngrijire
D. sunt cu germeni cel mai adesea multirezistenţi
*1 Leucocituria se defineşte (se caracterizează)
printr-un număr de leucocite la ECBU E. sunt foarte greu de diagnosticat
A. mai mare de 10/ml (pag. 1291)
B. mai mare de 100/ml 6 Leucocituria izolată (cu uroculturi negative)
C. egal cu 1000/ml cu leucocite alterate se poate întâlni în:
D. mai mare de 1000/ml A. imunodepresie
E. mai mare de 10000/ml B. infecţie urinară decapitată de tratament
antimicotic
(pag. 1292)
C. litiază vezicală
*2 Leucocituria cu leucocite nealterate, în
D. infecţii cu germeni atipici (Klebsiella, Proteus)
cilindru, este caracteristică pentru:
E. tumoare de vezică
A. glomerulopatia cronică
(pag. 1292)
B. nefropatia tubulointerstiţială cronică
C. feocromocitom 7 Pentru prevenţia infecţiilor urinare
nosocomiale la pacienţii cu sondaj vezical
D. tuberculoza urinară sunt recomandate:
E. infecţia cu chlamydia A. administrarea de laxative pentru a preveni
(pag. 1292) constipaţia

*3 În pielonefrita acută simplă, durata B. păstrarea unei diureze importante


tratamentului antibiotic trebuie să fie de: C. limitarea duratei sondajului
A. 12 zile D. menţinerea sacului colector în poziţie declivă
B. 15 zile E. spălarea zilnică a sondei cu soluţii antibiotice
C. 18 zile (pag. 1291)
D. 21 zile 8 Dintre diversele clase de antibiotice, în
E. 31 zile tratamentul prostatitelor cronice sunt
indicate:
(pag. 1289)
A. cefalosporine de generaţia a 2-a
*4 La un pacient cu prostatită acută la care se
instalează şi retenţia acută de urină este B. cefalosporine de generaţia a 3-a
indicat(ă): C. fluorochinolone
A. sondajul uretral D. aminoglicozide
B. cateter suprapubian E. cicline
C. nefrostomie bilaterală (pag. 1290)
D. cateterism ureteral bilateral 9 Semnele funcţionale urinare prezente în
E. cistoscopia prostatita cronică sunt:
(pag. 1290) A. disurie
B. hematurie
C. ejaculare retrogradă
D. anejaculare
E. arsuri micţionale
(pag. 1290)

Pag. 115 din 130


10 Semnele clinice ale cistitei acute simple sunt: *5 La pacienţii cu retenţie acută de urină la care
s-a practicat sondaj uretral este recomandată
A. urină tulbure şi urât mirositoare
golirea vezicală progresivă pentru a evita:
B. polakiurie
A. infecţia de tract urinar
C. imperiozitate
B. hematuria ex vacuo
D. febră
C. dispariţia retenţiei azotate
E. disurie
D. apariţia dezechilibrelor hidro-electrolitice
(pag. 1287)
E. apariţia tulburărilor echilibrului acido-bazic
Tema nr. 85 (pag. 1295)
Retenţia acută de urină
6 Globul vezical se caracterizează prin:
Bibliografie asociata temei:
A. bombarea regiunii lombare
Book Des ECN, editia in limba romana, redactor
Laurent Karila, 2011 B. matitate
pag. 1295-1296 C. durere în fosele iliace, bilateral
D. asociere cu cefalee
*1 Diagnosticul retenţiei acute de urină este:
E. palparea regiunii hipogastrice trezeşte nevoia de
A. clinic
a urina
B. radiologic (pag. 1295)
C. ecografic
7 La pacienţii cu retenţie acută de urină,
D. tomografic sondajul uretral este contraindicat în
E. radiourografic următoarele cazuri:
(pag. 1295) A. prostatită acută
*2 Retenţia acută de urina este: B. traumatism de bazin

A. situaţia clinică în care rinichii nu mai produc urină C. traumatism de uretră

B. fază tardivă ce apare în evoluţia insuficienţei D. tromboză hemoroidală


renale cronice E. sechele post-accident vascular cerebral
C. imposibilitatea totală şi brutală de a urina (pag. 1295)
D. un fenomen asociat colicii renale 8 La pacienţii cu retenţie acută de urină, în
E. un fenomen mai frecvent întâlnit la femei decât la afara drenajului urinei, sunt necesare măsuri
bărbaţi asociate:
(pag. 1295) A. cuantificarea urinei recoltate
*3 Pentru diagnosticul retenţiei acute de urină, B. o bună hidratare a pacientului
la pacienţii obezi, poate fi utilă: C. repaus la pat pentru 24 de ore
A. uroflowmetria D. tratament anticoagulant
B. citologia urinară E. ECBU pe urina drenată
C. cistografia retrogradă (pag. 1295)
D. ecografia vezicală la patul pacientului
9 Monitorizarea pacienţilor cu retenţie acută de
E. ureteropielografia retrogradă urină la care s-a realizat drenajul urinei
(pag. 1295) include:
A. monitorizarea diurezei
*4 La pacienţii cu retenţie acută de urină se
impune: B. monitorizarea ionogramei urinare
A. drenajul de urgenţă al urinei prin sondaj vezical C. monitorizarea ionogramei sanguine
B. drenajul de urgenţă al urinei prin nefrostomie D. monitorizarea culorii urinei
bilaterală E. monitorizarea tensiunii arteriale
C. drenajul de urgenţă al urinei prin cateterizarea (pag. 1296)
ureterală retrogradă bilaterală
D. drenajul de urgenţă al urinei prin montarea unui
prezervativ anti-incontinenţă
E. drenajul urinei după examen neurologic
(pag. 1295)

Pag. 116 din 130


10 Dintre următoarele clase de medicamente, *4 Prevenirea recidivelor litiazei urice se
care sunt factori declanşatori/favorizanţi ai realizează prin:
retenţiei acute de urină?
A. acidifierea urinei
A. inhibitorii enzimei de conversie B. cură de diureză
B. alfa blocante C. regim hiperpurinic
C. alfa stimulante D. tratament diuretic
D. beta blocante E. fitoterapie
E. morfina (pag. 1314)
(pag. 1295)
*5 Oxalatul de calciu din compoziţia calculilor
Tema nr. 86 calcici poate fi:
Litiaza urinară A. dihidratat
Bibliografie asociata temei: B. trihidratat
Book Des ECN, editia in limba romana, redactor C. tetrahidratat
Laurent Karila, 2011
D. pentaahidratat
pag. 1312-1315 * fara denumiri comerciale ale
medicamentelor E. hexatahidratat
(pag. 1312)
*1 Drenajul de urgenţă – prin montarea de
sondă ureterală /dublu J sau nefrostomie 6 Dintre următoarele uropatii, care sunt
percutană – la un pacient cu litiază urinară implicate în etiologia litiazei urinare?
este necesar dacă:
A. rinichi în potcoavă
A. este vorba de litiază recidivată B. rinichi spongios
B. colica renală este complicată C. rinichi polichistic
C. colica renală este precedată de hematurie totală D. ureterocel
D. pacientul este necooperant şi se anticipează că E. megaureter
nu va respecta indicaţiile de tratament
(pag. 1312)
E. calculul are 5 mm
(pag. 1313) 7 Ancheta etiologică la un pacient cu litiază
urinară include:
*2 Pentru ablaţia calculului la un pacient cu
A. interogatoriu: antecedente de uretrită, obiceiul de
litiază urinară se poate folosi:
a fuma, consum de alcool
A. nefrostomia percutanată B. identificarea unei uropatii obstructive
B. uretroscopia C. spectrofotometrie în infraroşu a calculului
C. cistoscopia D. spectrofotometrie în ultraviolet a calculului
D. rezecţia transuretrală E. spectrofotometrie LASER
E. litotriţia extracorporeală (pag. 1314)
(pag. 1314)
8 Litiaza fosfoamoniacomagneziană este
*3 În colica renală simplă se administrează: favorizată de:
A. diuretic A. pH urinar acid
B. antibiotic i.v. apoi per os B. pH urinar alcalin
C. AINS i.v. apoi per os C. infecţii urinare acute
D. alfa stimulant D. infecţii urinare cronice
E. morfină E. glomerulonefrite acute
(pag. 1313) (pag. 1312)

Pag. 117 din 130


9 Pe radiografia abdominală simplă efectuată *3 Beta blocantele sunt utilizate în tratamentul
la un pacient cu litiază urinară, în faţa hipertrofiei benigne de prostată pentru că
calcificărilor prezente pe radiografie, prin modul lor de acţiune realizează:
diagnosticul diferenţial al acestora se va face
A. relaxarea muşchilor netezi prostatici
cu:
B. deschiderea colului vezical
A. litiaza biliară
C. diminuarea volumului prostatei
B. calcificări ganglionare
D. scăderea vascularizaţiei prostatei
C. calcificări pleurale
E. nu sunt utilizate
D. ateroame aortice
(pag. 1317)
E. calcificări condrocostale
(pag. 1314) *4 Inhibitorii de 5 alfa reductază sunt utilizaţi în
tratamentul hipertrofiei benigne de prostată
10 Diagnosticul diferenţial al colicii nefretice se pentru că prin modul lor de acţiune
face cu alte cauze de durere lombară acută: realizează:
A. pneumopatie A. relaxarea muşchilor netezi prostatici şi
B. colica biliară deschiderea colului vezical
B. diminuarea volumului prostatic
C. colon iritabil
C. controlul hematuriei caracteristice hipertrofiei
D. durerea articulară din gonartroză
benigne de prostată
E. pancreatita acută
D. sterilizarea urinei
(pag. 1314)
E. optimizarea diurezei şi controlul nicturiei
Tema nr. 87 (pag. 1317)
Patologia tumorala benigna si maligna a aparatului
*5 Inhibitorii de 5 alfa reductază - utilizaţi în
uro-genital masculin: hipertrofia benignă de
tratamentul hipertrofiei benigne de prostată –
prostată, tumori de prostată, tumori de rinichi, tumori
au:
de testicul
Bibliografie asociata temei: A. eficacitate imediată
Book Des ECN, editia in limba romana, redactor B. eficacitate după 3 luni
Laurent Karila, 2011 C. eficacitate după 6 luni
pag. 1316-1318, 1319-1322, 1323-1327, 1328-1331
D. eficacitate după 9 luni
* fara denumiri comerciale ale medicamentelor
E. eficacitate după 12 luni
*1 În hipertrofia benignă de prostată cu (pag. 1317)
simptomatologie severă, scorul internaţional
al simptomelor de prostată (IPSS) este: 6 În hipertrofia benignă de prostată, semnele
funcţionale urinare obstructive sunt:
A. 0 - 7
B. 8 – 19 A. jet urinar întrerupt

C. 20 – 35 B. jet urinar sanguinolent

D. 20 – 45 C. jet urinar tulbure

E. 20 – 50 D. disurie

(pag. 1316) E. picături retardate


(pag. 1316)
*2 În hipertrofia benignă de prostată, în cadrul
depistării individuale a cancerului de 7 În hipertrofia benignă de prostată, semnele
prostată, bilanţul complementar trebuie să funcţionale urinare iritative sunt:
includă:
A. polakiurie
A. PSA total B. usturimi la micţiune
B. PSA total şi PSA liber C. nicturie
C. Fosfataza alcalină prostatică D. urgenturie
D. Antigenul carcinoembrionar E. incontinenţă de urină prin revărsare
E. Lactic dehidrogenaza (pag. 1316)
(pag. 1316)

Pag. 118 din 130


8 În hipertrofia benignă de prostată, bilanţul * 13 În cancerul prostatic avansat local – conform
complementar trebuie să includă: stadializării TNM – tratamentul poate fi
curativ şi metoda aleasă va fi reprezentată
A. Aprecierea funcţiei renale prin dozarea ureei,
de:
creatininei, acidului uric şi RFG (rata filtrării
glomerulare) A. RTUP
B. ECBU B. prostatectomie totală
C. Ecografie reno-vezico-prostatică C. radioterapie prelungită
D. Debitmetrie urinară D. radio-hormono-terapie prelungită
E. Măsurarea volumului vezical premicţional E. radioterapie prostatică conformaţională
(pag. 1315) (pag. 1322)
9 În evoluţia hipertrofiei benigne de prostată * 14 În cancerul prostatic metastatic – conform
pot apărea următoarele complicaţii datorate stadializării TNM – tratamentul este paliativ şi
impactului asupra aparatului urinar superior: metoda aleasă va fi reprezentată de:
A. hidronefroză A. hormonoterapie
B. polichistoză B. prostatectomie totală
C. rinichi spongios C. radioterapie prelungită
D. insuficienţă renală cronică obstructivă D. radio-hormono-terapie prelungită
E. hematurie macroscopică E. radioterapie prostatică conformaţională
(pag. 1317) (pag. 1322)
10 În evoluţia hipertrofiei benigne de prostată * 15 Triptorelina este folosită în tratamentul
pot apărea următoarele complicaţii datorate hormonal al cancerului de prostată si este un:
impactului vezical:
A. antiandrogen
A. diverticuli vezicali
B. blocant androgenic complet
B. litiază vezicală
C. estrogen
C. retenţie acută de urină
D. agonist LHRH
D. micţiuni prin revărsare
E. androgen
E. hidaturie (pag. 1322)
(pag. 1317)
16 Pentru diagnosticul de certitudine al
* 11 În scopul depistării individuale a cancerului cancerului prostatic sunt necesare biopsiile
de prostată, dozarea PSA seric total trebuie: prostatice recoltate prin puncţie.
A. începută la vârsta de 50 de ani şi repetată annual Complicaţiile posibile ale puncţiei sunt
reprezentate de:
B. începută la vârsta de 50 de ani şi repetată la
fiecare 3 ani A. pneumaturie
C. începută la vârsta de 50 de ani şi repetată la B. hematurie
fiecare 2 ani C. rectoragie
D. începută la vârsta de 50 de ani şi repetată la D. melenă
vârsta de 75 de ani
E. boală hemoroidală
E. începută la vârsta de 60 de ani şi repetată la
fiecare 6 luni (pag. 1320)

(pag. 1319) 17 PSA este o proteină specifică prostatei şi


creşterea sa poate avea şi alte etiologii în
* 12 La un pacient cu o valoare determinată a afara cancerului prostatic. Care dintre cele
PSA total mai mare de 4 ng/ml, sunt de mai jos poate duce la creşterea PSA?
necesare:
A. hematuria
A. masaj prostatic
B. tumora vezicală
B. prostatectomie radicală laparoscopică
C. prostatita
C. biopsii prostatice
D. hipertrofia benignă de prostată
D. scintigrafie osoasă
E. ecografia abdominală a prostatei
E. stabilirea riscului pentru progresie D´AMICO
(pag. 1319)
(pag. 1319)

Pag. 119 din 130


18 Pentru diagnosticul de certitudine al * 23 În cadrul examinării testiculare la un pacient
cancerului prostatic sunt necesare biopsiile la care diagnosticul prezumtiv este de
prostatice recolatae prin puncţie. Precauţiile tumoare testiculară, semnul Chevassu
prealabile puncţiei includ: pozitiv înseamnă:
A. preparare rectală A. identificarea unei mase dure, indolore
B. tratament anticoagulant B. opacitate la transiluminare
C. antibioprofilaxie C. identificarea dificilă a deferentului
D. tratament antiagregant plachetar D. conservarea şanţului epididimo-testicular
E. injectare intraprostatică de substanţă de contrast E. durere la palparea testiculului
(pag. 1320) (pag. 1328)

19 Care dintre următorii pacienţi cu cancer de * 24 La un pacient cu tumoare testiculară,


prostată stadializaţi TNM se încadrează în examenul clinic general caută:
stadiul l?
A. adenopatie inghinală superficială
A. T1c /N0/ M0 B. adenopatie inghinală profundă
B. T2a/N0/M0 C. adenopatie retroperitoneală
C. T2b/N0/M0 D. adenopatie perineală
D. T2c/N0/M0 E. adenopatie femurală
E. T3a/N0/M0 (pag. 1328)
(pag. 1321)
* 25 Orice dubiu în diagnosticul diferenţial al unei
20 În cancerul prostatic localizat – conform tumori testiculare impune:
stadializării TNM – tratamentul este curativ şi
A. RMN pelvis
pacientul poate opta – ca metodă de
tratament – pentru: B. CT pelvis
A. RTUP C. Orhiectomie
B. prostatectomie totală D. Explorare chirurgicală pe cale scrotală
C. radioterapie prelungită E. Explorare chirurgicală pe cale inghinală
D. radio-hormono-terapie (pag. 1328)

E. radioterapie prostatică conformaţională 26 Markerii serici în tumorile testiculare sunt:


(pag. 1321) A. alfa fetoproteina
* 21 Vârful de incidenţă (de frecvenţă) al B. HCG
cancerului testicular este situat în intervalul: C. LDH
A. 15-25 ani D. CAE
B. 20-35 ani E. TSH
C. 25-45 ani (pag. 1329)
D. 35-45 ani 27 Tratamentul chirurgical al tumorilor
E. 30-50 ani testiculare este reprezentat de orhiectomia
(pag. 1328) pe cale inghinală şi trebuie să îndeplinească
următoarele condiţii:
* 22 Cel mai important factor de risc pentru
A. clampare prealabilă a cordonului spermatic
cancerul testicular este:
B. ligatura în bloc a cordonului spermatic
A. atrofia testiculară posttraumatică
C. montarea de proteză testiculară (în absenţa
B. atrofia testiculară infecţioasă infecţiei)
C. torsiunea cordonului spermatic D. explorarea chirurgicală a bursei scrotale
D. varicocelul controlaterale
E. criptorhidia E. limfodisecţia inghinală superficială homolaterală
(pag. 1328) (pag. 1329)

Pag. 120 din 130


28 Din punct de vedere al răspunsului la * 33 În tumorile maligne de rinichi, forma
tratament complementar, seminomul este: anatomopatologică cea mai frecventă este:
A. radiorezistent A. carcinomul tubulo-papilar tip l
B. radiosensibil B. carcinomul tubulo-papilar tip ll
C. chimiorezistent C. carcinom la celulele cromofobe
D. chimiosensibil D. carcinom la tubii colectori Bellini
E. hormonodependent E. carcinom cu celule clare
(pag. 1330) (pag. 1326)

29 După orhiectomie, pacienţii cu tumori * 34 Carcinomul renal cu celule clare se dezvoltă


testiculare non-seminomatoase beneficiază începând din:
de tratament:
A. celulele tubului contort
A. chimioterapic B. celulele tubului colector
B. chirurgical C. celulele epiteliului din capsula Bowman
C. radioterapic D. uroteliul calicelor mici
D. hormonoterapie E. uroteliul calicelor mari
E. corticoterapie (pag. 1326)
(pag. 1331)
* 35 În tumorile maligne ale rinichiului, gradul
30 Monitorizarea pacienţilor cu tumori nuclear Fuhrman se stabileşte pe baza:
testiculare (operaţi şi care au necesitat/nu au
A. examenului CT
necesitat tratament complementar) include:
B. examenului uro-CT
A. markeri serici
C. examenului anatomopatologic
B. CT toraco-abdomino-pelvin
D. examenului RMN
C. Autopalpare
E. examenului scintigrafic
D. Radiografie pulmonară – annual
(pag. 1326)
E. Radiografie abdominală – annual
(pag. 1331) 36 La examinarea ecografică a unui chist renal,
leziunea are următoarele caracteristici:
* 31 Clasificarea Bosniak utilizată în chistul renal
A. anecogenă cu întărire posterioară
se referă la:
B. izoecogenă cu minimă întărire posterioară
A. aprecierea gradului de diferenţiere celulară
pentru celulele din lichidul intrachistic C. hiperecogenă cu întărire anterioară
B. aprecierea aspectului macroscopic al lichidului D. avasculară
intrachistic E. pereţi rigizi
C. clasificarea chisturilor funcţie de dimensiunile (pag. 1327)
acestora
D. clasarea chisturilor de la benign la foarte suspect 37 Dintre următoarele afirmaţii referitoare la
oncocitomul renal, bifaţi-le pe cele corecte:
E. clasarea chisturilor de la benign la malign
A. diagnosticul diferenţial cu cancerul renal este
(pag. 1327) uşor
* 32 Identificarea ecografică sau tomografică a B. diagnosticul diferenţial cu cancerul renal este
unui chist renal de 4 cm clasificat Bosniak l dificil
va fi urmată de: C. diagnosticul diferenţial cu cancerul renal nu este
A. tratament conservator necesar
B. puncţionare percutanată sub control ecografic D. imagistic este anecogen, omogen, cu pereţi fini
C. evacuare prin chirurgie laparoscopică E. imagistic se identifică cicatrice centrală în
mijlocul leziunii
D. evacuare prin chirurgie deschisă
(pag. 1327)
E. tratament diuretic
(pag. 1327)

Pag. 121 din 130


38 Dintre următoarele afirmaţii referitoare la *2 La un pacient varstnic, cu hipertrofia
angiomiolipomul renal, bifaţi-le pe cele prostatei decelata la tuseul rectal, cresterea
corecte: brusca si importanta a ureei si creatininei
serice sugereaza:
A. poate fi bilateral
B. este totdeauna unilateral A. infectie urinara

C. este totdeauna bilateral B. insuficienta renala acuta functionala (prerenala)

D. este o tumoare mezenchimatoasă benignă C. insuficienta renala acuta organica (renala)

E. este cea mai frecventă tumoare benignă solidă a D. insuficienta renala acuta obstructiva (postrenala)
rinichiului E. insuficienta renala cronica.
(pag. 1326) (pag. 1350)
39 Care dintre următoarele variante de abordare *3 Cea mai frecventa cauza a insuficientei
terapeutică ale angiomiolipomului renal pot fi renale acute organice (renale) este:
corecte (funcţie de particularităţile cazului)?
A. necroza tubulara acuta
A. embolizare
B. nefropatia interstitiala acuta
B. chimioterapie
C. sindromul nefrotic impur
C. radioterapie
D. glomerulonefrita rapid progresiva
D. tratament chirurgical conservator
E. nefroangioscleroza maligna.
E. abstenţie terapeutică (pag. 1351)
(pag. 1327)
4 Necroza tubulara acuta poate fi cauza
40 După nefrectomia lărgită la un pacient cu insuficientei renale acute daca:
tumoare renală malignă, monitorizarea va
A. apare dupa administarea de: substante toxice:
include:
antibiotice, substante de contrast iodate
A. creatininemia B. ecografia renala este normala
B. VSH-ul C. clinic exista frecvent deshidratare extracelulara
C. urografia i.v. D. sunt absente: HTA, hematuria sau albuminuria
D. CT abdominal E. se produce retentie acuta de urina.
E. Scintigrafia renală (pag. 1351)
(pag. 1326)
5 Care din urmatoarele afectiuni pot determina
Tema nr. 88 aparitia insuficientei renale acute functionale
Insuficienţa renală acută si cronica. Anuria (prerenale):

Bibliografie asociata temei: A. fistule digestive


Book Des ECN, editia in limba romana, redactor B. arsuri
Laurent Karila, 2011 C. fibroza retroperitoneala
pag. 1350-1352, 1353-1357
D. periarterita nodoasa
1 Selectati cauzele obstructive (postrenale) ale E. insuficienta cardiaca
insuficientei renale acute: (pag. 1351)
A. adenom sau cancer prostatic
*6 Stadiul 3 al insuficientei renale cronice este
B. litiaza urinara definit prin rata filtrarii glomerulare estimata
C. insuficienta cardiaca de:
D. sindrom nefrotic A. > 90 ml/min/1,73 m ²
E. fibroza retroperitoneala B. 60 - 90 ml/min/1,73 m ²
(pag. 1350) C. 30 - 59 ml/min/1,73 m ²
D. 15 - 29 ml/min/1,73 m ²
E. < 15 ml/min/1,73 m ²
(pag. 1354)

Pag. 122 din 130


7 Complicatiile cardiovasculare ale Tema nr. 89
insuficientei renale cronice sunt: Nefropatii glomerulare si vasculare
A. HTA Bibliografie asociata temei:
B. hipotensiunea arteriala Book Des ECN, editia in limba romana, redactor
C. pericardita Laurent Karila, 2011
pag. 1364-1366, 1367-1369
D. cardiopatia hipertrofica
E. endocardita bacteriana 1 Sindromul glomerular este definit de
(pag. 1353) urmatoarele semne clinico-biologice:
A. proteinurie, adesea abundenta
*8 Initerea terapiei de substitutie renala la
pacientii cu insuficienta renala cronica este B. hematurie (microscopica sau macroscopica) de
indicata in: cauza non-urologica
A. stadiul 1 C. leucociturie
B. stadiul 2 D. HTA
C. stadiul 3 E. edeme ale membrelor inferioare.
D. stadiul 4 (pag. 1364)

E. stadiul 5. 2 La pacientii cu sindrom glomerular, bilantul


(pag. 1355) biologic minimal cuprinde:
A. glicemia si Hb A1c
9 Care sunt indicatiile absolute pentru initierea
epurarii extrarenale (EER) la pacientii cu B. Ac anti AND
insuficienta renala cronica ?: C. ANCA
A. pericardita uremica D. Ac anti MBG
B. HTA severa E. antigenul specific prostatic
C. hipervolemie care nu poate fi controlata cu (pag. 1364)
diuretice
*3 Care din urmatoarele enunturi referitoare la
D. acidoza metabolica severa
sindromul nefritic acut este FALS :
E. hiperkalemie rezistenta la tratament medical
A. aparitia brusca a unei hematurii microscopice
(pag. 1357)
B. apare proteinurie glomerulara
* 10 Care dintre urmatoarele enunturi NU asigura C. se insoteste de HTA
nefroprotectie in insuficienta renala cronica :
D. apare insuficienta renala acuta
A. controlul presiunii arteriale < 140/90 mmHg
E. prezinta leucociturie si cilindri leucocitari
B. controlul proteinuriei < 0,5 g/24 ore
(pag. 1364)
C. existenta unui echilibru metabolic al diabetului
*4 Care dintre urmatoarele nefropatii
D. evitarea administrarii de substante si
glomerulare este proliferativa
medicamente nefrotoxice
E. controlul factorilor de risc cardiovascular A. cu leziuni glomerulare minime (LGM)

(pag. 1356) B. hialinoza segmentara si focala (HSF)


C. nefropatia cu Ig A
D. glomerulonefrita extramembranoasa (GEM)
E. diabet.
(pag. 1365)

Pag. 123 din 130


*5 Care din urmatoarele enunturi referitoare la 10 Diagnosticul de sindrom hemolitic-uremic se
nefropatia cu Ig A este FALS : formuleaza daca sunt prezente:
A. este cea mai frecventa forma de GN cronica, cu A. anemie hemolitica
exceptia diabetului B. trombocitoza
B. se intalneste la persoane varstnice, de sex
C. insuficienta renala acuta cu hematurie si
feminin
proteinurie
C. prezinta hematurie +++, proteinurie cu debit
D. hipocalcemie
variabil, HTA, IRC
E. trombopenie prin consum
D. apar depozite mezangiale de IgA la IF
(pag. 1369)
E. 30% din cazuri evolueaza spre IRC in 20 de ani.
(pag. 1365) Tema nr. 90
Anemia
*6 Care este criteriul care pemite diferentierea
nefropatiilor glomerulare proliferative de cele Bibliografie asociata temei:
neproliferative: Book Des ECN, editia in limba romana, redactor
A. prezenta/absenta hematuriei > 100000 /ml Laurent Karila, 2011
pag. 1376-1378
B. HTA
C. insuficienta renala *1 Anemia este definita de:
D. gradul proteinuriei A. paloarea cutaneo-mucoasa
E. edemele. B. scaderea nivelului de hemoglobina sub 12 g/dl la
(pag. 1365) femeie si sub 13 g/dl la barbat
C. scaderea VEM sub 80 fl
*7 Selectati enuntul referitor la sindromul Alport
care este FALS; D. scaderea sideremiei

A. boala ereditara legata de o anomalie de E. prezenta asteniei


structura a colagenului IV (pag. 1376)
B. cea mai frecventa forma de glomerulonefrita *2 In anemia microcitara volumul eritrocitar
cronica, cu exceptia diabetului zaharat mediu (VEM) este:
C. sindrom glomerular, hipoacuzie de perceptie,
A. ≤ 80 fl
uneori afectare oculara
B. 81 - 85 fl
D. transmitere autosomal dominanta legata de
cromozomul X in 85 % din cazuri C. 85 - 90 fl
E. evolutie spre insuficienta renala cronica D. 90 -100 fl
terminala in special la femei. E. > 100 fl
(pag. 1366) (pag. 1376)
8 Manifestarile clinico-biologice sugestive *3 Cele doua examinari cheie care ajuta la
pentru stenoza de artera renala sunt precizarea cauzei anemiei microcitare sunt:
urmatoarele:
A. electroforeza de Hb si PCR
A. HTA severa, rezistenta la tripla terapie
B. feritinemia si PCR
B. agravarea unei insuficiente renale cronice dupa
introducerea tratamentului cu IEC / ARA 2 C. sideremia si reticulocitele
C. proteinurie peste 3,5 g/24 ore D. frotiul sanguin si haptoglobina
D. hiperpotasemie E. bilirubina si testul Coombs
E. suflu abdominal (pag. 1376)
(pag. 1367) *4 Explorarea fundamentala care stabileste
caracterul regenerativ sau aregenerativ al
9 Selectati nefropatiile vasculare cronice: unei anemii normo- sau macrocitare este:
A. stenoza de artera renala A. haptoglobina
B. nefroangioscleroza maligna B. feritinemia
C. sindromul hemolitic-uremic C. numarul de reticulocite
D. nefroangioscleroza benigna
D. testul Coombs
E. boala emboliilor cu cristale de colesterol
E. sideremia
(pag. 1367-1368)
(pag. 1377)
Pag. 124 din 130
*5 Urmarirea eficacitatii tratamentului cu fier se 10 Tratamentul carentei martiale presupune:
realizeaza prin determinarea:
A. tratament etiologic
A. bilirubinei neconjugate B. administrarea de fier per os 200 mg/zi, in cure de
B. PCR 4 luni
C. electroforezei de Hb C. administrarea de fier per os 100 mg/zi, in cure de
4 luni
D. feritinemiei
D. urmarirea eficacitatii tratamentului cu fier prin
E. haptoglobinei
determinarea feritinemiei
(pag. 1378)
E. urmarirea eficacitatii tratamentului prin
6 In cazul unei anemii microcitare (VEM ≤ 80 fl) efectuarea mielogramei
ne gandim in principal la o : (pag. 1378)
A. anemie hemolitica
Tema nr. 91
B. anemie carentiala martiala Anomalii ale hemostazei şi coagulării
C. anemie posthemoragica acuta Bibliografie asociata temei:
D. anemie inflamatorie Book Des ECN, editia in limba romana, redactor
E. anemie prin deficit de folati Laurent Karila, 2011
(pag. 1376) pag. 1383-1384

7 Cauzele principale ale unei anemii *1 Timpul de protrombina Quick (PT)


aregenerative normocitare sunt: exploreaza:
A. insuficienta renala cronica A. hemostaza primara
B. inflamatia cronica B. hemostaza secundara, calea intrinseca a
coagularii
C. mieloame, limfoame
C. hemostaza secundara, calea extrinseca a
D. leucemia acuta coagularii
E. carenta de vitamina B12 si folati D. hemostaza secundara, calea comuna
(pag. 1378) E. fibrinoliza
8 Principalele cauze ale carentei martiale sunt: (pag. 1383)
A. leucemiile acute *2 Factorii plasmatici ai coagularii dependenti
B. deficitul absorbtiei fierului de vitamina K sunt:
C. sangerarile de cauza ginecologica A. I, V, VIII, XI
D. patologia digestiva B. II, VII, IX, X
E. sangerarile voluntare C. II,V,XI, XII
(pag. 1377) D. I, V, VIII, XIII

9 Care din urmatoarele afirmatii sunt adevarate E. II, VIII, XI, XII
in cazul unei anemii de origine centrala? (pag. 1383)
A. reticulocite < 150.000/mmc *3 Timpul de protrombina Quick (PT)
B. reticulocite > 150.000/mmc exploreaza activitatea factorilor:
C. anemie aregenerativa A. II, V, VII, X
D. anemie regenerativa B. I, VIII, IX, X
E. necesita mielograma pentru precizarea C. II, V, IX, XI
diagnosticului D. I, V, VIII, XII
(pag. 1378)
E. II, VIII, IX, XIII
(pag. 1383)

Pag. 125 din 130


*4 Timpul de cefalina activata (TCA) exploreaza: 9 Cauzele frecvente in care PT este scazut si
TCA prelungit sunt:
A. hemostaza primara
B. hemostaza secundara, calea extrinseca a A. insuficienta hepatica
coagularii B. boala von Willebrand
C. hemostaza secundara, calea intrinseca a C. coagularea intravasculara diseminata
coagularii
D. hemofilia A
D. hemostaza secundara, calea comuna
E. administrarea de AVK
E. fibrinoliza (pag. 1383)
(pag. 1383)
10 In coagularea intravasculara diseminata
*5 Testele care exploreaza hemostaza primara (CIVD) se intalnesc:
sunt:
A. PT scazut
A. PT, fibrinogen
B. fibrinogen scazut
B. TCA, D- dimeri
C. trombopenie
C. fibrinogen, dozare FXIII
D. D-dimeri crescuti
D. numar trombocite, timp de sangerare/PFA
E. TCA normal
E. PT, TCA (pag. 1384)
(pag. 1383)
Tema nr. 92
6 In deficitul de factor VII plasmatic :
Mielomul multiplu
A. PT este normal Bibliografie asociata temei:
B. TCA este normal Book Des ECN, editia in limba romana, redactor
C. PT este scazut Laurent Karila, 2011
D. TCA este prelungit pag. 1390-1394

E. PT este crescut *1 Cel mai frecvent tip de mielom multiplu, in


(pag. 1383) functie de imunoglobulina monoclonala
secretata, este:
7 In hemofilia A :
A. mielomul multiplu tip IgA
A. factorul VIII plasmatic este scazut
B. mielomul multiplu tip IgG
B. factorul IX plasmatic este scazut
C. mielomul multiplu tip IgD
C. PT este normal
D. mielomul multiplu tip IgE
D. TCA este prelungit
E. mielomul multiplu tip Ig M
E. PT este scazut
(pag. 1390)
(pag. 1383)
*2 Procentul minim al plasmocitozei medulare
8 In cazul unui PT normal si TCA prelungit pentru stabilirea diagnosticului de mielom
examinarile care trebuie efectuate cu multiplu este:
prioritate sunt:
A. ≥ 10%
A. determinarea nivelului de F VIII
B. ≥ 20%
B. determinarea nivelului de F IX
C. ≥ 30%
C. determinarea nivelului de F V
D. ≥ 40%
D. cautarea unui anticoagulant circulant
E. ≥ 50%
E. determinarea de D-dimeri
(pag. 1393)
(pag. 1383)

Pag. 126 din 130


*3 Criteriile CRAB in mielomul multiplu se 7 Cauzele sindromului hemoragic din
refera la prezenta: mielomul multiplu sunt reprezentate de :
A. hiperuricemie, insuficienta renala, amiloidoza, A. trombopenie
leziuni osoase B. deficit de F XIII
B. hipercalcemie, insuficienta renala, anemie,
C. trombopatie indusa de hiperproteinemie
leziuni osoase
D. dereglari ale coagularii induse de proprietatile
C. hiperproteinemie, insuficienta respiratorie,
imunoglobulinei monoclonale
anemie, neuropatie
E. activitate anti-X a amiloidozei AL
D. hipercalcemie, radiculalgie, artropatie, sindrom
infectios (pag. 1391)
E. hipervascozitate,imunodepresie, atrofie cutanata, 8 Complicatiile determinate de componentul
endocrinopatie monoclonal in mielomul multiplu sunt:
(pag. 1392)
A. hipercalcemia
*4 Factorul de prognostic negativ legat de B. insuficienta renala
gazda in mielomul multiplu se refera la :
C. complicatiile tromboembolice
A. nivelul β2 microglobulinei crescut D. complicatiile neurologice
B. hipercalcemie E. sindromul de hipervascozitate
C. varsta inaintata (pag. 1392)
D. chimiorezistenta
9 Conform clasificarii lui Salmon si Durie
E. LDH crescuta privind evaluarea masei tumorale, mielomul
(pag. 1394) multiplu in stadiul III necesita prezenta unuia
din urmatoarele criterii
*5 Diagnosticul de mielom indolent (smoldering
myeloma) se stabileste pe baza urmatoarelor A. hemoglobina < 8,5 g/dl
criterii: B. calcemie > 3 mmol/l
A. prezenta in ser si/sau urina a unei proteine C. anomalii cromozomiale
monoclonale < 3g/100ml + plasmocitoza D. multiple leziuni litice osoase
medulara ≥10% + prezenta a cel putin unuia din
criteriile CRAB E. niveluri ridicate de Ig monoclonale:Ig > 70g/L,
IgA >50g/L, Ig monoclonal urinar ≥12 g/24 ore
B. prezenta unei proteine monoclonale in ser <
3g/100ml + plasmocitoza medulara < 10% + (pag. 1394)
absenta criteriilor CRAB
10 Diagnosticul de gamapatie monoclonala cu
C. prezenta unei proteine monoclonale in ser ≥ semnificatie nedeterminata (MGUS) se
3g/100ml si/sau a unei plasmocitoze medulare ≥ stabileste pe baza urmatoarelor criterii:
10% + absenta criteriilor CRAB
A. prezenta unei proteine monoclonale in ser ≥
D. localizare unica a plasmocitelor monoclonale la 3g/100ml
nivelul oaselor/tisular + maduva osoasa normala
B. prezenta unei proteine monoclonale in ser <
E. localizare unica a plasmocitelor monoclonale la 3g/100ml
nivel osos/tisular + absenta criteriilor CRAB
C. plasmocitoza medulara < 10%
(pag. 1393)
D. plasmocitoza medulara ≥ 10%
6 Diagnosticul de mielom multiplu se
E. absenta criteriilor CRAB
stabileste pe baza urmatoarelor criterii:
(pag. 1393)
A. prezenta in ser si/sau urina a unei proteine
monoclonale ( cu exceptia unui mielom
nesecretant)
B. plasmocitoza medulara < 10%
C. plasmocitoza medulara ≥ 10%
D. prezenta a cel putin unuia din criteriile CRAB
E. hiperuricemie
(pag. 1393)

Pag. 127 din 130


Tema nr. 93 *4 Urmatoarele afirmatii sunt adevarate despre
Cancerul: cancerogeneza, dezvoltare tumorala, clasificarea stadiala a cancerului:
clasificare, factorii de risc preventia si depistarea A. Clasificarea FIGO se utilizeaza pentru cancerul
Bibliografie asociata temei: colo-rectal
Book Des ECN, editia in limba romana, redactor B. Clasificarea Dukes se utilizeaza pentru
Laurent Karila, 2011 cancerului colului uterin
pag. 1421-1425, 1426-1428 * fara subcapitolul C. Invazia ganglionara se exprima prin simbolul N
epidemiologie pag 1420-1421, fara ultimul paragraf
D. Clasificarea Breslow se utilizeaza pentru
carcinoamele cutanate spinocelulare
*1 Cancerul:
E. Extensia metastatica se exprima prin simbolul T
A. Este o boala mono-factoriala
(pag. 1424)
B. Are origine monoclonala
C. Un cancer cu determinism genetic se dezvolta *5 Urmatoarele afirmatii privitoare la factorii de
intr-o prima etapa somatica risc pentru aparitia si dezvoltarea cancerului
sunt adevarate:
D. Este necesara o perioada de 5 luni pana la 1 an
pentru a se dezvolta de la prima celula pana la A. Fumatul pasiv este responsabil de peste 4000
manifestarea clinica de decese anuale in Franta
E. Un cancer sporadic se dezvolta intr-o prima B. Riscul de a dezvolta cancer pulmonar scade cu
etapa germinala 25% in cazul in care doar partenerul de viata
fumeaza iar persoana in cauza nu fumeaza.
(pag. 1421-1422)
C. Dintre factorii profesionali arsenicul este cel mai
*2 Alterari moleculare implicate in aparitia si frecvent intalnit (in Franta, responsabil de peste
dezvoltarea cancerului: 550 de cazuri de mezoteliom)
A. Inactivarea cailor de transductie a semnalului de D. Alimentatia joaca un rol probabil in aparitia
proliferare celulalara cancerului de san
B. Eliminarea rezistentei la “moarte celulara” E. Infectia cu virusul papiloma uman (HPV) este
(apoptoza) responsabila de aparitia limfoamelor
C. Inhibitia neo-angiogenezei (pag. 1426-1427)
D. Potential de imortalizare prin inactivarea 6 Alterari moleculare implicate in aparitia si
telomerazei dezvoltarea cancerului:
E. Independenta in raport cu semnalele de inhibare A. Activarea cailor de transductie a semnalului de
a cresterii proliferare celulalara
(pag. 1422)
B. Rezistenta la “moarte celulara” (apoptoza)
*3 Urmatoarele afirmatii sunt adevarate despre C. Potentialul de neo-angiogenezei
clasificarea cancerelor in functie de origine:
D. Independenta in raport cu semnalele de inhibare
A. Leiomiosarcomul se dezvolta din tesutul a cresterii
muscular striat E. Potential de imortalizare prin inactivarea
B. Rabdomiosarcomul se dezvolta din tesutul telomerazei
muscular neted (pag. 1422)
C. Fibrosarcomul se dezvolta din tesutul conjunctiv
7 Urmatoarele afirmatii sunt adevarate despre
D. Carcinoamele paramalpighiene se dezvolta din clasificarea cancerelor in functie de origine:
tesut adipos
A. Adenocarcinoamele se dezvolta la nivelul
E. Adenocarcinoamele se dezvolta din epiteliul epiteliului glandular
malpighian
B. Carcinoamele paramalpighiene sunt
(pag. 1423)
caracteristice tractului urinar
C. Leiomiosarcoamele se dezvolta la nivelul
musculaturii neted
D. Neuroblastoamele sunt tumori cu origine
ectodermica
E. Carcinoamele epidermoide sunt tumori
mezoectodermice
(pag. 1423)

Pag. 128 din 130


8 Urmatoarele afirmatii sunt adevarate despre Tema nr. 94
clasificarea cancerelor in functie de origine: Leucemii (acute si cronice) si limfoame maligne
A. Fibrosarcoamele se dezvolta la nivelul tesutului Bibliografie asociata temei:
conjunctiv
Book Des ECN, editia in limba romana, redactor
B. Rabdomiosarcoamele se dezvolta la nivelul Laurent Karila, 2011
muschiului neted pag. 1434-1435, 1436-1437, 1438-1439
C. Liposarcoamele sunt tumori caracteristice
tesutului adipos *1 Pentru stabilirea diagnosticului de leucemie
D. Nefroblastoamele sunt tumori cu origine acuta procentul de blasti la nivel medular
ectodermica trebuie sa fi cel putin:

E. Adenocarcinoamele sunt caracteristice tractului A. 10%


urinar B. 20%
(pag. 1423) C. 30%
9 Urmatoarele afirmatii privitoare la factorii de D. 40%
risc pentru aparitia si dezvoltarea cancerului E. 50%
sunt adevarate:
(pag. 1434)
A. Dintre factorii profesionali aminele aromatice
sunt cel mai frecvent intalnit (in Franta, *2 Leucemia limfoida cronica este o proliferare
responsabil de peste 2.200 de cazuri de cancer limfoida monoclonala constituita din:
pulmonar)
A. limfocite mature cu fenotip B in 95% din cazuri
B. Fumatul pasiv este responsabil pentru 2500 -
B. limfocite mature cu fenotip T in 95% din cazuri
3000 de decese anuale in Franta
C. limfocite imature cu fenotip B in 5% din cazuri
C. Hipoestrogenemia relativa creste riscul de
cancer de san D. limfocite imature cu fenotip T in 95% din cazuri
D. Infectia cu virusul papiloma uman (HPV) este E. limfocite T citotoxice in 95% din cazuri
responsabila de aparitia unora dintre cancerele (pag. 1436)
din sfera ORL
E. Cancerul de san sporadic este legat de prezenta *3 In leucemia limfoida cronica mielograma
onco-genelor BRCA1 si BRCA2 arata o infiltare cu :
(pag. 1426-1428) A. plasmocite
B. limfocite mici
10 Urmatoarele afirmatii privitoare la factorii de
risc pentru aparitia si dezvoltarea cancerului C. limfoblasti
sunt adevarate: D. prolimfocite
A. Alimentatia joaca un rol probabil in aparitia E. promielocite
cancerului gastric
(pag. 1436)
B. Obezitatea (mai ales dupa menopauza) este un
factor de risc relativ pentru aparitia cancerului de *4 In limfomul de manta este prezenta
san urmatoarea anomalie citogenetica:
C. Fumatul este responsabil de aproximativ 25 – A. t(15;17)
30% de decese prin cancer
B. inv.16
D. Alcoolul este un factor de risc independent
pentru aparitia si dezvoltarea cancerelor C. t(14;18)

E. Infectiile cu virusurile HTLV1 si/sau HTLV2 sunt D. t(11;14)


responsabile de aparitia cancerului colului uterin E. t(9;22)
(pag. 1426-1427) (pag. 1438)

*5 Limfoamele maligne in stadiul II, conform


clasificarii Ann Arbor, prezinta:
A. un singur teritoriu ganglionar afectat
B. cel putin doua teritorii afectate de aceeasi parte
a diafragmei
C. afectare ganglionara supra- si subdiafragmatica
D. afectare viscerala (hepatica, pulmonara)
E. afectare medulara
(pag. 1439)
Pag. 129 din 130
6 Anomaliile citogenetice cu prognostic
favorabil in leucemiile acute sunt:
A. t(8;21)
B. t(15;17)
C. cariotipurile complexe
D. inv.16
E. t(16;16)
(pag. 1435)

7 In leucemiile acute semnele clinice


nespecifice consecinta a insuficientei
medulare sunt reprezentate de :
A. anemie
B. neutropenie si infectii
C. sindrom hemoragic
D. adenopatii
E. hepato-splenomegalie
(pag. 1434)

8 Pacientii cu leucemie limfoida cronica stadiul


B conform clasificarii Binet prezinta:
A. mai putin de trei arii ganglionare afectate
B. mai mult de trei arii ganglionare afectate
C. prognostic intermediar
D. anemie si/sau trombopenie
E. supravietuire medie de 6 ani
(pag. 1436)

9 Diagnosticul si prognosticul in leucemiile


acute se bazeaza pe:
A. examinarea morfologica a blastilor din sange si
maduva osoasa hematogena
B. imunofenotipare
C. examen citogenetic
D. studiu molecular
E. biopsie ganglionara
(pag. 1434)

10 Care din urmatoarele afirmatii sunt adevarate


in cazul limfomului Burkitt?
A. afecteaza copii sau adulti tineri
B. este o urgenta terapeutica
C. prezinta anomalia citogenetica t(9;22)
D. in cazul unei mase tumorale mari exista un risc
crescut de liza tumorala
E. se insoteste de plasmocitoza medulara
(pag. 1438,1439)

Pag. 130 din 130

S-ar putea să vă placă și